SlideShare uma empresa Scribd logo
1 de 98
Baixar para ler offline
UNIVERSIDADE TUIUTI DO PARANÁ
ERIC LUIZ CAETANO
FELIPE TOLEDO DE ALMEIDA
HELENA LORUSSO
JÚLIO CÉSAR DROSZCZAK
LISTA DE EXERCÍCIOS – ELEMENTOS DE MÁQUINAS
CURITIBA
2015
ERIC LUIZ CAETANO
FELIPE TOLEDO DE ALMEIDA
HELENA LORUSSO
JÚLIO CÉSAR DROSZCZAK
LISTA DE EXERCÍCIOS – ELEMENTOS DE MÁQUINAS
Trabalho apresentado ao Curso de Engenharia
Mecânica, da Universidade Tuiuti do Paraná,
como requisito avaliativo do 1º bimestre da
disciplina de Elementos de Maquinas I.
Professor: Paulo Lagos
CURITIBA
2015
SUMÁRIO
MOVIMENTO CIRCULAR..............................................................................8
EXERCÍCIO 01 –.........................................................................................8
EXERCÍCIO 1.1 (Eric) –..............................................................................9
EXERCÍCIO 1.2 (Felipe) –........................................................................10
EXERCÍCIO 1.3 (Helena) - .......................................................................11
EXERCÍCIO 1.4 (Júlio) –..........................................................................11
EXERCÍCIO 02 –.......................................................................................12
EXERCÍCIO 2.1 (Eric) –............................................................................13
EXERCÍCIO 2.2 (Felipe) –........................................................................14
EXERCÍCIO 2.3 (Helena) - .......................................................................14
EXERCÍCIO 2.4 (Júlio) –..........................................................................15
EXERCÍCIO 03 –.......................................................................................16
EXERCÍCIO 3.1 (Eric) –............................................................................17
EXERCÍCIO 3.2 (Felipe) –........................................................................17
EXERCÍCIO 3.3 (Helena) - .......................................................................17
EXERCÍCIO 3.4 (Júlio) –..........................................................................18
RELAÇÃO DE TRANSMISSÃO...................................................................19
EXECÍCIO 04 – .........................................................................................19
EXERCÍCIO 4.1 (Eric) –............................................................................21
EXERCÍCIO 4.2 (Felipe) –........................................................................22
EXERCÍCIO 4.3 (Helena) - .......................................................................24
EXERCÍCIO 4.4 (Júlio) –..........................................................................26
EXERCÍCIO 05 –.......................................................................................27
EXERCICIO 5.1 (Eric) –............................................................................30
EXERCICIO 5.2 (Felipe) –........................................................................32
EXERCÍCIO 5.3 (Helena) - .......................................................................34
EXERCÍCIO 5.4 (Júlio) –..........................................................................36
TORÇÃO SIMPLES .....................................................................................39
EXERCÍCIO 06 –.......................................................................................39
EXERCÍCIO 6.1 (Eric) –............................................................................39
EXERCICIO 6.2 (Felipe) –........................................................................40
EXERCÍCIO 6.3 (Helena) - .......................................................................40
EXERCÍCIO 6.4 (Júlio) –..........................................................................40
EXERCÍCIO 07 –.......................................................................................41
EXERCÍCIO 7.1 (Eric) –............................................................................41
EXERCICIO 7.2 (Felipe) –........................................................................42
EXERCÍCIO 7.3 (Helena) - .......................................................................42
EXERCÍCIO 7.4 (Júlio) –..........................................................................43
TORQUE NAS TRANSMISSÕES ................................................................43
EXERCÍCIO 08 –.......................................................................................43
EXERCÍCIO 8.1 (Eric) –............................................................................44
EXERCICIO 8.2 (Felipe) –........................................................................45
EXERCÍCIO 8.3 (Helena) - .......................................................................46
EXERCÍCIO 8.4 (Júlio) –..........................................................................46
POTÊNCIA ...................................................................................................47
EXERCÍCIO 09 –.......................................................................................47
EXERCÍCIO 9.1 (Eric) –............................................................................48
EXERCICIO 9.2 (Felipe) –........................................................................49
EXERCÍCIO 9.3 (Helena) - .......................................................................49
EXERCÍCIO 9.4 (Júlio) –..........................................................................50
EXERCÍCIO 10 –.......................................................................................51
EXERCÍCIO 10.1 (Eric) –..........................................................................51
EXERCICIO 10.2 (Felipe) –......................................................................52
EXERCÍCIO 10.3 (Helena) - .....................................................................53
EXERCÍCIO 10.4 (Júlio) –........................................................................53
EXERCÍCIO 11 –.......................................................................................54
EXERCÍCIO 11.1 (Eric) –..........................................................................54
EXERCICIO 11.2 (Felipe) –......................................................................55
EXERCÍCIO 11.3 (Helena) - .....................................................................55
EXERCÍCIO 11.4 (Júlio) –........................................................................56
EXERCÍCIO 12 –.......................................................................................56
EXERCÍCIO 12.1 (Eric) –..........................................................................57
EXERCICIO 12.2 (Felipe) –......................................................................57
EXERCÍCIO 12.3 (Helena) - .....................................................................58
EXERCÍCIO 12.4 (Júlio) –........................................................................58
EXERCÍCIO 13 –.......................................................................................59
EXERCÍCIO 13.1 (Eric) –..........................................................................61
EXERCICIO 13.2 (Felipe) –......................................................................63
EXERCÍCIO 13.3 (Helena) - .....................................................................65
EXERCÍCIO 13.4 (Júlio) –........................................................................67
EXERCÍCIO 14 –.......................................................................................69
EXERCÍCIO 14.1 (Eric) –..........................................................................72
EXERCICIO 14.2 (Felipe) –......................................................................75
EXERCÍCIO 14.3 (Helena) - .....................................................................78
EXERCÍCIO 14.4 (Júlio) –........................................................................81
RELAÇÃO DE TRANSMISSÃO POR ENGRENAGENS .............................84
EXERCÍCIO 15 –.......................................................................................84
EXERCÍCIO 15.1 (Eric) –..........................................................................87
EXERCICIO 15.2 (Felipe) -.......................................................................89
EXERCÍCIO 15.3 (Helena) –.....................................................................91
EXERCÍCIO 15.4 (Júlio) –........................................................................93
REFERÊNCIAS ............................................................................................97
ÍNDICE DE FIGURAS
FIGURA 1 .......................................................................................................8
FIGURA 2 .......................................................................................................9
FIGURA 3 .....................................................................................................10
FIGURA 4 .....................................................................................................11
FIGURA 5 .....................................................................................................12
FIGURA 6 .....................................................................................................13
FIGURA 7 .....................................................................................................14
FIGURA 8 .....................................................................................................14
FIGURA 9 .....................................................................................................15
FIGURA 10 ...................................................................................................16
FIGURA 11 ...................................................................................................18
FIGURA 12 ...................................................................................................18
FIGURA 13 ...................................................................................................19
FIGURA 14 ...................................................................................................21
FIGURA 15 ...................................................................................................23
FIGURA 16 ...................................................................................................24
FIGURA 17 ...................................................................................................26
FIGURA 18 ...................................................................................................28
FIGURA 19 ...................................................................................................30
FIGURA 20 ...................................................................................................32
FIGURA 21 ...................................................................................................34
FIGURA 22 ...................................................................................................37
FIGURA 23 ...................................................................................................39
FIGURA 24 ...................................................................................................40
FIGURA 25 ...................................................................................................41
FIGURA 26 ...................................................................................................42
FIGURA 27 ...................................................................................................42
FIGURA 28 ...................................................................................................43
FIGURA 29 ...................................................................................................43
FIGURA 30 ...................................................................................................44
FIGURA 31 ...................................................................................................45
FIGURA 32 ...................................................................................................46
FIGURA 33 ...................................................................................................47
FIGURA 34 ...................................................................................................48
FIGURA 35 ...................................................................................................49
FIGURA 36 ...................................................................................................50
FIGURA 37 ...................................................................................................50
FIGURA 38 ...................................................................................................51
FIGURA 39 ...................................................................................................52
FIGURA 40 ...................................................................................................52
FIGURA 41 ...................................................................................................53
FIGURA 42 ...................................................................................................56
FIGURA 43 ...................................................................................................57
FIGURA 44 ...................................................................................................58
FIGURA 45 ...................................................................................................59
FIGURA 46 ...................................................................................................61
FIGURA 47 ...................................................................................................63
FIGURA 48 ...................................................................................................67
FIGURA 49 ...................................................................................................69
FIGURA 50 ...................................................................................................73
FIGURA 51 ...................................................................................................75
FIGURA 52 ...................................................................................................81
FIGURA 53 ...................................................................................................84
FIGURA 54 ...................................................................................................89
FIGURA 55 ...................................................................................................94
MOVIMENTO CIRCULAR
EXERCÍCIO 01 –
A roda da figura possui d = 300 mm e gira com velocidade angular ω = 10π
rad/s.
FIGURA 1
FONTE: MELCONIAN. SARKIS: Elementos de Máquinas – 9 Ed. Revisada. Editora Érica. Ano 2009.
Determine:
a) Período (T)
𝑇 =
2𝜋
𝜔
𝑇 =
2𝜋
10𝜋
𝑇 =
1
5
𝑠 = 0,2 𝑠
b) Frequência (f)
𝑓 =
1
𝑇
𝑓 =
1
0,2
= 5 𝐻𝑧
c) Rotação (n)
𝑛 = 60𝑓
𝑛 = 60 . 5
𝑛 = 300 𝑟𝑝𝑚
d) Velocidade Periférica (𝒱p)
𝑟 =
𝑑
2
𝑟 =
0,3
2
𝑟 = 0,15 𝑚
𝒱𝑝 = 𝜔. 𝑟
𝒱𝑝 = 10𝜋 . 0,15
𝒱𝑝 = 1,5𝜋 𝑚/𝑠 = 4,71 𝑚/
EXERCÍCIO 1.1 (Eric) –
Uma partícula se movendo em MCU (Movimento Circular Uniforme)
completa uma volta a cada 10 segundos em uma circunferência de diâmetro d= 10
cm. Determine:
FIGURA 2
FONTE: Autor Júlio César Droszczak.
a) Período (T)
O enunciado nos diz que a partícula completa uma volta a cada 10 segundos, logo o
período (T)= 10 s.
b) Velocidade angular (𝜔)
𝑇 =
2𝜋
𝜔

10 =
2𝜋
𝜔

𝜔 =
2𝜋
10
= 0,2𝜋 rad/s

c) Frequência (f)

𝑓 =
1
𝑇
𝑓 =
1
10
= 0,1 𝐻𝑧

d) Rotação (n)
𝑛 = 60𝑓
𝑛 = 60 . 0,1 = 6 𝑟𝑝𝑚
e) Velocidade Periférica (Vp)
𝑟 =
𝑑
2
𝑟 =
10
2
= 5𝑐𝑚 = 0,05𝑚
𝑉𝑝 = 𝜔. 𝑟
𝑉𝑝 = 0,2𝜋 . 0,05 = 0,01𝜋 𝑚 𝑠⁄ 𝑜𝑢 0,031 𝑚/𝑠
EXERCÍCIO 1.2 (Felipe) –
Uma polia que gira no sentido horário e cujo diâmetro é d = 500mm, sua
rotação é de n = 600rpm. Determine: a) Frequência; b) Velocidade periférica;
Velocidade angular e d) Período.
FIGURA 3
FONTE: http://www.casadaspolias.com.br/produtos/especificacoes-tecnicas.html
a) Frequência (𝑓)
𝑛 = 60. 𝑓
600 = 60. 𝑓
𝑓 =
600
60
= 10 𝐻𝑍
b) Velocidade periférica (𝑉𝑝)
𝑟 =
𝑑
2
=
500
2
= 250 𝑚𝑚
𝑟 = 0,25 𝑚
( 𝑉𝑝) = 20 . 0,25 = 5𝜋
𝑚
𝑠
𝑜𝑢 15,7
𝑚
𝑠
c) Período (𝑇)
𝑇 =
1
𝑓
=
1
10
= 0,1𝑠
d) Velocidade angular (ω)
𝜔 =
2𝜋
𝑇
=
2𝜋
0,1
= 62,83
𝑟𝑎𝑑
𝑠
EXERCÍCIO 1.3 (Helena) -
A roda trabalha numa rotação n=1710rpm. Determine:
a) Velocidade Angular (𝜔)
𝜔 =
1710𝜋
30
𝜔 =
1710𝜋
30
𝜔 = 57𝜋 𝑟𝑎𝑑/𝑠
b) Período (T)
𝑇 =
2𝜋
𝜔
𝑇 =
2𝜋
57𝜋
𝑇 = 0.035𝑠
c) Frequência (𝑓)
𝑓 =
1
𝑇
𝑓 =
1
0,035
𝑓 = 28,5 𝐻𝑧
EXERCÍCIO 1.4 (Júlio) –
Uma pedra de esmeril de d = 120 mm e acionada por um motor de rotação n
= 1200 rpm. Determine:
FIGURA 4
FONTE: http://www.vegamaquinas.com.br/retifica-para-torno-pr-1313-229827.htm
a) Velocidade angular (ω)
𝜔 =
𝜋𝑛
30
𝜔 =
𝜋1200
30
𝜔 = 40𝜋 𝑟𝑎𝑑/𝑠
b) Velocidade Periférica (𝒱p)
𝒱𝑝 = 𝜔. 𝑟
𝒱𝑝 = 40𝜋. 0.06
𝒱𝑝 = 2.4𝜋 𝑚/𝑠
c) Período (T)
𝑇 =
2𝜋
𝜔
𝑇 =
2𝜋
40𝜋
𝑇 =
1
20
= 0,05 𝑠
d) Frequência (f)
𝑓 =
1
𝑇
𝑓 =
1
0,05
𝑓 = 20 𝐻𝑧
EXERCÍCIO 02 –
O motor elétrico possui como característica de desempenho a rotação n =
1740rpm. Determine as seguintes características de desempenho do motor:
FIGURA 5
FONTE: MELCONIAN. SARKIS: Elementos de Máquinas – 9 Ed. Revisada. Editora Érica. Ano 2009.
a) Velocidade Angular (ω)
𝜔 =
𝜋𝑛
30
𝜔 =
1470𝜋
30
𝜔 = 58𝜋 𝑟𝑎𝑑/𝑠
b) Período (T)
𝑇 =
2𝜋
𝜔
𝑇 =
2𝜋
58𝜋
𝑇 =
1
29
= 0,0345 𝑠
c) Frequência (f)
𝑓 =
1
𝑇
𝑓 =
1
0,0345
𝑓 = 29 𝐻𝑧
EXERCÍCIO 2.1 (Eric) –
(Cefet-SP) Um motor executa 600 rpm. Determine a frequência e o período
no SI. Determine também a velocidade angular.
FIGURA 6
FONTE: MELCONIAN. SARKIS: Elementos de Máquinas – 9 Ed. Revisada. Editora Érica. Ano 2009.
a) Frequência (f)
𝑛 = 60𝑓
600 = 60𝑓
𝑓 =
600
60
= 10 𝐻𝑧
b) Período (T)
𝑓 =
1
𝑇
10 =
1
𝑇
𝑇 =
1
10
= 0,1 𝑠
c) Velocidade Angular (ω)

𝑇 =
2𝜋
𝜔
0,1 =
2𝜋
𝜔
 =
2𝜋
0,1
= 20𝜋 𝑟𝑎𝑑/𝑠
EXERCÍCIO 2.2 (Felipe) –
Um motor elétrico tem como característica um período de T= 0,029s.
Determine as seguintes características de desempenho deste motor: a) Frequência
(𝑓); b) Rotação (𝑛) e c) Velocidade angular ().
FIGURA 7
FONTE: MELCONIAN. SARKIS: Elementos de Máquinas – 9 Ed. Revisada. Editora Érica. Ano 2009.
a) Frequência (𝑓)
𝑓 =
1
𝑇
= 0,029 = 34,48 𝐻𝑧
b) Rotação (𝑛)
𝑛 = 60. 𝑓
𝑛 = 60.34,48 = 2069 𝑟𝑝𝑚
c) Velocidade angular (𝑤)
𝜔 =
2𝜋
𝑇
=
2𝜋
0,029
= 216,7
𝑟𝑎𝑑
𝑠
𝑜𝑢 68,96 𝑟𝑎𝑑/𝑠
EXERCÍCIO 2.3 (Helena) -
O motor elétrico possui como característica de desempenho a rotação n =
2730rpm. Determine as seguintes características de desempenho do motor:
FIGURA 8
FONTE: MELCONIAN. SARKIS: Elementos de Máquinas – 9 Ed. Revisada. Editora Érica. Ano 2009.
d) Velocidade Angular (ω)
𝜔 =
𝜋𝑛
30
𝜔 =
2730𝜋
30
𝜔 = 91𝜋 𝑟𝑎𝑑/𝑠
e) Período (T)
𝑇 =
2𝜋
𝜔
𝑇 =
2𝜋
91𝜋
𝑇 =
1
45.5
= 0,022 𝑠
f) Frequência (f)
𝑓 =
1
𝑇
𝑓 =
1
0,022
𝑓 = 45.45 𝐻𝑧
EXERCÍCIO 2.4 (Júlio) –
O pneu de um carro gira a uma rotação por minuto de n = 793 rpm .
Determine as seguintes características de desempenho do carro:
FIGURA 9
FONTE: http://pt.clipartlogo.com/premium/detail/car-or-truck-tire-line-art_109442720.html
a) Velocidade Angular (ω)
𝜔 =
𝜋𝑛
30
𝜔 =
𝜋793
30
𝜔 =
793𝜋
30
𝑟𝑎𝑑/𝑠
b) Período (T)
𝑇 =
2𝜋
𝜔
𝑇 =
2𝜋
793𝜋
30
𝑇 = (2𝜋)(
30
793𝜋
)
𝑇 =
60
793
= 0,0756 𝑠
c) Frequência (f)
𝑓 =
1
𝑇
𝑓 =
1
0,0756
𝑓 = 13,23 𝐻𝑧
EXERCÍCIO 03 –
O ciclista monta uma bicicleta aro 26 (d = 660 mm), viajando com um
movimento que faz com que as rodas girem n = 240 rpm. Qual a velocidade do
ciclista?
FIGURA 10
FONTE: MELCONIAN. SARKIS: Elementos de Máquinas – 9 Ed. Revisada. Editora Érica. Ano 2009.
Velocidade Periférica (𝒱p)
𝒱p =
𝜋. 𝑛. 𝑟
30
𝒱p =
𝜋. 240.0,33
30
𝒱p = 8,29𝑚/𝑠
Transformando para km/h:
𝒱p = 8,29 𝑥 3,6
𝒱p = 30𝑘𝑚/ℎ
EXERCÍCIO 3.1 (Eric) –
Um carrinho de rolimã foi construído com rodas de raio = 15 cm. Em uma
descida, as rodas do carrinho atingiram 600 rpm. Determine a velocidade que o
carrinho atingiu à essa rotação.
𝑉 =
𝜋. 𝑛. 𝑟
30
𝑉 =
𝜋 . 600 . 0,15
30
𝑉 = 9,42 𝑚/𝑠
Ou
𝑉 = 33,91 𝑘𝑚/ℎ
EXERCÍCIO 3.2 (Felipe) –
Um motoqueiro passeia em sua moto, sendo que o diâmetro de suas rodas é
de (d= 720 mm), neste trajeto o movimento que faz com que as rodas girem a uma
frequência de 6,83 HZ. Qual é a velocidade do motoqueiro? E qual é a sua rotação?
𝑛 = 60. 𝑓
𝑛 = 60 . 6,83 = 410 𝑟𝑝𝑚
𝑇 =
1
𝑓
=
1
6,83
= 0,146𝑠
𝜔 =
2𝜋
𝑇
=
2𝜋
0,146
= 43,03 𝑟𝑎𝑑/𝑠
𝑉𝑝 = 𝑤. 𝑟
𝑟 =
720
2
=
360
1000
= 0,36 𝑚
𝑉𝑝 = 43,03 .0,36 = 15,49
𝑚
𝑠
𝑜𝑢 55,76 𝑘𝑚/ℎ
EXERCÍCIO 3.3 (Helena) -
Um ventilador de D = 522mm, trabalhando com um movimento circular que
faz com que as pás girem a n = 28500 rpm. Qual a velocidade periférica do
ventilador?
FIGURA 11
FONTE: http://portuguese.alibaba.com/product-gs/hot-new-products-for-2015-kitchen-outer-rotor-
450mm-ac-axial-fan-60210360658.html
Velocidade Periférica (𝒱p)
𝒱p =
𝜋. 𝑛. 𝑟
30
𝒱p =
𝜋. 2850.0,261
30
𝒱p = 77,9𝑚/𝑠
Transformando para km/h:
𝒱p = 77,9 𝑥 3,6
𝒱p = 280,42𝑘𝑚/ℎ
EXERCÍCIO 3.4 (Júlio) –
Uma Roda D’água de diâmetro D = 236,22in, gira com uma rotação de n =
27 rpm. Qual a velocidade da Roda D’água? (1in = 25,4mm).
FIGURA 12
FONTE: http://www.cepa.if.usp.br/energia/energia1999/Grupo2B/Hidraulica/roda.htm.
Transformando Polegadas em metros:
1in = 25,4mm
236,22 x 25,4 = 5999,999mm ≅ 6m
Velocidade Periférica (𝒱p)
𝒱 =
𝜋. 𝑛. 𝑟
30
𝒱 =
𝜋. 27.3
30
𝒱 =
81𝜋
30
𝒱 = 8,48 𝑚/𝑠
𝒱 = 30,54 𝑘𝑚/ℎ
RELAÇÃO DE TRANSMISSÃO
EXECÍCIO 04 –
A transmissão por correias é composta por duas polias com os seguintes
diâmetros, respectivamente:
Polia 1 (motora) – d1=100mm
Polia 2 (movida) – d2=180mm
FIGURA 13
FONTE: MELCONIAN. SARKIS: Elementos de Máquinas – 9 Ed. Revisada. Editora Érica. Ano 2009.
A polia 1 atua com velocidade angular ω1 = 39π rad/s.
Determinar:
a) Período da polia 1 (T1)
𝑇1 =
2𝜋
𝜔
𝑇1 =
2𝜋
39𝜋
𝑇1 = 0,051𝑠
b) Frequência da polia 1 (f1)
𝑓1 =
1
𝑇1
𝑓1 =
1
0,051
𝑓1 = 19,5 𝐻𝑧
c) Rotação da polia 1 (n1)
𝑛1 = 60. 𝑓1
𝑛1 = 60 . 19,5
𝑛1 = 1170 𝑟𝑝𝑚
d) Velocidade Angular da polia 2 (ω2)
𝜔2 =
𝜔1 𝑑1
𝑑2
𝜔2 =
39𝜋 . 100
180
𝜔2 = 21,67𝜋 𝑟𝑎𝑑/𝑠
e) Frequência da polia 2 (f2)
𝑓2 =
1
𝑇2
𝑓2 =
1
0,092
𝑓2 ≅ 10,835 𝐻𝑧
f) Período da polia 2 (T2)
𝑇2 =
2𝜋
𝜔2
𝑇2 =
2𝜋
21,67𝜋
𝑇2 ≅ 0,092 𝑠
g) Rotação da polia 2 (n2)
𝑛2 = 60. 𝑓2
𝑛2 = 60 . 10,835
𝑛2 = 650 𝑟𝑝𝑚
h) Velocidade periférica da transmissão (𝒱p)
𝒱𝑝 = 𝜔. 𝑟
𝒱𝑝 ≅ 21,67𝜋 . 0,09
𝒱𝑝 ≅ 6,127 𝑚/𝑠
i) Relação de transmissão (i)
𝐼 =
𝑑2
𝑑1
𝐼 =
180
100
𝐼 = 1,8
EXERCÍCIO 4.1 (Eric) –
Uma transmissão por correias ampliadora de velocidade possui as seguintes
características:
Polia 1 motora 𝑑1 = 160 𝑚𝑚
Polia 2 movida 𝑑2 = 140 𝑚𝑚
A polia 1 atua com velocidade angular 𝜔1 = 50𝜋 𝑟𝑎𝑑/𝑠.
FIGURA 14
FONTE: Autor Eric Luiz Caetano.
Determine:
a) Período da polia 1 (𝑇1)
𝑇1 =
2𝜋
𝜔1
𝑇1 =
2𝜋
50𝜋
=
2
50
𝑠 𝑜𝑢 0,04 𝑠
b) Frequência da polia 1 (𝑓1)
𝑓1 =
1
𝑇1
𝑓1 =
1
0,04
= 25 𝐻𝑧
c) Rotação da polia 1 (𝑛1)
𝑛1 = 60𝑓1
𝑛1 = 60 . 25 = 1500 𝑟𝑝𝑚
d) Velocidade angular da polia 2 (𝜔2)
𝜔2 =
𝜔1 𝑑1
𝑑2
𝜔2 =
50𝜋 . 160
140
= 57,14𝜋 𝑟𝑎𝑑/𝑠
e) Frequência da polia 2 (𝑓2)
𝑓2 =
𝜔2
2𝜋
𝑓2 =
57,14𝜋
2𝜋
= 28,57 𝐻𝑧
f) Período da polia 2 (𝑇2)
𝑇2 =
2𝜋
𝜔2
𝑇2 =
2𝜋
57,14𝜋
= 0,035 𝑠
g) Rotação da polia 2 (𝑛2)
𝑛2 =
𝑛1 𝑑1
𝑑2
𝑛2 =
1500 . 160
140
= 1714,28 𝑟𝑝𝑚
h) Velocidade periférica da transmissão (Vp)
𝑉𝑝 = 𝜔1 𝑟1
𝑉𝑝 = 50𝜋 . 0,08 = 4𝜋 𝑚 𝑠⁄ 𝑜𝑢 12,56 𝑚/𝑠
i) Relação de transmissão (i)
𝐼 =
𝑑2
𝑑1
𝐼 =
140
160
= 0,875
EXERCÍCIO 4.2 (Felipe) –
A transmissão por correias é composta por duas polias, cujo seus diâmetros
são:
Polia 1 Motora d1 = 50 mm
Polia 2 Movida d2 = 80 mm
FIGURA 15
FONTE: MELCONIAN. SARKIS: Elementos de Máquinas – 9 Ed. Revisada. Editora Érica. Ano 2009.
a) Período da polia (𝑇1)
𝑇1 =
2𝜋
𝜔1
=
2𝜋
28𝜋
= 0,0714 𝑠
b) Frequência da polia ( 𝑓1):
𝑓1 =
1
𝑇1
=
28
2
= 14 𝐻𝑧
c) Rotação da polia (𝑁1):
𝑁1 = 60. 𝑓1
𝑁1 = 60 . 14 = 840 𝑟𝑝𝑚
d) Velocidade angular da polia 2 (𝜔2):
𝜔2 = 𝜔1.
𝑑1
𝑑2
= 28𝜋.
50
80
= 17,5 𝜋
𝑟𝑎𝑑
𝑠
e) Frequência da polia 2 (𝑓2):
𝑓2 =
𝜔2
2𝜋
=
17,5𝜋
2𝜋
= 8,75 𝐻𝑧
f) Período da polia 2 (𝑇2):
𝑇2 =
2𝜋
17,5𝜋
= 0,114 𝑠
g) Rotação da polia 2 (𝑛2):
𝑛2 = 𝑛1.
𝑑1
𝑑2
=
840.50
80
= 525 𝑟𝑝𝑚
h) Velocidade periférica (𝑉𝑝):
𝑉𝑝 = 𝜔1.
𝑑1
2
=
28.0,05
2
= 0,7
i) Relação de transmissão (𝑖)
𝑖 =
𝑑2
𝑑1
=
80
50
= 1,6
EXERCÍCIO 4.3 (Helena) -
Uma transmissão por correias composta por duas polias:
Polia 1 – d1=120mm
Polia 2 – d2=220mm
A polia 1 atua com rotação n=1140rpm. Determine:
FIGURA 16
FONTE: Autor Júlio César Droszczak
a) Velocidade Angular da polia 1 (𝜔1)
𝜔1 =
𝜋𝑛
30
𝜔1 =
1140𝜋
30
𝜔1 = 38𝜋 𝑟𝑎𝑑/𝑠
b) Frequência da polia 1 (𝑓1)
𝑓1 =
1
𝑇1
𝑇1 =
2𝜋
𝜔1
𝑓1 =
𝜔1
2𝜋
𝑓1 =
38𝜋
2𝜋
𝑓1 = 19 𝐻𝑧
c) Período da polia 1 (𝑇1)
𝑇1 =
2𝜋
𝜔1
𝑇1 =
2𝜋
38𝜋
𝑇1 = 0,0526𝑠
d) Velocidade Angular da polia 2 (𝜔2)
𝜔2 =
𝜔1 𝑑1
𝑑2
𝜔2 =
38𝜋 . 120
220
𝜔2 =
38𝜋 . 120
220
𝜔2 = 20,727𝜋 𝑟𝑎𝑑/𝑠
e) Período da polia 2 (𝑇2)
𝑇2 =
2𝜋
𝜔2
𝑇2 =
2𝜋
20,727𝜋
𝑇1 = 0,096𝑠
f) Frequência da polia 2 (𝑓2)
𝑓2 =
1
𝑇2
𝑓2 =
1
0,096
𝑓2 = 10,36 𝐻𝑧
g) Rotação da polia 2 (𝑛2)
𝑛2 = 60𝑓
𝑛2 = 60 . 10,36
𝑛2 = 621,6𝑟𝑝𝑚
h) Velocidade periférica da transmissão (Vp)
𝑉𝑝 = 𝜔1. 𝑟1
𝑉𝑝 = 38𝜋 . 0,06
𝑉𝑝 = 7,16 𝑚/𝑠
i) Relação de transmissão (I)
𝐼 =
𝑑2
𝑑1
𝐼 =
220
120
𝐼 = 1,83
EXERCÍCIO 4.4 (Júlio) –
Um motor que esta chavetado a uma polia de d1=160mm de diâmetro,
desenvolve n1=1200 rpm e move um eixo de transmissão cuja polia tem d2=300mm
de diâmetro. Calcule:
FIGURA 17
FONTE: MELCONIAN. SARKIS: Elementos de Máquinas – 9 Ed. Revisada. Editora Érica. Ano 2009.
a) Frequência da polia 1 (f1)
𝑛1 = 60. 𝑓1
1200 = 60 . 𝑓1
𝑓1 =
1200
60
𝑓1 = 20 𝐻𝑧
b) Período da polia 1 (T1)
𝑓1 =
1
𝑇1
20 =
1
𝑇1
𝑇1 =
1
20
𝑇1 = 0.05 𝑠
c) Velocidade Angular da polia 1 (ω 1)
𝜔1 =
𝜋. 1200
30
𝜔1 =
𝜋. 1200
30
𝜔1 = 40𝜋 𝑟𝑎𝑑/𝑠
d) Velocidade Angular da polia 2 (ω2)
𝜔2 =
𝜔1 𝑑1
𝑑2
𝜔2 =
40𝜋 . 160
300
𝜔2 = 21.33𝜋 𝑟𝑎𝑑/𝑠
e) Período da polia 2 (T2)
𝑇2 =
2𝜋
𝜔2
𝑇2 =
2𝜋
21,33𝜋
𝑇2 ≅ 0,094 𝑠
f) Frequência da polia 2 (f2)
𝑓2 =
1
𝑇2
𝑓2 =
1
0,094
𝑓2 ≅ 10,638 𝐻𝑧
g) Rotação da polia 2 (n2)
𝑛2 = 60. 𝑓2
𝑛2 = 60 . 10,638
𝑛2 = 638.28 𝑟𝑝𝑚
h) Velocidade periférica da transmissão (𝒱p)
𝒱𝑝 = 𝜔. 𝑟
𝒱𝑝 ≅ 21,33𝜋 . 0,15
𝒱𝑝 ≅ 10,05 𝑚/𝑠
i) Relação de transmissão (i)
𝐼 =
𝑑2
𝑑1
𝐼 =
300
160
𝐼 = 1,875
EXERCÍCIO 05 –
A transmissão por correias de um motor a combustão para automóvel, que
aciona simultaneamente as polias da bomba de água e do alternador: d1 = 120 mm
[Motor]; d2 = 90 mm [Bomba D`água]; d3 = 80 mm [Alternador]. A velocidade
econômica do motor ocorre a rotação de n1 = 2800 rpm.
FIGURA 18
FONTE: MELCONIAN. SARKIS: Elementos de Máquinas – 9 Ed. Revisada. Editora Érica. Ano 2009.
Nessa condição pode-se determinar:
Polia 1 (Motor).
a) Velocidade angular (ω1)
𝜔1 =
𝜋𝑛1
30
𝜔1 =
𝜋2800
30
𝜔1 = 93,33𝜋 𝑟𝑎𝑑/𝑠 𝑜𝑢 293,2 𝑟𝑎𝑑/𝑠
b) Frequência (f1)
𝑓1 =
𝜔1
2𝜋
𝑓1 =
93,33𝜋
2𝜋
𝑓1 = 46,665 𝐻𝑧
Polia 2 (Bomba D`água).
c) Velocidade angular (ω2)
𝜔2 =
𝑑1 𝜔1
𝑑2
𝜔2 =
120𝑥93,33𝜋
90
𝜔2 = 124,44𝜋 𝑟𝑎𝑑/𝑠
d) Frequência (f2)
𝑓2 =
𝜔2
2𝜋
𝑓2 =
124,44𝜋
2𝜋
𝑓2 = 62,22 𝐻𝑧
e) Rotação (n2)
𝑛2 = 60𝑓2
𝑛2 = 60𝑥62,22
𝑛2 = 3733,2 𝑟𝑝𝑚
Polia 3 (Alternador).
f) Velocidade angular (ω3)
𝜔2 =
𝑑1 𝜔1
𝑑3
𝜔2 =
120𝑥93,33𝜋
80
𝜔2 = 140𝜋 𝑟𝑎𝑑/𝑠
g) Frequência (f3)
𝑓2 =
𝜔3
2𝜋
𝑓2 =
140𝜋
2𝜋
𝑓2 = 70 𝐻𝑧
h) Rotação (n3)
𝑛2 = 60𝑓3
𝑛2 = 60𝑥70
𝑛2 = 4200 𝑟𝑝𝑚
Transmissão
i) Velocidade periférica (𝒱p)
𝒱p = 𝜔1. 𝑟1
𝒱p = 93,33𝜋. 0,06
𝒱p ≅ 17,59 𝑚/𝑠
j) Relação de transmissão (Motor/Polia 1) (i1)
𝑖1 =
𝑑1
𝑑2
𝑖1 =
120
90
𝑖1 = 1,33
k) Relação de transmissão (Polia 2/Polia 3) (i3)
𝑖3 =
𝑑2
𝑑3
𝑖3 =
120
80
𝑖3 = 1,5
EXERCICIO 5.1 (Eric) –
Uma transmissão por correias de um automóvel possui as seguintes
características:
Polia 1 motor 𝑑1 = 160 𝑚𝑚
Polia 2 bomba d’ água 𝑑2 = 120 𝑚𝑚
Polia 3 alternador 𝑑3 = 110 𝑚𝑚
FIGURA 19
FONTE: Autor Eric Luiz Caetano.
Para a rotação constante de 3000 rpm do motor, determine:
a) Velocidade angular da polia 1 (𝜔1)
𝜔1 =
𝜋𝑛1
30
𝜔1 =
𝜋 . 3000
30
= 100𝜋 𝑟𝑎𝑑 𝑠⁄ 𝑜𝑢 314,15 𝑟𝑎𝑑/𝑠
b) Frequência da polia 1 (𝑓1)
𝑓1 =
𝜔1
2𝜋
𝑓1 =
100𝜋
2𝜋
= 50 𝐻𝑧
c) Velocidade angular da polia 2 (𝜔2)
𝜔2 =
𝜔1 𝑑1
𝑑2
𝜔2 =
100𝜋 . 160
120
= 133,3𝜋 𝑟𝑎𝑑 𝑠⁄ 𝑜𝑢 418,8 𝑟𝑎𝑑/𝑠
d) Frequência da polia 2 (𝑓2)
𝑓2 =
𝜔2
2𝜋
𝑓2 =
133,3𝜋
2𝜋
= 66,5 𝐻𝑧
e) Rotação da polia 2 (𝑛2)
𝑛2 = 60𝑓2
𝑛2 = 60 . 66,5 = 3990 𝑟𝑝𝑚
f) Velocidade angular da polia 3 (𝜔3)
𝜔3 =
𝜔1 𝑑1
𝑑3
𝜔3 =
100𝜋 . 160
110
= 145,4𝜋 𝑟𝑎𝑑 𝑠⁄ 𝑜𝑢 456,9 𝑟𝑎𝑑/𝑠
g) Frequência da polia 3 (𝑓3)
𝑓3 =
𝜔3
2𝜋
𝑓3 =
145,4𝜋
2𝜋
= 72,7 𝐻𝑧
h) Rotação da polia 3 (𝑛3)
𝑛3 = 60𝑓3
𝑛3 = 60 . 72,7 = 4362 𝑟𝑝𝑚
i) Velocidade Periférica (Vp)
𝑉𝑝 = 𝜔1. 𝑟1
𝑉𝑝 = 100𝜋 . 0,08 = 8𝜋 𝑚 𝑠⁄ 𝑜𝑢 25,13 𝑚/𝑠
j) Relação de Transmissão (𝑖1)
𝑖1 =
𝑑1
𝑑2
𝑖1 =
160
120
= 1,33 𝑜𝑢 33%
k) Relação de Transmissão (𝑖2)
𝑖2 =
𝑑1
𝑑3
𝑖2 =
160
110
= 1,45 𝑜𝑢 45%
l) Relação de Transmissão (𝑖3)
𝑖3 =
𝑑2
𝑑3
𝑖3 =
120
110
= 1,09 𝑜𝑢 9%
EXERCICIO 5.2 (Felipe) –
Uma transmissão por correias de um motor.
FIGURA 20
FONTE: MELCONIAN. SARKIS: Elementos de Máquinas – 9 Ed. Revisada. Editora Érica. Ano 2009.
d1: 150 mm (motor)
d2: 100 mm (Bomba d`água)
d3: 90 mm (alternador)
Sabe-se que a velocidade econômica do motor ocorre a uma rotação de n=
3000 rpm. Nessa condição podemos determinar:
a) Velocidade angular da polia 1 (𝜔1):
𝜔1 =
𝜋. 𝑛1
30
𝜔1 =
𝜋 . 3000
30
= 100𝜋 𝑟𝑎𝑑 𝑠⁄
b) Frequência da polia 1 (𝑓1):
𝑓1 =
𝜔1
2𝜋
𝑓1 =
100𝜋
2𝜋
= 50 𝐻𝑧
c) Velocidade angular da polia 2 (𝜔2):
𝜔2 =
𝜔1 𝑑1
𝑑2
𝜔2 =
150 . 100𝜋
100
𝜔2 = 150𝜋 𝑟𝑎𝑑/𝑠
d) Frequência da polia 2 (𝑓2):
𝑓2 =
𝜔2
2𝜋
𝑓2 =
150𝜋
2𝜋
= 75 𝐻𝑧
e) Rotação da polia 2 (𝑛2):
𝑛2 = 60𝑓2
𝑛2 = 60 .75 = 4500 𝑟𝑝𝑚
f) Velocidade angular da polia 3 (𝜔3):
𝜔3 =
𝜔1 𝑑1
𝑑3
𝜔3 =
100𝜋 . 150
90
= 166,66𝜋 𝑟𝑎𝑑 𝑠⁄
g) Frequência na polia 3 (𝑓3):
𝑓3 =
𝜔3
2𝜋
𝑓3 =
166,66𝜋
2𝜋
= 83,33 𝐻𝑧
h) Rotação da polia 3 (𝑛3):
𝑛3 = 60𝑓3
𝑛3 = 60 . 83,33 = 4999,8 𝑟𝑝𝑚
i) Velocidade periférica (𝑉𝑝):
𝑉𝑝 = 100𝜋. 0,075 = 7,5𝜋 𝑚/𝑠
j) Relação de Transmissão (i1):
𝑖1 =
𝑑1
𝑑2
𝑖1 =
150
100
= 1,5
k) Relação de Transmissão (𝑖2)
𝑖2 =
𝑑1
𝑑3
𝑖2 =
150
90
= 1,667
l) Relação de Transmissão (𝑖3)
𝑖3 =
𝑑2
𝑑3
𝑖3 =
100
90
= 1,111
EXERCÍCIO 5.3 (Helena) -
As polias de um motor à combustão são acionadas simultaneamente.
Polia 1 (motor) – d1=100mm
Polia 2 (bomba d’agua) – d2=80mm
Polia 3 (alternador) – d3=60mm
O motor trabalha numa rotação n=2000rpm.
FIGURA 21
FONTE: MELCONIAN. SARKIS: Elementos de Máquinas – 9 Ed. Revisada. Editora Érica. Ano 2009.
Determine:
a) Velocidade angular na polia 1 (𝜔1)
𝜔1 =
2000𝜋
30
𝜔1 = 66,67𝜋 𝑟𝑎𝑑/𝑠
b) Frequência na polia 1 (𝑓1)
𝑓1 =
1
𝑇
𝑓1 =
𝜔
2𝜋
𝑓1 =
66,67𝜋
2𝜋
𝑓1 = 33,3 𝐻𝑧
c) Velocidade angular na polia 2 (𝜔2)
𝜔2 =
𝜔1 𝑑1
𝑑2
𝜔2 =
66,67𝜋 . 100
80
𝜔2 = 83,34𝜋 𝑟𝑎𝑑/𝑠
d) Frequência na polia 2 (𝑓2)
𝑓2 =
1
𝑇
𝑓2 =
𝜔2
2𝜋
𝑓2 =
83,34𝜋
2𝜋
𝑓2 = 41,67 𝐻𝑧
e) Rotação na Polia 2 (𝑛2)
𝑛2 = 60𝑓2
𝑛2 = 60 . 41,47
𝑛2 = 2500,125 𝑟𝑝𝑚
f) Velocidade angular na polia 3 (𝜔3)
𝜔3 =
𝜔2 𝑑2
𝑑3
𝜔3 =
83,34𝜋 . 80
60
𝜔3 = 111,12𝜋 𝑟𝑎𝑑/𝑠
g) Frequência na polia 3 (𝑓3)
𝑓3 =
1
𝑇
𝑓3 =
𝜔3
2𝜋
𝑓3 =
111,12𝜋
2𝜋
𝑓3 = 55,56 𝐻𝑧
h) Rotação na Polia 3 (𝑛3)
𝑛3 = 60𝑓3
𝑛3 = 60 . 55,56
𝑛3 = 3333,6 𝑟𝑝𝑚
i) Velocidade Periférica (Vp)
𝑉𝑝 = 𝜔1. 𝑟1
𝑉𝑝 = 66,67𝜋. 0,5
𝑉𝑝 = 10,47 𝑚/𝑠
j) Relação de Transmissão (𝐼1)
𝐼1 =
𝑑1
𝑑2
𝐼1 =
100
80
𝐼1 = 1,25
k) Relação de Transmissão (𝐼2)
𝐼2 =
𝑑1
𝑑3
𝐼2 =
100
60
𝐼2 = 1,67
EXERCÍCIO 5.4 (Júlio) –
Um sistema de transmissão por correias de uma determinada máquina
movida por um motor elétrico chavetado a uma polia, move simultaneamente duas
outras polias de diâmetros d3=80mm; d2=100mm, com rotações n1=2700 e n2=8100
rpm. Determine:
FIGURA 22
FONTE: MELCONIAN. SARKIS: Elementos de Máquinas – 9 Ed. Revisada. Editora Érica.
Ano 2009.
Polia 1 (Motor):
a) Diâmetro Polia 1 (Motor) [d1]
𝑑1 =
𝑑2. 𝑛2
𝑛1
𝑑1 =
100.8100
2700
𝑑1 = 300𝑚𝑚
b) Velocidade angular (ω1)
𝜔1 =
𝜋𝑛1
30
𝜔1 =
𝜋. 2700
30
𝜔1 = 90𝜋 𝑟𝑎𝑑/𝑠
c) Frequência (f1)
𝑓1 =
𝜔1
2𝜋
𝑓1 =
90𝜋
2𝜋
𝑓1 = 45 𝐻𝑧
Polia 2:
d) Velocidade angular (ω2)
𝜔2 =
𝑑1 𝜔1
𝑑2
𝜔2 =
300.90𝜋
100
𝜔2 = 270𝜋 𝑟𝑎𝑑/𝑠
e) Frequência (f2)
𝑓2 =
𝜔2
2𝜋
𝑓2 =
270𝜋
2𝜋
𝑓2 = 135 𝐻𝑧
f) Rotação (n2)
𝑛2 = 60𝑓2
𝑛2 = 60.135
𝑛2 = 8100 𝑟𝑝𝑚
Polia 3:
g) Velocidade angular (ω3)
𝜔3 =
𝑑1 𝜔1
𝑑3
𝜔3 =
300.90𝜋
80
𝜔3 = 337,5𝜋 𝑟𝑎𝑑/𝑠
h) Frequência (f3)
𝑓3 =
𝜔3
2𝜋
𝑓3 =
337,5𝜋
2𝜋
𝑓3 = 168,75 𝐻𝑧
i) Rotação (n3)
𝑛3 = 60𝑓3
𝑛3 = 60.168,75
𝑛3 = 10125 𝑟𝑝𝑚
Características de transmissão:
j) Velocidade periférica (𝒱p)
𝒱p = 𝜔1. 𝑟1
𝒱p = 90𝜋. 0,15
𝒱p ≅ 42,41 𝑚/𝑠
k) Relação de transmissão (Motor/Polia 1) (i1)
𝑖1 =
𝑑1
𝑑2
𝑖1 =
300
100
𝑖1 = 3
l) Relação de transmissão (Polia 2/Polia 3) (i3)
𝑖3 =
𝑑2
𝑑3
𝑖3 =
100
80
𝑖3 = 1,25
TORÇÃO SIMPLES
EXERCÍCIO 06 –
Determinar torque de aperto na chave que movimenta as castanhas da placa
do torno. A carga aplicada nas extremidades da haste é F = 80N. O comprimento da
haste é L = 200mm.
FIGURA 23
FONTE: MELCONIAN. SARKIS: Elementos de Máquinas – 9 Ed. Revisada. Editora Érica.
Ano 2009.
𝑀 𝑇 = 2. 𝐹. 𝑆
𝑀 𝑇 = 2𝑥80𝑥100
𝑀 𝑇 = 16000𝑁. 𝑚𝑚
𝑀 𝑇 = 16𝑁. 𝑚
EXERCÍCIO 6.1 (Eric) –
Para se ter um torque correto no parafuso do volante de um motor 1.8 AP é
necessário aproximadamente 80 Nm. Com uma chave com hastes de 25 cm,
determine a força necessária que se deve aplicar nas duas extremidades da chave.
𝑀𝑡 = 2 . 𝐹 . 𝑆 
80 = 2 . 𝐹 . 0,25
𝐹 =
80
0,5
= 160 𝑁
EXERCICIO 6.2 (Felipe) –
Determine o torque de aperto na chave que realiza a abertura e fechamento
do parafuso por onde é retirado o óleo de uma motocicleta de uso urbano comum.
Sabendo que a carga aplicada nas extremidades da haste é 50 N, e o comprimento
da haste é de L=280 mm.
𝑀 𝑇 = 2 . 𝐹𝑠
𝑀 𝑇 = 2 . 50 . 140
𝑀 𝑇 = 14000 𝑁𝑚
Ou
𝑀 𝑇 = 14 𝑁𝑚
EXERCÍCIO 6.3 (Helena) -
Um mecânico precisa fazer a manutenção de um motor. Para abri-lo ele
precisará aplicar uma força de 30N numa chave de 150mm de comprimento.
Determine o torque aplicado pelo mecânico:
𝑀 𝑇 = 2 . 𝐹 . 𝑆 
𝑀 𝑇 = 2 . 30 . 0,15
𝑀 𝑇 = 9 𝑁𝑚
EXERCÍCIO 6.4 (Júlio) –
A figura representa a força aplicada na vertical, sobre uma chave de boca,
por um motorista de caminhão tentando desatarraxar uma das porcas que fixa uma
roda.
FIGURA 24
FONTE: http://pt.slideshare.net/CentroApoio/exercequilibrio-corpo-rigido.
O ponto de aplicação da força dista 150mm do centro da porca e o módulo
da força máxima aplicada é F = 400N. Nesta situação, suponha que o motorista está
próximo de conseguir desatarraxar a porca. Em seguida, o motorista acopla uma
extensão à chave de boca, de forma que o novo ponto de aplicação da força 750mm
do centro da porca. Calcule o novo valor do módulo da força F’, em Newtons,
necessária para que o motorista novamente esteja próximo de desatarraxar a porca.
𝑀 𝑇1
= 2. 𝐹1. 𝑆1
𝑀 𝑇1
= 2𝑥400𝑥150
𝑀 𝑇1
120000𝑁𝑚𝑚
𝑀 𝑇2
= 2. 𝐹2. 𝑆2
𝑀 𝑇2
= 2𝑥𝐹2 𝑥750
𝑀 𝑇2
= 1500𝐹2
𝑀 𝑇2
= 𝑀 𝑇1
1500𝐹2 = 120000
𝐹2 =
120000
1500
𝐹2 = 80𝑁
EXERCÍCIO 07 –
Determinar torque (MT) no parafuso da roda do automóvel. A carga aplicada
pelo operador em cada braço da chave é F = 120N. O comprimento dos braços é L =
200 mm.
FIGURA 25
FONTE: MELCONIAN. SARKIS: Elementos de Máquinas – 9 Ed. Revisada. Editora Érica. Ano 2009.
𝑀 𝑇 = 2. 𝐹. 𝑆
𝑀 𝑇 = 2𝑥120𝑥100
𝑀 𝑇 = 48000 𝑁. 𝑚𝑚
𝑀 𝑇 = 16 𝑁. 𝑚
EXERCÍCIO 7.1 (Eric) –
O manual de um certo veículo determina que o torque ideal para os
parafusos da roda é de 100 nm. A chave de roda que estava no conjunto do veículo
tem braços de 30 cm. Determine a força necessária que deve-se aplicar nas duas
extremidades da chave.
FIGURA 26
FONTE: Autor Eric Luiz Caetano.
𝑀 𝑇 = 2 . 𝐹 . 𝑆
100 = 2 . 𝐹 . 0,3
𝐹 =
100
0,6
= 166,6 𝑁
EXERCICIO 7.2 (Felipe) –
Dada à figura, determine o torque de aperto (𝑀 𝑇) no parafuso no trilho de
um elevador. A carga aplicada pelo operador em cada braço da chave é de F= 180
N, e o comprimento dos braços da chave são de L= 210 mm.
FIGURA 27
FONTE: Autor Felipe Toledo.
𝑀 𝑇 = 2 . F . L
𝑀 𝑇 = 2 . 180.210
𝑀 𝑇 = 75600 Nm
ou
𝑀 𝑇 = 75,6 Nm
EXERCÍCIO 7.3 (Helena) -
Para trocar o pneu de um carro é necessário levantá-lo com um “macaco”.
Dado o torque de 20Nm e o comprimento L= 200mm da manivela de acionamento
do levantador, determine a força aplicada na operação:
𝑀 𝑇 = 2 . 𝐹 . 𝐿 
20 = 2 . 𝐹 . 0,20
𝐹 =
20
0,4
= 50 𝑁
EXERCÍCIO 7.4 (Júlio) –
Para a figura abaixo calcule o torque provocado pela manivela de
comprimento L = 250mm, em relação ao centro do eixo e considerando a carga de
acionamento igual a F = 600N.
FIGURA 28
FONTE: http://formacaopiloto.blogspot.com.br/2014_06_01_archive.html.
𝑀 𝑇 = 2. 𝐹. 𝑆
𝑀 𝑇 = 2𝑥600𝑥250
𝑀 𝑇 = 300000 𝑁. 𝑚𝑚
𝑀 𝑇 = 300 𝑁. 𝑚
TORQUE NAS TRANSMISSÕES
EXERCÍCIO 08 –
A transmissão por correia é composta pela polia motora (1) que possui
diâmetro d1 = 100mm e a polia movida (2) que possui diâmetro d2 = 240mm. A
transmissão é acionada por uma força tangencial FT = 600N. Determinar:
FIGURA 29
FONTE: MELCONIAN. SARKIS: Elementos de Máquinas – 9 Ed. Revisada. Editora Érica.
Ano 2009.
a) Torque na Polia (1)
𝑟1 =
𝑑1
2
𝑟1 =
100
2
𝑟1 = 50𝑚𝑚
𝑟1 = 0.05𝑚
𝑀 𝑇 = 𝐹 𝑇. 𝑟1
𝑀 𝑇 = 600𝑥0,05
𝑀 𝑇 = 30𝑁𝑚
b) Torque na Polia (2)
𝑟1 =
𝑑2
2
𝑟1 =
240
2
𝑟1 = 120𝑚𝑚
𝑟1 = 0.12𝑚
𝑀 𝑇 = 𝐹 𝑇. 𝑟1
𝑀 𝑇 = 600𝑥0,12
𝑀 𝑇 = 72𝑁𝑚
EXERCÍCIO 8.1 (Eric) –
Uma transmissão por correia ampliadora de velocidade é movimentada por
uma força inicial tangencial de FT = 500 N. A polia motora dessa transmissão possui
um diâmetro de 150 mm e a polia movida possui um diâmetro de 100 mm.
Determine o torque na polia motora e na polia movida.
FIGURA 30
FONTE: Autor Júlio César Droszczak
𝑀𝑡1 = 𝐹𝑡. 𝑟1
𝑀𝑡1 = 500 . (
0,15
2
)
𝑀𝑡1 = 37,5 𝑁𝑚
𝑀𝑡2 = 𝐹𝑡. 𝑟2
𝑀𝑡2 = 500 . (
0,1
2
)
𝑀𝑡2 = 25 𝑁𝑚
EXERCICIO 8.2 (Felipe) –
A transmissão por correias, representada na figura, é composta pela polia
motora 1 que possui diâmetro de d1= 230 mm e a polia movida 2 possui diâmetro
d2= 500 mm. A transmissão será acionada por uma força tangencial 𝐹 𝑇=850 N.
FIGURA 31
FONTE: MELCONIAN. SARKIS: Elementos de Máquinas – 9 Ed. Revisada. Editora Érica. Ano 2009.
COM ALTERAÇÃO DO AUTOR.
Determine o torque na polia 1:
Determine o torque na polia 2:
RESOLUÇÃO:
a) Torque na polia 1:
Raio da polia 1:
𝑟1 =
𝑑1
2
=
230
2
= 115 𝑚𝑚
𝑟1 = 115 𝑚𝑚 𝑜𝑢 𝑟1 = 0,115𝑚
Torque na polia:
𝑀 𝑇1
= 𝐹 𝑇. 𝑟1
𝑀 𝑇1
= 850𝑁 . 0,115𝑚
𝑀 𝑇1
= 97,75 𝑁𝑚
b) Torque na polia 2:
Raio da polia 2:
𝑟2 =
𝑑2
2
=
500
2
= 250 𝑚𝑚
𝑟2 = 250𝑚𝑚 𝑜𝑢 𝑟2 = 0,25𝑚
Torque na polia
𝑀 𝑇2
= 𝐹 𝑇. 𝑟2
𝑀 𝑇2
= 850 .0,25 = 212,5 𝑁𝑚
EXERCÍCIO 8.3 (Helena) -
Uma transmissão por correia é movimentada por uma força inicial tangencial
de FT = 300N. A polia motora dessa transmissão possui um diâmetro de 250 mm e a
polia movida trabalha com um torque de 75Nm. Determine:
a) Torque na polia motora
𝑀 𝑇1
= 𝐹 𝑇. 𝑟1
𝑀 𝑇1
= 300 . (
0,25
2
)
𝑀 𝑇1
= 37,5 𝑁𝑚
b) Diâmetro da polia movida.
𝑀 𝑇1
= 𝐹 𝑇 . (
𝐷2
2
)
75 = 300 . (
𝐷2
2
)
𝐷2 = 0,50𝑚 = 500𝑚𝑚
EXERCÍCIO 8.4 (Júlio) –
A transmissão por correia é composta pela polia motora (A) que possui
diâmetro d1 = 80mm e a polia movida (B) que possui diâmetro d2 = 210mm. A
transmissão é acionada por uma força tangencial FT = 730N. Determinar:
FIGURA 32
FONTE: http://www.vdl.ufc.br/solar/aula_link/lfis/semestre01/Fisica_I/Aula_03/02.html
a) Torque na Polia (A)
𝑟1 =
𝑑1
2
𝑟1 =
80
2
𝑟1 = 40𝑚𝑚
𝑟1 = 0.04𝑚
𝑀 𝑇 = 𝐹 𝑇. 𝑟1
𝑀 𝑇 = 730𝑥0,04
𝑀 𝑇 = 29,2𝑁𝑚
b) Torque na Polia (B)
𝑟1 =
𝑑2
2
𝑟1 =
210
2
𝑟1 = 105𝑚𝑚
𝑟1 = 0.105𝑚
𝑀 𝑇 = 𝐹 𝑇. 𝑟1
𝑀 𝑇 = 730𝑥0,105
𝑀 𝑇 = 76,65𝑁𝑚
POTÊNCIA
EXERCÍCIO 09 –
O elevador projetado para transportar carga máxima CMÁX. = 7000N (10
pessoas). O peso do elevador é PE = 1KN e o contrapeso possui a mesma carga CP
= 1KN. Determine a potência do motor M para que o elevador se desloque com
velocidade constante V = 1m/s.
FIGURA 33
FONTE: MELCONIAN. SARKIS: Elementos de Máquinas – 9 Ed. Revisada. Editora Érica. Ano 2009.
Resolução:
O peso do elevador é compensado pelo contrapeso, eliminando o seu efeito.
Portanto. Para dimensionar a potência do motor, a carga a ser utilizada é CMÁX. =
7000N
Potência do motor (PMOTOR)
𝑃 𝑀𝑂𝑇𝑂𝑅 = 𝐹. 𝑉
𝑃 𝑀𝑂𝑇𝑂𝑅 = 7000𝑥1
𝑃 𝑀𝑂𝑇𝑂𝑅 = 7000𝑊
𝑃𝐶𝑉 =
𝑃 𝑊
735,5
𝑃𝐶𝑉 =
7000
735,5
𝑃𝐶𝑉 ≅ 9,5 𝐶𝑉
EXERCÍCIO 9.1 (Eric) –
Um elevador de veículos tem a capacidade máxima de 4800 N (cerca de 6
pessoas com massas de 80 kg). O elevador tem sem peso irrelevante, pois há um
contrapeso que possui o mesmo peso do elevador. O elevador possui uma
velocidade constante de 5 m/s. Determine a potência do motor que movimenta este
elevador.
FIGURA 34
FONTE: http://www.gpmotorsbrasil.com.br/info1.html
(Força de tração no cabo é igual à força peso da capacidade máxima:
4800N)
𝑃 𝑚𝑜𝑡𝑜𝑟 = 𝐹𝑐𝑎𝑏𝑜 . 𝑉
𝑃 𝑚𝑜𝑡𝑜𝑟 = 7000 .5
𝑃 𝑚𝑜𝑡𝑜𝑟 = 35000 𝑊 𝑜𝑢 35 𝐾𝑊
𝑃 𝑚𝑜𝑡𝑜𝑟 =
35000
735,5
= 47,28 𝐶𝑣
EXERCICIO 9.2 (Felipe) –
Em um elevador comum, cuja seu projeto tem como especificação de carga
máxima 560 kg (70 kg/por pessoa). Sabendo que o contra peso e a cabina possuem
a mesma carga de 1 kN.
Determine a potência do motor M para que o elevador de desloque com
velocidade constante de V= 2,5 m/s. Determine a potencia do motor.
FIGURA 35
FONTE: http://seguranca-na-construcao.dashofer.pt/?s=modulos&v=capitulo&c=7655
𝐾𝑔 𝑝𝑎𝑟𝑎 𝑁 = 𝐾𝑔. 100 = 560. 10 = 5600 𝑁
𝑃 𝑚𝑜𝑡𝑜𝑟 = 𝐹𝑐𝑎𝑏𝑜. 𝑉
𝑃 𝑚𝑜𝑡𝑜𝑟 = 5600 .2,5
𝑃 𝑚𝑜𝑡𝑜𝑟 = 14000 w
𝑊 𝑝𝑎𝑟𝑎 𝐶𝑉 =
𝑃( 𝑊)
735,5
=
14000
735,5
= 19 𝐶𝑉
EXERCÍCIO 9.3 (Helena) -
Um elevador de carga foi projetado para transportar carga máxima CMÁX. =
10kN. O peso do elevador é PE = 1,4KN e o contrapeso possui a mesma carga CP =
1,4KN. Determine a potência do motor M para que o elevador se desloque com
velocidade constante V = 0,8m/s.
FIGURA 36
FONTE:
http://www.pauluzzi.com.br/alvenaria.php?PHPSESSID=ccd0dd0c90aa9901b2a2e49d3182897c
𝑃 𝑀𝑂𝑇𝑂𝑅 = 𝐹. 𝑉
𝑃 𝑀𝑂𝑇𝑂𝑅 = 10000𝑥0,8
𝑃 𝑀𝑂𝑇𝑂𝑅 = 8000𝑊
𝑃𝐶𝑉 =
𝑃 𝑊
735,5
𝑃𝐶𝑉 =
8000
735,5
𝑃𝐶𝑉 ≅ 10,88 𝐶𝑉
EXERCÍCIO 9.4 (Júlio) –
Cada um dos dois motores a jato de um avião Boeing 767 desenvolve uma
propulsão (força que acelera o avião) igual a F = 197000N. Quando o avião esta
voando a V = 250m/s, qual a potência instantânea que cada motor desenvolve? Em
W e CV.
FIGURA 37
FONTE: http://pt.wikipedia.org/wiki/Turbina_aeron%C3%A1utica
𝑃 𝑀𝑂𝑇𝑂𝑅 = 𝐹. 𝑉
𝑃 𝑀𝑂𝑇𝑂𝑅 = 197000.250
𝑃 𝑀𝑂𝑇𝑂𝑅 = 49250000𝑊
𝑃𝐶𝑉 =
𝑃 𝑊
735,5
𝑃𝐶𝑉 =
49250000
735,5
𝑃𝐶𝑉 ≅ 66961,25 𝐶𝑉
EXERCÍCIO 10 –
Um servente de pedreiro erguendo uma lata de concreto com peso PC =
200N. A corda e a polia são ideais. A altura da laje é h = 8m, o tempo de subida é t =
20s. Determine a potência útil do trabalho do operador.
FIGURA 38
FONTE: MELCONIAN. SARKIS: Elementos de Máquinas – 9 Ed. Revisada. Editora Érica. Ano 2009.
R = Como a carga esta sendo elevada com movimento uniforme, conclui-se
que a aceleração do movimento é nula, portanto:
𝐹 𝑂𝑃𝐸𝑅𝐴𝐷𝑂𝑅. = 𝐹𝐶 = 200𝑁
𝑉𝑆𝑈𝐵𝐼𝐷𝐴 =
ℎ
𝑡
𝑉𝑆𝑈𝐵𝐼𝐷𝐴 =
8
20
𝑉𝑆𝑈𝐵𝐼𝐷𝐴 = 0,04 𝑚/𝑠
Potência útil do operador.
𝑃 = 𝐹 𝑂𝑃.. 𝑉𝑆𝑈𝐵.
𝑃 = 200𝑥0,4
𝑃 = 80𝑊
EXERCÍCIO 10.1 (Eric) –
Um motor de 5 KW está posicionado no quinto andar de um prédio e
trabalha para puxar uma carga P = 750N através de uma corda e uma polia.
Determine a velocidade de subida dessa carga.
FIGURA 39
FONTE: Autor Eric Luiz Caetano.
𝑃 𝑚𝑜𝑡𝑜𝑟 = 𝐹𝑐𝑎𝑟𝑔𝑎 . 𝑉
5000 = 750 . 𝑉
𝑉 =
5000
750
= 6,66 𝑚/𝑠
EXERCICIO 10.2 (Felipe) –
Em um canteiro de obra um servente de pedreiro ergue um balde de cimento
com peso 𝑃𝑐 = 10 𝐾𝑔. Considerando que tanto a polia quanto a corda serão
consideradas como ideais. A altura em que será levantado é de h= 5 metros, e sua
velocidade de subida de 0,556 m/s. Determine qual será o tempo de subida, e qual
será a potência útil do trabalho do operador.
FIGURA 40
FONTE: MELCONIAN. SARKIS: Elementos de Máquinas – 9 Ed. Revisada. Editora Érica. Ano
2009.COM ALTERAÇÃO DO AUTOR.
𝐾𝑔 𝑝𝑎𝑟𝑎 𝑁 = 10 . 10 = 100𝑁
𝐹𝑜 𝑝
= 𝐹𝑜𝑟ç𝑎 𝑎𝑝𝑙𝑖𝑐𝑎𝑑𝑎 𝑝𝑒𝑙𝑜 𝑜𝑝𝑒𝑟𝑎𝑑𝑜𝑟
𝑃𝑐 = 𝑃𝑒𝑠𝑜 𝑑𝑎 𝑏𝑎𝑙𝑑𝑒 𝑑𝑒 𝑐𝑖𝑚𝑒𝑛𝑡𝑜
𝐹𝑜 𝑝
= 𝐹𝑐 = 100𝑁
Tempo total de subida: (𝑡 𝑆)
𝑡 𝑠 =
ℎ
𝑉𝑠
=
5 𝑚
0,556 𝑚/𝑠
= 9 𝑠
Potência útil do operador:
𝑃 = 𝐹𝑜 𝑝
. 𝑉𝑠
𝑃 = 100 𝑁 .0,556
𝑚
𝑠
𝑃 = 55,56 𝑊
EXERCÍCIO 10.3 (Helena) -
Uma pessoa fazendo uma mudança para o segundo andar de um prédio
cujas escadas são muito estreitas, precisa puxar o sofá de peso P = 5000N pela
parte de fora do prédio. Dados altura até a janela h=12m e o tempo para colocar
para dentro de 2min, determine a potência útil do trabalho aplicado pela pessoa.
𝑉𝑆 =
ℎ
𝑡
𝑉𝑆 =
12
120
𝑉𝑆 = 0,1 𝑚/𝑠
Potencia útil
𝑃 = 𝐹𝑃𝐸𝑆... 𝑉𝑆𝑈𝐵.
𝑃 = 500𝑥0,1
𝑃 = 50𝑊
EXERCÍCIO 10.4 (Júlio) –
Uma pessoa erguendo um balde de água de um andar a outro com um peso
de P = 300N. A altura do andar é de h = 6m, o tempo necessário para erguer o balde
desde o andar de baixo até o andar de cima é de t = 27s. Determine a potência útil
de trabalho da pessoa. Considerando a corda e a polia como sendo ideais.
FIGURA 41
FONTE:http://crv.educacao.mg.gov.br/sistema_crv/index.aspx?ID_OBJETO=58360&tipo=ob
&cp=780031&cb=&n1=&n2=M%EF%BF%BDdulos%20Did%EF%BF%BDticos&n3=Ensino%20M%EF
%BF%BDdio&n4=F%EF%BF%BDsica&b=s.
𝐹𝑃𝐸𝑆𝑆𝑂𝐴. = 𝐹𝐶 = 300𝑁
𝑉𝑆𝑈𝐵𝐼𝐷𝐴 =
ℎ
𝑡
𝑉𝑆𝑈𝐵𝐼𝐷𝐴 =
8
20
𝑉𝑆𝑈𝐵𝐼𝐷𝐴 = 0,22 𝑚/𝑠
Potência útil do operador.
𝑃 = 𝐹𝑃𝐸𝑆... 𝑉𝑆𝑈𝐵.
𝑃 = 300𝑥0,22
𝑃 = 66,660𝑊
EXERCÍCIO 11 –
Um motor elétrico com potência P = 0,25W esta erguendo uma lata de
concreto com peso PC = 200N. A corda e a polia são ideais. A altura da laje é h =
8m. Determine:
a) Velocidade de subida da lata de concreto (VSUB.).
𝐹 𝑀𝑂𝑇𝑂𝑅. = 𝐹𝐶 = 200𝑁
𝑃 = 250𝑊
𝑃 = 𝐹 𝑀𝑂𝑇.. 𝑉𝑆𝑈𝐵.
𝑉𝑆𝑈𝐵. =
𝑃
𝐹 𝑀𝑂𝑇.
𝑉𝑆𝑈𝐵. =
250
200
𝑉𝑆𝑈𝐵. = 1,25𝑚/𝑠
b) Tempo de subida da lata (tSUB.).
𝑉𝑆𝑈𝐵. =
ℎ
𝑡 𝑆𝑈𝐵.
𝑡 𝑆𝑈𝐵. =
ℎ
𝑉𝑆𝑈𝐵.
𝑡 𝑆𝑈𝐵. =
8
1,25
𝑡 𝑆𝑈𝐵. = 6,4𝑠
EXERCÍCIO 11.1 (Eric) –
Para empurrar uma caixa, duas pessoas aplicam uma força de 800N de
modo que a caixa se move com velocidade constante em uma distância de 10
metros no tempo de 5 segundos. Desconsiderando o atrito, determinar a velocidade
em que a caixa se move e a potência útil das duas pessoas.
𝑣𝑚 =
∆𝑆
∆𝑡
𝑣𝑚 =
10
5
= 2 𝑚/𝑠
𝑃ú𝑡𝑖𝑙 = 𝐹 . 𝑣𝑚
𝑃ú𝑡𝑖𝑙 = 800 .2
𝑃ú𝑡𝑖𝑙 = 1600 𝑊
EXERCICIO 11.2 (Felipe) –
Seguindo a mesmo raciocínio do exercício anterior, substituiremos o
servente de pedreiro por um motor elétrico com potência de 0,43 KW. Determine:
a) Velocidade de subida do balde de concreto: ( 𝑉𝑠)
𝐾𝑊 𝑝𝑎𝑟𝑎 𝑊 = 𝐾𝑊 . 1000 = 0,43 . 1000 = 430 𝑊
𝐾𝑔 𝑝𝑎𝑟𝑎 𝑁 = 10 . 10 = 100𝑁
𝑉𝑠 =
𝑃 𝑚𝑜𝑡𝑜𝑟
𝐹𝑠𝑢𝑏𝑖𝑑𝑎
=
430𝑊
100𝑁
= 4,5 𝑚/𝑠
b) Tempo de subida do balde: ( 𝑡 𝑠)
𝑡 𝑠 =
ℎ
𝑉𝑠
=
5
4,5
= 1,11 𝑠
EXERCÍCIO 11.3 (Helena) -
Um andaime elétrico é acionado através de um motor de potencia P.
Considerando a altura h = 50m, velocidade v = 0,8m/s e o peso P = 200N,
determine:
a) Tempo de subida do andaime
𝑉𝑆𝑈𝐵. =
ℎ
𝑡 𝑆𝑈𝐵.
𝑡 𝑆𝑈𝐵. =
ℎ
𝑉𝑆𝑈𝐵.
𝑡 𝑆𝑈𝐵. =
50
0,8
𝑡 𝑆𝑈𝐵. = 62,5𝑠
b) Potencia do motor
𝑃 = 𝐹 𝑀𝑂𝑇.. 𝑉𝑆𝑈𝐵.
𝐹 𝑚𝑜𝑡. =
𝑃
𝑉𝑠𝑢𝑏.
𝐹 𝑚𝑜𝑡. =
200
0,8
𝐹 𝑚𝑜𝑡. = 250𝑊
EXERCÍCIO 11.4 (Júlio) –
(UNESP-SP) Um motor de potência útil P = 125W, funcionando como
elevador, eleva a altura h = 10m, com velocidade constante, um corpo de peso igual
a 50N.
FIGURA 42
FONTE: http://fisicaevestibular.com.br/exe_din_15.htm.
a) Velocidade de subida da lata de concreto (VSUB.).
𝐹 𝑀𝑂𝑇𝑂𝑅. = 𝐹𝐶 = 50𝑁
𝑃 = 125𝑊
𝑃 = 𝐹 𝑀𝑂𝑇.. 𝑉𝑆𝑈𝐵.
𝑉𝑆𝑈𝐵. =
𝑃
𝐹 𝑀𝑂𝑇.
𝑉𝑆𝑈𝐵. =
125
50
𝑉𝑆𝑈𝐵. = 2,5 𝑚/𝑠
b) Tempo de subida da lata (tSUB.).
𝑉𝑆𝑈𝐵. =
ℎ
𝑡 𝑆𝑈𝐵.
𝑡 𝑆𝑈𝐵. =
ℎ
𝑉𝑆𝑈𝐵.
𝑡 𝑆𝑈𝐵. =
10
2,5
𝑡 𝑆𝑈𝐵. = 4𝑠
EXERCÍCIO 12 –
Uma pessoa empurra o carrinho de supermercado, aplicando uma carga de
F = 150N, deslocando-se em um percurso de 42m no tempo de 1min. Determine a
potência que movimenta o veículo.
FIGURA 43
FONTE: MELCONIAN. SARKIS: Elementos de Máquinas – 9 Ed. Revisada. Editora Érica.
Ano 2009.
𝑉𝐶𝐴𝑅. =
𝑆
𝑡
𝑉𝐶𝐴𝑅. =
42
60
𝑉𝐶𝐴𝑅. = 0,7𝑚/𝑠
𝑃 = 𝐹. 𝑉𝐶𝐴𝑅.
𝑃 = 150𝑥0,7
𝑃 = 105𝑊
EXERCÍCIO 12.1 (Eric) –
Dois trabalhadores estão puxando do terceiro andar de um prédio uma caixa
com várias latas de concreto. O total em peso das latas e da caixa é de 800 N.
Supondo que não há perdas de potência pela corda e que a caixa sobe a uma
velocidade constante de 1,2 m/s, determine a potência útil de cada trabalhador,
supondo que ambos trabalham com a mesma intensidade.
𝑃 =
𝐹
2
. 𝑉𝑠
𝑃 =
800
2
. 1,2
𝑃 = 480 𝑊
EXERCICIO 12.2 (Felipe) –
Um cilindro hidráulico aplica a força de F= 320 N sobre um carro de
ferramentas em uma determinada fábrica, seu deslocamento durante um percurso
de 70 metros no tempo de 54 segundos. Determine a velocidade deste carrinho, e
qual será a potência que movimentará este carro de ferramentas.
1) Velocidade do carro de ferramentas: (𝑉𝑐)
𝑉𝑐 =
𝑠
𝑡
=
70𝑚
54𝑠
= 1,296
𝑚
𝑠
2) Potência do carro: ( 𝑃)
𝑃 = 𝐹 . 𝑉𝑐
𝑃 = 320 𝑁 . 1,296
𝑚
𝑠
𝑃 = 414,72 𝑊
EXERCÍCIO 12.3 (Helena) -
Uma diarista precisa movimentar um móvel 2,5 metros para conseguir fazer
a limpeza. Levando em consideração que ela fez isso em 15s e aplicou uma força de
100N, determine:
a) Velocidade deslocamento: (𝑉𝑑)
𝑉𝑑 =
𝑠
𝑡
=
2,5
15
= 0,167𝑚/𝑠
b) Potência do movimento: ( 𝑃)
𝑃 = 𝐹 . 𝑉𝑑
𝑃 = 100 . 0,167
𝑃 = 16,7 𝑊
EXERCÍCIO 12.4 (Júlio) –
Um trabalhador precisa movimentar um carrinho de mão cheio de pedras por
100m para descarregar, para isso ele aplica uma carga P = 350N, a potência que
movimenta o veículo é de 270W. Quanto tempo ele levará para chegar ao local
especificado para descarga?
FIGURA 44
FONTE:http://crv.educacao.mg.gov.br/sistema_crv/index.aspx?ID_OBJETO=58360&tipo=ob&cp=780
031&cb=&n1=&n2=M%EF%BF%BDdulos%20Did%EF%BF%BDticos&n3=Ensino%20M%EF%BF%B
Ddio&n4=F%EF%BF%BDsica&b=s
𝑃 = 𝐹. 𝑉𝐶𝐴𝑅.
𝑉𝐶𝐴𝑅. =
𝑃
𝐹
𝑉𝐶𝐴𝑅. =
270
350
𝑉𝐶𝐴𝑅. = 0,771 𝑚/𝑠
𝑉𝐶𝐴𝑅. =
𝑆
𝑡
𝑡 =
𝑆
𝑉𝐶𝐴𝑅.
𝑡 =
100
0,771
𝑡 = 129,7𝑠
EXERCÍCIO 13 –
A transmissão por Corrêa é acionada por um motor elétrico com potência de
P = 5,5KW, com rotação de n = 1720 rpm, chavetado a polia (1) do sistema. As
polias possuem respectivamente os seguintes diâmetros: d1 = 120mm [Polia (1)
Motora]; d2 = 300mm [Polia (2) Movida]. Desprezar as perdas. Determinar para a
transmissão:
FIGURA 45
FONTE: MELCONIAN. SARKIS: Elementos de Máquinas – 9 Ed. Revisada. Editora Érica. Ano 2009.
a) Velocidade angular da polia (1) [ω1]
𝜔1 =
𝑛𝜋
30
𝜔1 =
1720𝜋
30
𝜔1 = 57,33𝜋 𝑟𝑎𝑑/𝑠
b) Frequência da polia (1) [f1]
𝑓1 =
𝑛1
60
𝑓1 =
1720
60
𝑓1 = 28,66 𝐻𝑧
c) Torque da polia (1) [MT1]
A rotação da polia (1) é a mesma rotação do motor, pois a polia esta
chavetada no eixo-árvore do motor.
𝑀 𝑇1
=
𝑃
𝜔1
𝑀 𝑇1
=
5500
57,33𝜋
𝑀 𝑇1
= 30,5𝑁𝑚
d) Velocidade angular da polia (2) [ω2]
𝜔2 =
𝑑1. 𝜔1
𝑑2
𝜔2 =
120𝑥53,33𝜋
300
𝜔2 = 22,93𝜋 𝑟𝑎𝑑/𝑠
e) Frequência da polia (2) [f2]
𝑓2 =
𝜔2
2𝜋
𝑓2 =
22,93𝜋
2𝜋
𝑓2 = 11,465 𝐻𝑧
f) Torque da polia (2) [MT2]
𝑀 𝑇1
=
𝑃
𝜔2
𝑀 𝑇1
=
5500
22,93𝜋
𝑀 𝑇1
= 76,3𝑁𝑚
g) Rotação da polia (2) [n2]
𝑛2 = 60𝑓2
𝑛2 = 60𝑥11,465
𝑛2 = 688 𝑟𝑝𝑚
h) Relação de transmissão [i]
𝑖 =
𝑑2
𝑑1
𝑖 =
300
120
𝑖 = 2,5
i) Velocidade periférica da transmissão [VP]
A velocidade periférica da transmissão é a mesma da polia (1) com a da
polia (2), portanto podemos utilizar:
𝑉𝑃 = 𝜔1. 𝑟1 = 𝜔2. 𝑟2
𝑉𝑃 = 𝜔1. 𝑟1
𝑉𝑃 = 57,33𝜋𝑥0,06
𝑉𝑃 = 3,44𝜋 𝑚/𝑠
𝑉𝑃 = 10,8 𝑚/𝑠
j) Força tangencial [FT]
𝐹 𝑇 =
𝑀 𝑇1
𝑟1
=
𝑀 𝑇2
𝑟2
𝐹 𝑇 =
𝑀 𝑇1
𝑟1
𝐹 𝑇 =
30,5
0,06
𝐹 𝑇 = 508,3𝑁
EXERCÍCIO 13.1 (Eric) –
O pinhão de uma transmissão de uma moto é movimentado por um motor de
25 cv de potência. O pinhão possui um diâmetro de 100 mm e a coroa possui um
diâmetro de 200 mm. Para uma rotação de 2400 rpm, determine:
FIGURA 46
FONTE: Autor Eric Luiz Caetano.
a) Velocidade angular do pinhão
𝜔1 =
𝑛 . 𝜋
30
𝜔1 =
2400 . 𝜋
30
𝜔1 = 80𝜋 𝑟𝑎𝑑 𝑠⁄
b) Frequência do pinhão
𝑓1 =
𝑛
60
𝑓1 =
2400
60
𝑓1 = 40 𝐻𝑧
c) Torque do pinhão
𝑀𝑡 =
𝑃
𝜔1
𝑀𝑡 =
𝑃
80𝜋
𝑃 = 𝑃𝐶𝑣 .735,5
𝑃 = 25 .735,5
𝑃 = 18387,5 𝑊
𝑀𝑡 =
18387,5
80𝜋
𝑀𝑡 = 73,16 𝑁𝑚
d) Velocidade angular da coroa
𝜔2 =
𝜔1 𝑑1
𝑑2
𝜔2 =
80𝜋 . 100
200
𝜔2 = 40𝜋
e) Frequência da coroa
𝑓2 =
𝜔2
2𝜋
𝑓2 =
40𝜋
2𝜋
𝑓2 = 20 𝐻𝑧
f) Torque da coroa
𝑀𝑡 =
𝑃
𝜔2
𝑀𝑡 =
18387,5
40𝜋
𝑀𝑡 = 146,32 𝑁𝑚
g) Rotação da coroa
𝑛2 = 60 . 𝑓2
𝑛2 = 60 . 20
𝑛2 = 1200 𝑟𝑝𝑚
h) Relação de transmissão
𝑖 =
𝑑2
𝑑1
𝑖 =
200
100
𝑖 = 2
i) Velocidade periférica
𝑉𝑝 = 𝜔1 . 𝑟1
𝑉𝑝 = 80𝜋 . 0,05
𝑉𝑝 = 12,57 𝑚/𝑠
j) Força tangencial da transmissão
𝐹𝑡 =
𝑀𝑡1
𝑟1
𝐹𝑡 =
73,16
0,05
𝐹𝑡 = 1463,2 𝑁
EXERCICIO 13.2 (Felipe) –
É dada uma transmissão por correias, representada na figura, é acionada
por um motor elétrico com potência P= 4 kW com rotação n=1400 rpm, tendo em
vista que a rotação 𝑛1 será a mesma do motor pois a polia encontra-se chavetada ao
eixo arvore do motor. Como representada na figura a seguir:
FIGURA 47
FONTE: MELCONIAN. SARKIS: Elementos de Máquinas – 9 Ed. Revisada. Editora Érica. Ano 2009.
COM ALTERAÇÃO DO AUTOR.
Diâmetros das polias:
𝑑1 = 100 𝑚𝑚( 𝑑𝑖â𝑚𝑒𝑡𝑟𝑜 𝑑𝑎 𝑝𝑜𝑙𝑖𝑎 1)
𝑑2 = 280 𝑚𝑚 (𝑑𝑖â𝑚𝑒𝑡𝑟𝑜 𝑑𝑎 𝑝𝑜𝑙𝑖𝑎 2)
Determinar para transmissão:
a) Velocidade angular da polia 1 ( 𝜔1)
b) Frequência da polia 1 ( 𝑓1)
c) Torque da polia 1(𝑀 𝑇1
)
d) Velocidade angular da polia 2 ( 𝜔2)
e) Frequência da polia 2 ( 𝑓2)
f) Rotação da polia 2 ( 𝑛2)
g) Torque da polia 2 (𝑀 𝑇2
)
h) Relação de transmissão (i)
i) Velocidade periférica da transmissão (𝑉𝑝)
j) Força tangencial da transmissão( 𝐹 𝑇)
Resolução:
a) Velocidade angular da polia 1 ( 𝜔1)
𝜔1 =
𝑛𝜋
30
=
1400𝜋
30
= 146.6
𝑟𝑎𝑑
𝑠
𝑜𝑢 46,6𝜋 𝑟𝑎𝑑/𝑠
b) Frequência da polia 1 ( 𝑓1)
𝑓1 =
𝑛1
60
=
1400
60
= 23,33 𝐻𝑧
c) Torque da polia 1(𝑀 𝑇1
)
𝑀 𝑇1
=
𝑃
𝑤1
=
4000
146,6
= 27,28 𝑁𝑚
d) Velocidade angular da polia 2 ( 𝜔2)
𝜔2 =
𝑑1
𝑑2
. 𝑤1 =
100.146,6
280
= 52,35
𝑟𝑎𝑑
𝑠
e) Frequência da polia 2 ( 𝑓2)
𝑓2 =
𝑤2
2𝜋
=
52,35
6,283
= 8,33 𝐻𝑧
f) Rotação da polia 2 ( 𝑛2)
𝑛2 = 60. 𝑓2 = 60 . 8,33 = 499,8 𝑟𝑝𝑚
g) Torque da polia 2 (𝑀 𝑇2
)
𝑀 𝑇2
=
𝑃
𝑤2
=
4000𝑊
52,35𝑟𝑎𝑑
𝑠
= 76,4 𝑁𝑚
h) Relação de transmissão (i)
𝑖 =
𝑑2
𝑑1
=
280
100
= 2,8
i) Velocidade periférica da transmissão (𝑉𝑝)
𝑉𝑝 = 𝑤1. 𝑟1 𝑜𝑢 𝑤2. 𝑟2
𝑟1 =
100
2
= 50 =
50
1000
= 0,05𝑚
𝑉𝑝 = 146,6. 0,05 = 7,33
𝑚
𝑠
j) Força tangencial da transmissão( 𝐹 𝑇)
𝐹 𝑇 =
𝑀 𝑇1
𝑟1
=
27,28
0,05
= 545,6 𝑁
EXERCÍCIO 13.3 (Helena) -
Uma transmissão é acionada por um motor elétrico com potência de P =
8KW, com rotação de n = 1720 rpm, chavetado a polia motora (1) do sistema. Dados
os diâmetros: d1 = 210mm; d2 = 450mm. Determine para a transmissão:
a) Velocidade angular da polia (1) [ω1]
𝜔1 =
𝑛𝜋
30
𝜔1 =
1720𝜋
30
𝜔1 = 57,33𝜋 𝑟𝑎𝑑/𝑠
b) Frequência da polia (1) [f1]
𝑓1 =
𝑛1
60
𝑓1 =
1720
60
𝑓1 = 28,66 𝐻𝑧
c) Torque da polia (1) [MT1]
𝑀 𝑇1
=
𝑃
𝜔1
𝑀 𝑇1
=
8000
57,33𝜋
𝑀 𝑇1
= 44,42𝑁𝑚
d) Velocidade angular da polia (2) [ω2]
𝜔2 =
𝑑1. 𝜔1
𝑑2
𝜔2 =
210𝑥57,33𝜋
450
𝜔2 = 26,75𝜋 𝑟𝑎𝑑/𝑠
e) Frequência da polia (2) [f2]
𝑓2 =
𝜔2
2𝜋
𝑓2 =
26,75𝜋
2𝜋
𝑓2 = 13,377 𝐻𝑧
f) Torque da polia (2) [MT2]
𝑀 𝑇1
=
𝑃
𝜔2
𝑀 𝑇1
=
8000
26,75𝜋
𝑀 𝑇1
= 95,2𝑁𝑚
g) Rotação da polia (2) [n2]
𝑛2 = 60𝑓2
𝑛2 = 60𝑥13,377
𝑛2 = 802,62 𝑟𝑝𝑚
h) Relação de transmissão [i]
𝑖 =
𝑑2
𝑑1
𝑖 =
450
210
𝑖 = 2,14
i) Velocidade periférica da transmissão [VP]
A velocidade periférica da transmissão é a mesma da polia (1) com a da polia (2),
portanto podemos utilizar:
𝑉𝑃 = 𝜔1. 𝑟1 = 𝜔2. 𝑟2
𝑉𝑃 = 𝜔1. 𝑟1
𝑉𝑃 = 57,33𝜋𝑥0,105
𝑉𝑃 = 6,02𝜋 𝑚/𝑠
𝑉𝑃 = 18,9 𝑚/𝑠
j) Força tangencial [FT]
𝐹 𝑇 =
𝑀 𝑇1
𝑟1
=
𝑀 𝑇2
𝑟2
𝐹 𝑇 =
𝑀 𝑇1
𝑟1
𝐹 𝑇 =
95,2
0,105
𝐹 𝑇 = 906,67𝑁
EXERCÍCIO 13.4 (Júlio) –
Um conjunto de transmissão por correias possui na polia movida um
diâmetro d2 = 200mm, e sabendo que a polia motora de diâmetro d1 = 100mm está
acoplada a um motor com potência de 1/2cv que gira a 1750 rpm. Como segue na
imagem abaixo.
FIGURA 48
FONTE: http://www.blogdaengenharia.com/wp-content/uploads/2013/05/PoliaseCorreias.pdf
Determine:
a) Velocidade angular da polia (1) [ω1]
𝜔1 =
𝑛𝜋
30
𝜔1 =
1750𝜋
30
𝜔1 = 58,33𝜋 𝑟𝑎𝑑/𝑠
b) Frequência da polia (1) [f1]
𝑓1 =
𝑛1
60
𝑓1 =
1750
60
𝑓1 = 29,166 𝐻𝑧
c) Torque da polia (1) [MT1]
Antes devo transformar a pot6encia de CV para W.
𝑃𝐶𝑉 =
𝑃 𝑊
735,5
𝑃 𝑊 = 𝑃𝐶𝑉. 735,5
𝑃 𝑊 = 0.5𝑥735,5
𝑃 𝑊 = 367,75𝑊
𝑀 𝑇1
=
𝑃
𝜔1
𝑀 𝑇1
=
367,75
58,33𝜋
𝑀 𝑇1
= 2,01𝑁𝑚
d) Velocidade angular da polia (2) [ω2]
𝜔2 =
𝑑1. 𝜔1
𝑑2
𝜔2 =
100𝑥58,33𝜋
200
𝜔2 = 29,165𝜋 𝑟𝑎𝑑/𝑠
e) Frequência da polia (2) [f2]
𝑓2 =
𝜔2
2𝜋
𝑓2 =
29,165𝜋
2𝜋
𝑓2 = 14,58 𝐻𝑧
f) Torque da polia (2) [MT2]
𝑀 𝑇1
=
𝑃
𝜔2
𝑀 𝑇1
=
367,75
29,165𝜋
𝑀 𝑇1
= 4,01𝑁𝑚
g) Rotação da polia (2) [n2]
𝑛2 = 60𝑓2
𝑛2 = 60𝑥14,58
𝑛2 = 874,8 𝑟𝑝𝑚
h) Relação de transmissão [i]
𝑖 =
𝑑2
𝑑1
𝑖 =
200
100
𝑖 = 2
i) Velocidade periférica da transmissão [VP]
A velocidade periférica da transmissão é a mesma da polia (1) com a da
polia (2), e sempre utilizar o raio em metros, portanto podemos utilizar:
𝑉𝑃 = 𝜔1. 𝑟1 = 𝜔2. 𝑟2
𝑉𝑃 = 𝜔1. 𝑟1
𝑉𝑃 = 58,33𝜋𝑥0,1
𝑉𝑃 = 5,833𝜋 𝑚/𝑠
𝑉𝑃 = 18,33 𝑚/𝑠
j) Força tangencial [FT]
𝐹 𝑇 =
𝑀 𝑇1
𝑟1
=
𝑀 𝑇2
𝑟2
𝐹 𝑇 =
𝑀 𝑇1
𝑟1
𝐹 𝑇 =
2,01
0,1
𝐹 𝑇 = 20,1𝑁
EXERCÍCIO 14 –
A esquematização da figura representa um motor a combustão para
automóvel, que aciona simultaneamente as polias da bomba D`água e do alternador.
As curvas de desempenho do motor apresentam para o torque máximo a potência P
= 35,3 KW (P = 48cv), atuando com rotação n = 2000 rpm. Determine para a
condição de torque máximo.
FIGURA 49
FONTE: MELCONIAN. SARKIS: Elementos de Máquinas – 9 Ed. Revisada. Editora Érica. Ano 2009.
Polia do motor (1)
a) Velocidade angular da polia (1) [ω1]
𝜔1 =
𝑛𝜋
30
𝜔1 =
2000𝜋
30
𝜔1 = 66,66𝜋 𝑟𝑎𝑑/𝑠
b) Frequência da polia (1) [f1]
𝑓1 =
𝑛1
60
𝑓1 =
2000
60
𝑓1 = 33,33 𝐻𝑧
c) Torque da polia (1) [MT1]
𝑀 𝑇1
=
𝑃
𝜔1
𝑀 𝑇1
=
35300
66,66𝜋
𝑀 𝑇1
= 168,56𝑁𝑚
Polia bomba D`água (2)
d) Velocidade angular da polia (2) [ω2]
𝜔2 =
𝑑1. 𝜔1
𝑑2
𝜔2 =
120𝑥66,66𝜋
90
𝜔2 = 88,88𝜋 𝑟𝑎𝑑/𝑠
e) Frequência da polia (2) [f2]
𝑓2 =
𝜔2
2𝜋
𝑓2 =
88,88𝜋
2𝜋
𝑓2 = 44,44 𝐻𝑧
f) Rotação da polia (2) [n2]
𝑛2 = 60𝑓2
𝑛2 = 60𝑥44,44
𝑛2 = 2666,4 𝑟𝑝𝑚
g) Torque da polia (2) [MT2]
𝑀 𝑇2
=
𝑃
𝜔2
𝑀 𝑇2
=
35300
88,88𝜋
𝑀 𝑇2
= 126,42𝑁𝑚
Polia Alternador (3)
h) Velocidade angular da polia (3) [ω3]
𝜔3 =
𝑑1. 𝜔1
𝑑3
𝜔3 =
120𝑥66,66𝜋
80
𝜔3 = 99,99𝜋 𝑟𝑎𝑑/𝑠
i) Frequência da polia (3) [f3]
𝑓3 =
𝜔3
2𝜋
𝑓3 =
99,99𝜋
2𝜋
𝑓3 = 49.995 𝐻𝑧
j) Rotação da polia (3) [n3]
𝑛3 = 60𝑓3
𝑛3 = 60𝑥49,995
𝑛3 = 2999,7 𝑟𝑝𝑚
k) Torque da polia (3) [MT3]
𝑀 𝑇3
=
𝑃
𝜔3
𝑀 𝑇3
=
35300
99,99𝜋
𝑀 𝑇3
= 112,37𝑁𝑚
Características da transmissão.
l) Relação de transmissão [i] (Motor/Bomba D`água)
𝑖 =
𝑑1
𝑑2
𝑖 =
120
90
𝑖 = 1,33
m) Relação de transmissão [i] (Motor/Alternador)
𝑖 =
𝑑1
𝑑2
𝑖 =
120
80
𝑖 = 1,5
n) Força tangencial [FT]
𝐹 𝑇 =
𝑀 𝑇1
𝑟1
=
𝑀 𝑇2
𝑟2
𝐹 𝑇 =
𝑀 𝑇1
𝑟1
𝐹 𝑇 =
168,56
0,06
𝐹 𝑇 = 2809,33𝑁
o) Velocidade periférica da transmissão [VP]
𝑉𝑃 = 𝜔1. 𝑟1 = 𝜔2. 𝑟2
𝑉𝑃 = 𝜔1. 𝑟1
𝑉𝑃 = 33,33𝜋𝑥0,12
𝑉𝑃 = 3,9996𝜋 𝑚/𝑠
𝑉𝑃 = 12,56 𝑚/𝑠
EXERCÍCIO 14.1 (Eric) –
Conforme os dados do exercício 5.1 (polia motora = 160mm, polia bomba
d’água = 120 mm, polia alternador = 110 mm), a uma rotação de 3000 rpm do motor
e o motor com potência de 110 cv, determine:
‘
FIGURA 50
FONTE: Autor Eric Luiz Caetano.
a) Velocidade angular da polia motora. (𝜔1)
𝜔1 =
𝑛 . 𝜋
30
𝜔1 =
3000 . 𝜋
30
𝜔1 = 100𝜋 𝑟𝑎𝑑 𝑠⁄
b) Frequência da polia motora (𝑓1)
𝑓1 =
𝜔1
2𝜋
𝑓1 =
100𝜋
2𝜋
= 50 𝐻𝑧
c) Torque na polia motora (𝑀𝑡)
𝑀𝑡 =
𝑃
𝜔1
𝑀𝑡 =
(110 . 735,5)
100𝜋
𝑀𝑡 = 257,5 𝑁𝑚
d) Velocidade angular da polia da bomba d’água (𝜔2)
𝜔2 =
𝜔1 𝑑1
𝑑2
𝜔2 =
100𝜋 . 160
120
𝜔2 = 133,3𝜋 𝑟𝑎𝑑 𝑠⁄
e) Frequência da polia da bomba d’água (𝑓2)
𝑓2 =
𝜔2
2𝜋
𝑓2 =
133,3𝜋
2𝜋
𝑓2 = 66,5 𝐻𝑧
f) Torque na polia da bomba d’água (𝑀𝑡)
𝑀𝑡 =
𝑃
𝜔2
𝑀𝑡 =
(110 . 735,5)
133,3𝜋
𝑀𝑡 = 193,63 𝑁𝑚
g) Rotação da polia da bomba d’água (𝑛2)
𝑛2 = 60 . 𝑓2
𝑛2 = 60 . 66,5
𝑛2 = 3390 𝑟𝑝𝑚
h) Velocidade angular da polia do alternador (𝜔3)
𝜔3 =
𝜔1 𝑑1
𝑑3
𝜔3 =
100𝜋 . 160
110
𝜔3 = 145,45𝜋 𝑟𝑎𝑑/𝑠
i) Frequência da polia do alternador (𝑓3)
𝑓3 =
𝜔3
2𝜋
𝑓3 =
145,45𝜋
2𝜋
𝑓3 = 72,72 𝐻𝑧
j) Torque na polia do alternador (𝑀𝑡)
𝑀𝑡 =
𝑃
𝜔3
𝑀𝑡 =
(110 . 735,5)
145,45𝜋
𝑀𝑡 = 177,06 𝑁𝑚
k) Rotação da polia do alternador ()
𝑛3 = 60 . 𝑓3
𝑛3 = 60 . 72,72
𝑛3 = 4363,2 𝑟𝑝𝑚
EXERCICIO 14.2 (Felipe) –
A figura abaixo mostra a esquematização de um motor a Diesel de um
automóvel de porte médio, que aciona simultaneamente as polias da bomba D`água
e do alternador. As curvas de desempenho do motor apresentam para o torque
máximo a potência P = 50 KW (P = 68cv), atuando com rotação n = 3000 rpm.
Determine para a condição de torque máximo.
FIGURA 51
FONTE: MELCONIAN. SARKIS: Elementos de Máquinas – 9 Ed. Revisada. Editora Érica. Ano 2009.
COM ALTERAÇÃO DO AUTOR.
Polia do motor (1)
a) Velocidade angular da polia (1) (ω1)
𝜔1 =
𝑛𝜋
30
𝜔1 =
3000𝜋
30
𝜔1 = 100𝜋 𝑟𝑎𝑑/𝑠
b) Frequência da polia (1) (f1)
𝑓1 =
𝑛1
60
𝑓1 =
3000
60
𝑓1 = 50 𝐻𝑧
c) Torque da polia (1) (MT1)
𝑀 𝑇1
=
𝑃
𝜔1
𝑀 𝑇1
=
50000
100𝜋
𝑀 𝑇1
= 159,15𝑁𝑚
Polia bomba D`água (2)
d) Velocidade angular da polia (2) (ω2)
𝜔2 =
𝑑1. 𝜔1
𝑑2
𝜔2 =
140𝑥100𝜋
110
𝜔2 = 127,27𝜋 𝑟𝑎𝑑/𝑠
e) Frequência da polia (2) (f2)
𝑓2 =
𝜔2
2𝜋
𝑓2 =
127,27𝜋
2𝜋
𝑓2 = 63,63 𝐻𝑧
f) Rotação da polia (2) (n2)
𝑛2 = 60𝑓2
𝑛2 = 60𝑥63,63
𝑛2 = 3818𝑟𝑝𝑚
g) Torque da polia (2) (MT2)
𝑀 𝑇2
=
𝑃
𝜔2
𝑀 𝑇2
=
50000
127,27𝜋
𝑀 𝑇2
= 125,06𝑁𝑚
Polia Alternador (3)
h) Velocidade angular da polia (3) (ω3)
𝜔3 =
𝑑1. 𝜔1
𝑑3
𝜔3 =
140𝑥100𝜋
100
𝜔3 = 140𝜋 𝑟𝑎𝑑/𝑠
i) Frequência da polia (3) (f3)
𝑓3 =
𝜔3
2𝜋
𝑓3 =
140𝜋
2𝜋
𝑓3 = 70 𝐻𝑧
j) Rotação da polia (3) (n3)
𝑛3 = 60𝑓3
𝑛3 = 60𝑥70
𝑛3 = 4200 𝑟𝑝𝑚
k) Torque da polia (3) (MT3)
𝑀 𝑇3
=
𝑃
𝜔3
𝑀 𝑇3
=
50000
140𝜋
𝑀 𝑇3
= 113,68𝑁𝑚
Características da transmissão.
l) Relação de transmissão (i) (Motor/Bomba D`água)
𝑖 =
𝑑1
𝑑2
𝑖 =
140
110
𝑖 = 1,27
m) Relação de transmissão (i) (Motor/Alternador)
𝑖 =
𝑑1
𝑑3
𝑖 =
140
100
𝑖 = 1,4
n) Força tangencial (FT)
𝐹 𝑇 =
𝑀 𝑇1
𝑟1
𝑜𝑢
𝑀 𝑇2
𝑟2
𝐹 𝑇 =
𝑀 𝑇1
𝑟1
𝐹 𝑇 =
159,15
0,07
𝐹 𝑇 = 2273,6𝑁
o) Velocidade periférica da transmissão (VP)
𝑉𝑃 = 𝜔1. 𝑟1 𝑜𝑢 𝜔2. 𝑟2
𝑉𝑃 = 𝜔1. 𝑟1
𝑉𝑃 = 100𝜋𝑥0,07
𝑉𝑃 = 7𝜋 𝑚/𝑠
𝑉𝑃 = 21,99 𝑚/𝑠
EXERCÍCIO 14.3 (Helena) -
Um motor à combustão aciona simultaneamente as polias da bomba D’água
e do alternador. As curvas de desempenho do motor apresentam para o torque
máximo a potência P = 25,5 KW, atuando com rotação n = 1720 rpm. Dados:
d1 (motor) = 100mm
d2 (bomba) = 80mm
d3 (alternador) = 75mm
Determine para a condição de torque máximo.
Polia do motor (1)
a) Velocidade angular da polia (1) [ω1]
𝜔1 =
𝑛𝜋
30
𝜔1 =
1720𝜋
30
𝜔1 = 57,33𝜋 𝑟𝑎𝑑/𝑠
b) Frequência da polia (1) [f1]
𝑓1 =
𝑛1
60
𝑓1 =
1720
60
𝑓1 = 28,66 𝐻𝑧
c) Torque da polia (1) [MT1]
𝑀 𝑇1
=
𝑃
𝜔1
𝑀 𝑇1
=
25500
57,33𝜋
𝑀 𝑇1
= 141,58𝑁𝑚
Polia bomba D`água (2)
d) Velocidade angular da polia (2) [ω2]
𝜔2 =
𝑑1. 𝜔1
𝑑2
𝜔2 =
100𝑥57,33𝜋
80
𝜔2 = 71,66𝜋 𝑟𝑎𝑑/𝑠
e) Frequência da polia (2) [f2]
𝑓2 =
𝜔2
2𝜋
𝑓2 =
71,66𝜋
2𝜋
𝑓2 = 35,83 𝐻𝑧
f) Rotação da polia (2) [n2]
𝑛2 = 60𝑓2
𝑛2 = 60𝑥35,83
𝑛2 = 2149,875 𝑟𝑝𝑚
g) Torque da polia (2) [MT2]
𝑀 𝑇2
=
𝑃
𝜔2
𝑀 𝑇2
=
25500
71,66𝜋
𝑀 𝑇2
= 113,27𝑁𝑚
Polia Alternador (3)
h) Velocidade angular da polia (3) [ω3]
𝜔3 =
𝑑1. 𝜔1
𝑑3
𝜔3 =
100𝑥57,33𝜋
75
𝜔3 = 76,44𝜋 𝑟𝑎𝑑/𝑠
i) Frequência da polia (3) [f3]
𝑓3 =
𝜔3
2𝜋
𝑓3 =
76,44𝜋
2𝜋
𝑓3 = 38,22 𝐻𝑧
j) Rotação da polia (3) [n3]
𝑛3 = 60𝑓3
𝑛3 = 60𝑥38,22
𝑛3 = 2293,2 𝑟𝑝𝑚
k) Torque da polia (3) [MT3]
𝑀 𝑇3
=
𝑃
𝜔3
𝑀 𝑇3
=
25500
76,44𝜋
𝑀 𝑇3
= 106,19𝑁𝑚
Características da transmissão.
l) Relação de transmissão [i] (Motor/Bomba D`água)
𝑖 =
𝑑1
𝑑2
𝑖 =
100
80
𝑖 = 1,25
m) Relação de transmissão [i] (Motor/Alternador)
𝑖 =
𝑑1
𝑑3
𝑖 =
100
75
𝑖 = 1,33
n) Força tangencial [FT]
𝐹 𝑇 =
𝑀 𝑇1
𝑟1
=
𝑀 𝑇2
𝑟2
𝐹 𝑇 =
𝑀 𝑇1
𝑟1
𝐹 𝑇 =
141,58
0,05
𝐹 𝑇 = 2831,6𝑁
o) Velocidade periférica da transmissão [VP]
𝑉𝑃 = 𝜔1. 𝑟1 = 𝜔2. 𝑟2
𝑉𝑃 = 𝜔1. 𝑟1
𝑉𝑃 = 57,33𝜋𝑥0,05
𝑉𝑃 = 2,8665𝜋 𝑚/𝑠
𝑉𝑃 = 9 𝑚/𝑠
EXERCÍCIO 14.4 (Júlio) –
A esquematização da figura abaixo representa um conjunto de engrenagens,
acionadas por um motor que tem como suas curvas de desempenho máximo a
potência de P = 47KW para um torque máximo e com uma rotação de 2830 rpm.
Este motor esta acoplada a engrenagem (C) e aciona simultaneamente as
engrenagens (B) e (A).
As características da engrenagem são:
Pinhão (C): ZC = 8 dentes, M = 2mm (Módulo).
Engrenagem (B): ZB = 11 dentes, M = 2mm (Módulo).
Engrenagem (A): ZA = 15 dentes, M = 2mm (Módulo).
FIGURA 52
FONTE:http://2.bp.blogspot.com/_4zd06fOobnY/TUcQnhg0FfI/AAAAAAAAAmM/fQHW2wXYFD0/s16
00/denovo.png
Determine para a condição de torque máximo.
Pinhão motor (C)
a) Velocidade angular do pinhão (C) [ωC]
𝜔 𝐶 =
𝑛𝜋
30
𝜔 𝐶 =
2830𝜋
30
𝜔 𝐶 = 94,33𝜋 𝑟𝑎𝑑/𝑠
b) Frequência do pinhão (C) [fC]
𝑓𝐶 =
𝑛 𝐶
60
𝑓𝐶 =
2830
60
𝑓𝐶 = 47,17 𝐻𝑧
c) Torque do pinhão (C) [MTC]
𝑀 𝑇 𝐶
=
𝑃
𝜔 𝐶
𝑀 𝑇 𝐶
=
47000
94,33𝜋
𝑀 𝑇 𝐶
= 158,6𝑁𝑚
Engrenagem (B)
d) Velocidade angular da engrenagem (B) [ωB]
Para encontrar velocidade angular, antes devo encontrar os diâmetros das
engrenagens C e B.
𝑑 𝐶 = 𝑀. 𝑍 𝐶
𝑑 𝐶 = 2𝑥8
𝑑 𝐶 = 16𝑚𝑚
𝑑 𝐵 = 𝑀. 𝑍 𝐵
𝑑 𝐵 = 2𝑥11
𝑑 𝐵 = 22𝑚𝑚
𝜔 𝐵 =
𝑑 𝐶. 𝜔 𝐶
𝑑 𝐵
𝜔 𝐵 =
16𝑥94,33𝜋
22
𝜔 𝐵 = 68,6𝜋 𝑟𝑎𝑑/𝑠
e) Frequência da engrenagem (B) [fB]
𝑓𝐵 =
𝜔 𝐵
2𝜋
𝑓𝐵 =
68,6𝜋
2𝜋
𝑓𝐵 = 34,3 𝐻𝑧
f) Rotação da engrenagem (B) [nB]
𝑛 𝐵 = 60𝑓𝐵
𝑛 𝐵 = 60𝑥34,3
𝑛 𝐵 = 2058,1 𝑟𝑝𝑚
g) Torque da engrenagem (B) [MTB]
𝑀 𝑇 𝐵
=
𝑃
𝜔 𝐵
𝑀 𝑇 𝐵
=
47000
68,6𝜋
𝑀 𝑇 𝐵
= 218,08𝑁𝑚
Engrenagem (A)
h) Velocidade angular da engrenagem (A) [ωA]
𝑑 𝐴 = 𝑀. 𝑍 𝐴
𝑑 𝐴 = 2𝑥15
𝑑 𝐴 = 30𝑚𝑚
𝜔 𝐴 =
𝑑 𝐶. 𝜔 𝐶
𝑑 𝐴
𝜔 𝐴 =
16𝑥94,33𝜋
30
𝜔 𝐴 = 50,31𝜋 𝑟𝑎𝑑/𝑠
i) Frequência da engrenagem (A) [fA]
𝑓𝐴 =
𝜔 𝐴
2𝜋
𝑓𝐴 =
50,31𝜋
2𝜋
𝑓𝐴 = 25,16 𝐻𝑧
j) Rotação da engrenagem (A) [nA]
𝑛 𝐴 = 60𝑓𝐴
𝑛 𝐴 = 60𝑥25,16
𝑛 𝐴 = 1509,3 𝑟𝑝𝑚
k) Torque da engrenagem (A) [MTA]
𝑀 𝑇 𝐴
=
𝑃
𝜔 𝐴
𝑀 𝑇 𝐴
=
47000
50,31𝜋
𝑀 𝑇 𝐴
= 297,37𝑁𝑚
Características da transmissão.
l) Relação de transmissão [i] [Pinhão (C)/Engrenagem (B)]
𝑖 =
𝑍 𝐵
𝑍 𝐶
𝑖 =
11
8
𝑖 = 1,375
m) Relação de transmissão [i] [Pinhão (C)/Engrenagem (A)]
𝑖 =
𝑍 𝐴
𝑍 𝐶
𝑖 =
15
8
𝑖 = 1,875
n) Força tangencial [FT]
𝐹 𝑇 =
𝑀 𝑇1
𝑟1
=
𝑀 𝑇2
𝑟2
𝐹 𝑇 =
𝑀 𝑇 𝐶
𝑟𝐶
𝐹 𝑇 =
158,6
0,016
𝐹 𝑇 = 9912,5𝑁
o) Velocidade periférica da transmissão [VP]
𝑉𝑃 = 𝜔 𝐶. 𝑟𝐶 = 𝜔 𝐵. 𝑟𝐵
𝑉𝑃 = 𝜔 𝐶. 𝑟𝐶
𝑉𝑃 = 94,33𝜋𝑥0,016
𝑉𝑃 = 1,5093𝜋 𝑚/𝑠
𝑉𝑃 = 4,74 𝑚/𝑠
RELAÇÃO DE TRANSMISSÃO POR ENGRENAGENS
EXERCÍCIO 15 –
A transmissão por engrenagem é acionada por meio do pinhão (1) acoplado
a um motor elétrico de IV polos com potência P = 15KW, (P = 20cv) e rotação n
1720 rpm. As características da engrenagem são: Pinhão (1): Z1 = 24 dentes, M =
4mm (Módulo). Coroa (2): Z2 = 73 dentes, M = 4mm (Módulo)
FIGURA 53
FONTE: MELCONIAN. SARKIS: Elementos de Máquinas – 9 Ed. Revisada. Editora Érica. Ano 2009.
Determinar para a transmissão:
Pinhão (1).
a) Velocidade angular do pinhão (1) [ω1]
𝜔1 =
𝑛𝜋
30
𝜔1 =
1720𝜋
30
𝜔1 = 57,33𝜋 𝑟𝑎𝑑/𝑠
b) Frequência da polia (1) [f1]
𝑓1 =
𝑛1
60
𝑓1 =
1720
60
𝑓1 = 28,67 𝐻𝑧
c) Torque da polia (1) [MT1]
𝑀 𝑇1
=
𝑃
𝜔1
𝑀 𝑇1
=
15000
57,33𝜋
𝑀 𝑇1
= 83,28𝑁𝑚
Coroa (2).
d) Velocidade angular da polia (2) [ω2]
𝜔2 =
𝑍1. 𝜔1
𝑍2
𝜔2 =
24𝑥57,33𝜋
73
𝜔2 = 18,85𝜋 𝑟𝑎𝑑/𝑠
e) Frequência da polia (2) [f2]
𝑓2 =
𝜔2
2𝜋
𝑓2 =
18,85𝜋
2𝜋
𝑓2 = 9,42 𝐻𝑧
f) Rotação da polia (2) [n2]
𝑛2 = 60𝑓2
𝑛2 = 60𝑥9,42
𝑛2 = 565,44 𝑟𝑝𝑚
g) Torque da polia (2) [MT2]
𝑀 𝑇2
=
𝑃
𝜔2
𝑀 𝑇2
=
15000
18,85𝜋
𝑀 𝑇2
= 253,3𝑁𝑚
Características da transmissão.
h) Velocidade periférica da transmissão [VP]
𝑉𝑃 = 𝜔1. 𝑟1
Ou
𝑉𝑃 = 𝜔2. 𝑟2
𝑟1 =
𝑑1
2
𝑑1 = 𝑀. 𝑍1
𝑑1 = 𝑀. 𝑍1
𝑑1 = 4𝑥24
𝑑1 = 96𝑚𝑚
𝑟1 =
𝑑1
2
𝑟1 =
96
2
𝑟1 = 48𝑚𝑚
𝑟1 = 0.048𝑚
𝑉𝑃 = 𝜔1. 𝑟1
𝑉𝑃 = 57,33𝜋𝑥0,048
𝑉𝑃 = 2,75𝜋 𝑚/𝑠
𝑉𝑃 = 8,645 𝑚/𝑠
i) Força tangencial [FT]
𝐹 𝑇 =
𝑀 𝑇1
𝑟1
𝐹 𝑇 =
83,28
0,048
𝐹 𝑇 = 1735𝑁
j) Relação transmissão [i]
𝑖 =
𝑍2
𝑍1
𝑖 =
73
24
𝑖 = 3,04
EXERCÍCIO 15.1 (Eric) –
Uma transmissão por engrenagens ampliadora de velocidade é composta
por um motor elétrico com potência de 12 cv (8826 W), que gira uma coroa com 52
dentes com uma rotação de 1800 rpm. Essa, por sua vez, movimenta um pinhão
com 27 dentes. Ambos os módulos das engrenagens são de 3 mm. Para essa
transmissão, determine:
a) Velocidade angular da coroa (1)
1 =
𝑛 . 𝜋
30
1 =
1800 . 𝜋
30
1 = 60𝜋 𝑟𝑎𝑑 𝑠⁄
b) Frequência da coroa (𝑓1)
𝑓1 =
𝜔1
2𝜋
𝑓1 =
60𝜋
2𝜋
𝑓1 = 30 𝐻𝑧
c) Torque na coroa (𝑀𝑡)
𝑀𝑡 =
𝑃
1
𝑀𝑡 =
8826
60𝜋
𝑀𝑡 = 46,82 𝑁𝑚
d) Velocidade angular do pinhão (2) (Z= número de dentes de cada
engrenagem)
2 =
1 . 𝑍1
𝑍2
2 =
60𝜋 . 52
27
2 = 115,5 𝜋 𝑟𝑎𝑑 𝑠⁄
e) Frequência do pinhão (𝑓2)
𝑓2 =
𝜔2
2𝜋
𝑓2 =
115,5 𝜋
2𝜋
𝑓2 = 57,75 𝐻𝑧
f) Torque no pinhão (𝑀𝑡2)
𝑀𝑡2 = 𝑀𝑡1 .
𝑍2
𝑍1
𝑀𝑡2 = 46,82 .
27
52
𝑀𝑡2 = 24,31 𝑁𝑚
g) Rotação do pinhão (𝑛2)
𝑛2 = 60 . 𝑓2
𝑛2 = 60 . 57,75
𝑛2 = 3465 𝑟𝑝𝑚
h) Relação de transmissão (𝑖)
𝑖 =
𝑍2
𝑍1
𝑖 =
27
52
𝑖 = 0,51
i) Força tangencial (𝑑01)
𝑑01 = 𝑚 . 𝑍1
𝑑01 = 3 . 52
𝑑01 = 156 𝑚𝑚
𝑑01 = 0,156 𝑚
𝐹 𝑇 =
2 . 𝑀 𝑇1
𝑑01
𝐹 𝑇 =
2 . 46,82
0,156
𝐹 𝑇 = 600,25 𝑁
j) Velocidade periférica (𝑟01)
𝑟01 =
𝑑01
2
𝑟01 =
0,156
2
𝑟01 = 0,078 𝑚
𝑉𝑝 = 𝜔1. 𝑟01
𝑉𝑝 = 60𝜋 . 0,078
𝑉𝑝 = 14,70 𝑚/𝑠
EXERCICIO 15.2 (Felipe) -
Em uma determinada transmissão por engrenagem, é feito o acionamento por meio
do pinhão (1) acoplado a um motor com potência P = 18KW, (P = 25cv) e rotação n
2100 rpm. As características da engrenagem são: Pinhão (1): Z1 = 28 dentes, M =
6mm (Módulo). Coroa (2): Z2 = 79 dentes, M = 6mm (Módulo)
FIGURA 54
FONTE: http://www.portalcad.com/blog/index.php/10-dicas-e-truques.
Determinar para a transmissão:
Pinhão (1).
a) Velocidade angular do pinhão (1) (ω1)
𝜔1 =
𝑛𝜋
30
𝜔1 =
2100𝜋
30
𝜔1 = 70𝜋 𝑟𝑎𝑑/𝑠
b) Frequência da polia (1) (𝑓1)
𝑓1 =
𝑛1
60
𝑓1 =
2100
60
𝑓1 = 35 𝐻𝑧
c) Torque da polia (1) (MT1)
𝑀 𝑇1
=
𝑃
𝜔1
𝑀 𝑇1
=
18000
57,33𝜋
𝑀 𝑇1
= 81,85𝑁𝑚
Coroa (2).
d) Velocidade angular da polia (2) (ω2)
𝜔2 =
𝑍1. 𝜔1
𝑍2
𝜔2 =
28𝑥70𝜋
79
𝜔2 = 24,81𝜋 𝑟𝑎𝑑/𝑠
e) Frequência da polia (2) (𝑓2)
𝑓2 =
𝜔2
2𝜋
𝑓2 =
24,81𝜋
2𝜋
𝑓2 = 12,4 𝐻𝑧
f) Rotação da polia (2) (n2)
𝑛2 = 60𝑓2
𝑛2 = 60𝑥12,4
𝑛2 = 744,3 𝑟𝑝𝑚
g) Torque da polia (2) (MT2)
𝑀 𝑇2
=
𝑃
𝜔2
𝑀 𝑇2
=
18000
24,81𝜋
𝑀 𝑇2
= 230,9𝑁𝑚
Características da transmissão.
h) Velocidade periférica da transmissão (VP)
𝑉𝑃 = 𝜔1. 𝑟1
Ou
𝑉𝑃 = 𝜔2. 𝑟2
𝑟1 =
𝑑1
2
𝑑1 = 𝑀. 𝑍1
𝑑1 = 𝑀. 𝑍1
𝑑1 = 6𝑥28
𝑑1 = 168𝑚𝑚
𝑟1 =
𝑑1
2
𝑟1 =
168
2
𝑟1 = 84𝑚𝑚
𝑟1 = 0.084𝑚
𝑉𝑃 = 𝜔1. 𝑟1
𝑉𝑃 = 70𝜋𝑥0,084𝑚
𝑉𝑃 = 5,88𝜋 𝑚/𝑠
𝑉𝑃 = 18,47 𝑚/𝑠
i) Força tangencial (FT)
𝐹 𝑇 =
𝑀 𝑇1
𝑟1
𝐹 𝑇 =
81,85
0,084
𝐹 𝑇 = 974,4𝑁
j) Relação transmissão (i)
𝑖 =
𝑍2
𝑍1
𝑖 =
79
28
𝑖 = 2,82
EXERCÍCIO 15.3 (Helena) –
A transmissão por engrenagem é acionada por meio do pinhão (1) acoplado
a um motor elétrico de IV polos com potência P = 20KW e rotação n=2000 rpm. As
características da engrenagem são: Pinhão (1): Z1 = 28 dentes, M = 6mm (Módulo).
Coroa (2): Z2 = 79 dentes, M = 6mm (Módulo)
Determinar para a transmissão:
Pinhão (1).
a) Velocidade angular do pinhão (1) [ω1]
𝜔1 =
𝑛𝜋
30
𝜔1 =
2000𝜋
30
𝜔1 = 66,67𝜋 𝑟𝑎𝑑/𝑠
b) Frequência da polia (1) [f1]
𝑓1 =
𝑛1
60
𝑓1 =
2000
60
𝑓1 = 33,33 𝐻𝑧
c) Torque da polia (1) [MT1]
𝑀 𝑇1
=
𝑃
𝜔1
𝑀 𝑇1
=
20000
66,67𝜋
𝑀 𝑇1
= 95,49𝑁𝑚
Coroa (2).
d) Velocidade angular da polia (2) [ω2]
𝜔2 =
𝑍1. 𝜔1
𝑍2
𝜔2 =
28𝑥66,67𝜋
79
𝜔2 = 23,63𝜋 𝑟𝑎𝑑/𝑠
e) Frequência da polia (2) [f2]
𝑓2 =
𝜔2
2𝜋
𝑓2 =
23,63𝜋
2𝜋
𝑓2 = 11,81𝐻𝑧
f) Rotação da polia (2) [n2]
𝑛2 = 60𝑓2
𝑛2 = 60𝑥11,81
𝑛2 = 708,9 𝑟𝑝𝑚
g) Torque da polia (2) [MT2]
𝑀 𝑇2
=
𝑃
𝜔2
𝑀 𝑇2
=
20000
23,63𝜋
𝑀 𝑇2
= 269,4𝑁𝑚
Características da transmissão.
h) Velocidade periférica da transmissão [VP]
𝑉𝑃 = 𝜔1. 𝑟1
Ou
𝑉𝑃 = 𝜔2. 𝑟2
𝑟1 =
𝑑1
2
𝑑1 = 𝑀. 𝑍1
𝑑1 = 𝑀. 𝑍1
𝑑1 = 6𝑥28
𝑑1 = 168𝑚𝑚
𝑟1 =
𝑑1
2
𝑟1 =
168
2
𝑟1 = 84𝑚𝑚
𝑟1 = 0.084𝑚
𝑉𝑃 = 𝜔1. 𝑟1
𝑉𝑃 = 66,67𝜋𝑥0,084
𝑉𝑃 = 5,60𝜋 𝑚/𝑠
𝑉𝑃 = 17,59 𝑚/𝑠
i) Força tangencial [FT]
𝐹 𝑇 =
𝑀 𝑇1
𝑟1
𝐹 𝑇 =
95,49
0,084
𝐹 𝑇 = 1136,785𝑁
j) Relação transmissão [i]
𝑖 =
𝑍2
𝑍1
𝑖 =
79
28
𝑖 = 2,82
EXERCÍCIO 15.4 (Júlio) –
Na imagem abaixo é representado um sistema de engrenagens onde a
engrenagem (1) pinhão é acionada por um motor de P = 2cv, por meio de um
acoplamento e a rotação é de n = 1000 rpm. As características das engrenagens
são: Pinhão (1): Z1 = 36 dentes, M = 2mm (Módulo). Coroa (2): Z2 = 91 dentes, M =
2mm (Módulo).
FIGURA 55
FONTE: http://www.etepiracicaba.org.br/cursos/apostilas/mecanica/1_ciclo/tec_projeto1.pdf
Determinar para a transmissão:
Pinhão (1).
a) Velocidade angular do pinhão (1) [ω1]
𝜔1 =
𝑛𝜋
30
𝜔1 =
1000𝜋
30
𝜔1 = 33,33𝜋 𝑟𝑎𝑑/𝑠
b) Frequência da polia (1) [f1]
𝑓1 =
𝑛1
60
𝑓1 =
1000
60
𝑓1 = 16,67 𝐻𝑧
c) Torque da polia (1) [MT1]
Antes devo transformar a pot6encia de CV para W.
𝑃𝐶𝑉 =
𝑃 𝑊
735,5
𝑃 𝑊 = 𝑃𝐶𝑉. 735,5
𝑃 𝑊 = 2𝑥735,5
𝑃 𝑊 = 1471𝑊
𝑀 𝑇1
=
𝑃
𝜔1
𝑀 𝑇1
=
1471
33,33𝜋
𝑀 𝑇1
= 14,05𝑁𝑚
Coroa (2).
d) Velocidade angular da polia (2) [ω2]
𝜔2 =
𝑍1. 𝜔1
𝑍2
𝜔2 =
36𝑥33,33𝜋
91
𝜔2 = 13,18𝜋 𝑟𝑎𝑑/𝑠
e) Frequência da polia (2) [f2]
𝑓2 =
𝜔2
2𝜋
𝑓2 =
13,18𝜋
2𝜋
𝑓2 = 6,59 𝐻𝑧
f) Rotação da polia (2) [n2]
𝑛2 = 60𝑓2
𝑛2 = 60𝑥6,59
𝑛2 = 395,4 𝑟𝑝𝑚
g) Torque da polia (2) [MT2]
𝑀 𝑇2
=
𝑃
𝜔2
𝑀 𝑇2
=
1471
13,18𝜋
𝑀 𝑇2
= 35,53𝑁𝑚
Características da transmissão.
h) Velocidade periférica da transmissão [VP]
𝑉𝑃 = 𝜔1. 𝑟1 = 𝜔2. 𝑟2
𝑟1 =
𝑑1
2
𝑑1 = 𝑀. 𝑍1
𝑑1 = 𝑀. 𝑍1
𝑑1 = 2𝑥36
𝑑1 = 72𝑚𝑚
𝑟1 =
𝑑1
2
𝑟1 =
72
2
𝑟1 = 36𝑚𝑚
𝑟1 = 0.036𝑚
𝑉𝑃 = 𝜔1. 𝑟1
𝑉𝑃 = 33,33𝜋𝑥0,036
𝑉𝑃 = 1,19988𝜋 𝑚/𝑠
𝑉𝑃 = 3,77 𝑚/𝑠
i) Força tangencial [FT]
𝐹 𝑇 =
𝑀 𝑇1
𝑟1
𝐹 𝑇 =
14,05
0,036
𝐹 𝑇 = 390,3𝑁
j) Rolação transmissão [i]
𝑖 =
𝑍2
𝑍1
𝑖 =
91
36
𝑖 = 2,5
REFERÊNCIAS
ALIBABA. Products. Disponível em: <http://portuguese.alibaba.com/product-gs/hot-
new-products-for-2015-kitchen-outer-rotor-450mm-ac-axial-fan-60210360658.html>.
Acesso em: 03 de abr. 2015.
BLOG DA ENGENHARIA. Polias e correias. Disponível em:
<http://www.blogdaengenharia.com/wp-
content/uploads/2013/05/PoliaseCorreias.pdf>. Acesso em: 03 de abr. 2015.
BLOGSPOT. Disponível em:
<http://2.bp.blogspot.com/_4zd06fOobnY/TUcQnhg0FfI/AAAAAAAAAmM/fQHW2wX
YFD0/s1600/denovo.png>. Acesso em: 03 de abr. 2015.
BLOGSPOT. Formação piloto. Disponível em:
<http://formacaopiloto.blogspot.com.br/2014_06_01_archive.html>. Acesso em: 03
de abr. 2015.
CASA DAS POLIAS. Disponível em:
<http://www.casadaspolias.com.br/produtos/especificacoes-tecnicas.html>. Acesso
em: 03 de abr. 2015.
CEPA.IF.USP. Energia. Hidráulica. Disponível em:
<http://www.cepa.if.usp.br/energia/energia1999/Grupo2B/Hidraulica/roda.htm>.
Acesso em: 03 de abr. 2015.
CLIP ART LOGO. Premiun. Disponível em:
<http://pt.clipartlogo.com/premium/detail/car-or-truck-tire-line-art_109442720.html>
Acesso em: 03 de abr. 2015.
EDUCAÇÃO MG GOV. Sistema CRV. Disponível em:
<http://crv.educacao.mg.gov.br/sistema_crv/index.aspx?ID_OBJETO=58360&tipo=o
b&cp=780031&cb=&n1=&n2=M%EF%BF%BDdulos%20Did%EF%BF%BDticos&n3=
Ensino%20M%EF%BF%BDdio&n4=F%EF%BF%BDsica&b=s>. Acesso em: 03 de
abr. 2015.
ETEPIRACICABA ORG. Cursos. Apostila mecânica. Disponível em:
<http://www.etepiracicaba.org.br/cursos/apostilas/mecanica/1_ciclo/tec_projeto1.pdf
>. Acesso em: 03 de abr. 2015.
FISICA VESTIBULAR. Disponível em:
<http://fisicaevestibular.com.br/exe_din_15.htm>. Acesso em: 03 de abr. 2015.
GP MOTORS BRASIL. Disponível em:
<http://www.gpmotorsbrasil.com.br/info1.html>. Acesso em: 03 de abr. 2015.
PAULUZZI. Alvenaria. Disponível em:
<http://www.pauluzzi.com.br/alvenaria.php?PHPSESSID=ccd0dd0c90aa9901b2a2e4
9d3182897c>. Acesso em: 03 de abr. 2015.
PORTAL CAD. Blog. 10 truques e dicas. Disponível em:
<http://www.portalcad.com/blog/index.php/10-dicas-e-truques>. Acesso em: 03 de
abr.
MELCONIAN. SARKIS: Elementos de Máquinas – 9 Ed. Revisada. Editora Érica.
Ano 2009.
SEGURANCA NA CONSTRUCAO. Disponível em: <http://seguranca-na-
construcao.dashofer.pt/?s=modulos&v=capitulo&c=7655>. Acesso em: 03 de abr.
2015.
SLIDESHARE. Centro Apoio. Exercício equilíbrio corpo rígido. Disponível em:
<http://pt.slideshare.net/CentroApoio/exercequilibrio-corpo-rigido>. Acesso em: 03 de
abr. 2015.
VDL UFC. Solar. Aula. Disponível em:
<http://www.vdl.ufc.br/solar/aula_link/lfis/semestre01/Fisica_I/Aula_03/02.html>.
Acesso em: 03 de abr. 2015.
VEGA MAQUINAS. Retifica para torno. Disponível em:
<http://www.vegamaquinas.com.br/retifica-para-torno-pr-1313-229827.htm> Acesso
em: 03 de abr. 2015.
WIKIPEDIA. Turbina aeronave. Disponível em:
<http://pt.wikipedia.org/wiki/Turbina_aeron%C3%A1utica>. Acesso em: 03 de abr.
2015.

Mais conteúdo relacionado

Mais procurados

Perguntas e respostas de manutenção
Perguntas e respostas de manutençãoPerguntas e respostas de manutenção
Perguntas e respostas de manutençãooantu
 
Rolamentos e mancais
Rolamentos e mancaisRolamentos e mancais
Rolamentos e mancaisDaniel Garcia
 
Resistência dos materiais r. c. hibbeler
Resistência dos materiais   r. c. hibbelerResistência dos materiais   r. c. hibbeler
Resistência dos materiais r. c. hibbelerMeireles01
 
Exercícios pneumática solução
Exercícios pneumática    soluçãoExercícios pneumática    solução
Exercícios pneumática soluçãoCynthia Janei
 
Apostila sistemas mecanicos
Apostila sistemas mecanicosApostila sistemas mecanicos
Apostila sistemas mecanicosLetícia Gomes
 
Exercicios resolvidos de resmat mecsol
Exercicios resolvidos de resmat mecsolExercicios resolvidos de resmat mecsol
Exercicios resolvidos de resmat mecsolDanieli Franco Mota
 
SENAI - Apostila de Elementos de Máquina.pdf
SENAI - Apostila de Elementos de Máquina.pdfSENAI - Apostila de Elementos de Máquina.pdf
SENAI - Apostila de Elementos de Máquina.pdfssuser14a3061
 
Resistência dos materiais - Exercícios Resolvidos
Resistência dos materiais - Exercícios ResolvidosResistência dos materiais - Exercícios Resolvidos
Resistência dos materiais - Exercícios ResolvidosMoreira1972
 
95950 parafusos e-porcas
95950 parafusos e-porcas95950 parafusos e-porcas
95950 parafusos e-porcasMaicon da Luz
 
97792047 n-0059-simbolos-graficos-para-desenhos-de-tubulacao (2)
97792047 n-0059-simbolos-graficos-para-desenhos-de-tubulacao (2)97792047 n-0059-simbolos-graficos-para-desenhos-de-tubulacao (2)
97792047 n-0059-simbolos-graficos-para-desenhos-de-tubulacao (2)Jupira Silva
 
Cálculos movimento circular
Cálculos movimento circularCálculos movimento circular
Cálculos movimento circularTableau Colégio
 
Apostilas senai - processo soldagem
Apostilas   senai - processo soldagemApostilas   senai - processo soldagem
Apostilas senai - processo soldagemPaulo Cezar santos
 
Exercicios de torção
Exercicios de torçãoExercicios de torção
Exercicios de torçãoRomualdo SF
 
Acionamentos Elétricos
Acionamentos ElétricosAcionamentos Elétricos
Acionamentos Elétricoselliando dias
 
Processo de Usinagem - Torneamento
Processo de Usinagem - TorneamentoProcesso de Usinagem - Torneamento
Processo de Usinagem - TorneamentoMiguel Gut Seara
 

Mais procurados (20)

Perguntas e respostas de manutenção
Perguntas e respostas de manutençãoPerguntas e respostas de manutenção
Perguntas e respostas de manutenção
 
Rolamentos e mancais
Rolamentos e mancaisRolamentos e mancais
Rolamentos e mancais
 
Resistência dos materiais r. c. hibbeler
Resistência dos materiais   r. c. hibbelerResistência dos materiais   r. c. hibbeler
Resistência dos materiais r. c. hibbeler
 
Exercícios pneumática solução
Exercícios pneumática    soluçãoExercícios pneumática    solução
Exercícios pneumática solução
 
Apostila sistemas mecanicos
Apostila sistemas mecanicosApostila sistemas mecanicos
Apostila sistemas mecanicos
 
Exercicios resolvidos de resmat mecsol
Exercicios resolvidos de resmat mecsolExercicios resolvidos de resmat mecsol
Exercicios resolvidos de resmat mecsol
 
SENAI - Apostila de Elementos de Máquina.pdf
SENAI - Apostila de Elementos de Máquina.pdfSENAI - Apostila de Elementos de Máquina.pdf
SENAI - Apostila de Elementos de Máquina.pdf
 
Notas de aulas_resistencia1
Notas de aulas_resistencia1Notas de aulas_resistencia1
Notas de aulas_resistencia1
 
Resistência dos materiais - Exercícios Resolvidos
Resistência dos materiais - Exercícios ResolvidosResistência dos materiais - Exercícios Resolvidos
Resistência dos materiais - Exercícios Resolvidos
 
95950 parafusos e-porcas
95950 parafusos e-porcas95950 parafusos e-porcas
95950 parafusos e-porcas
 
Torno Mecânico- Acessórios
Torno Mecânico- Acessórios Torno Mecânico- Acessórios
Torno Mecânico- Acessórios
 
97792047 n-0059-simbolos-graficos-para-desenhos-de-tubulacao (2)
97792047 n-0059-simbolos-graficos-para-desenhos-de-tubulacao (2)97792047 n-0059-simbolos-graficos-para-desenhos-de-tubulacao (2)
97792047 n-0059-simbolos-graficos-para-desenhos-de-tubulacao (2)
 
Exercício proposto furadeira de bancada
Exercício proposto furadeira de bancadaExercício proposto furadeira de bancada
Exercício proposto furadeira de bancada
 
Cálculos movimento circular
Cálculos movimento circularCálculos movimento circular
Cálculos movimento circular
 
Apostilas senai - processo soldagem
Apostilas   senai - processo soldagemApostilas   senai - processo soldagem
Apostilas senai - processo soldagem
 
Exercicios de torção
Exercicios de torçãoExercicios de torção
Exercicios de torção
 
Acionamentos Elétricos
Acionamentos ElétricosAcionamentos Elétricos
Acionamentos Elétricos
 
Rm exerc resolvidos
Rm exerc resolvidosRm exerc resolvidos
Rm exerc resolvidos
 
Processo de Usinagem - Torneamento
Processo de Usinagem - TorneamentoProcesso de Usinagem - Torneamento
Processo de Usinagem - Torneamento
 
Aula clo elementos de transmissão
Aula clo elementos de transmissãoAula clo elementos de transmissão
Aula clo elementos de transmissão
 

Destaque

Resistencia dos-materiais-para-entender-
Resistencia dos-materiais-para-entender-Resistencia dos-materiais-para-entender-
Resistencia dos-materiais-para-entender-Ma Dos Anjos Pacheco
 
Fisica exercicios gabarito 000
Fisica exercicios gabarito  000Fisica exercicios gabarito  000
Fisica exercicios gabarito 000comentada
 
RESISTENCIA DE MATERIALES: Métodos de energía
RESISTENCIA DE MATERIALES: Métodos de energía RESISTENCIA DE MATERIALES: Métodos de energía
RESISTENCIA DE MATERIALES: Métodos de energía Juan Miguel
 
Elementos de-maquinas - prof moro - ifsc
Elementos de-maquinas - prof moro - ifscElementos de-maquinas - prof moro - ifsc
Elementos de-maquinas - prof moro - ifscTiago Gomes
 
Taller S10 Costos y Presupuestos - Parte 1
Taller S10 Costos y Presupuestos - Parte 1Taller S10 Costos y Presupuestos - Parte 1
Taller S10 Costos y Presupuestos - Parte 1Alex Arribasplata
 
Elementos de máquinas sarkis melconian
Elementos de máquinas   sarkis melconianElementos de máquinas   sarkis melconian
Elementos de máquinas sarkis melconianAlex Fabiano Bueno
 
Apostila elementos de_maquinas
Apostila elementos de_maquinasApostila elementos de_maquinas
Apostila elementos de_maquinasneydom
 
Elementos de maquinas apostila Senai
Elementos de maquinas apostila SenaiElementos de maquinas apostila Senai
Elementos de maquinas apostila Senaisheylaladeiracosta
 
Formulas de Física
Formulas de Física Formulas de Física
Formulas de Física FabsVitti
 
Formulas de física
Formulas de físicaFormulas de física
Formulas de físicajeduca62
 
Formulario fisica
Formulario fisicaFormulario fisica
Formulario fisicajavier11074
 
Ejercicios resueltos blog definitivo
Ejercicios  resueltos  blog definitivoEjercicios  resueltos  blog definitivo
Ejercicios resueltos blog definitivoYanina Soledad Perez
 
Problemas resueltos fluidos
Problemas resueltos fluidosProblemas resueltos fluidos
Problemas resueltos fluidosedeive
 
Ejercicios resueltos dinamica de fluidos
Ejercicios resueltos dinamica de fluidosEjercicios resueltos dinamica de fluidos
Ejercicios resueltos dinamica de fluidosRoly Pariona Silva
 

Destaque (20)

Resistencia dos-materiais-para-entender-
Resistencia dos-materiais-para-entender-Resistencia dos-materiais-para-entender-
Resistencia dos-materiais-para-entender-
 
Fisica exercicios gabarito 000
Fisica exercicios gabarito  000Fisica exercicios gabarito  000
Fisica exercicios gabarito 000
 
RESISTENCIA DE MATERIALES: Métodos de energía
RESISTENCIA DE MATERIALES: Métodos de energía RESISTENCIA DE MATERIALES: Métodos de energía
RESISTENCIA DE MATERIALES: Métodos de energía
 
Elementos de-maquinas - prof moro - ifsc
Elementos de-maquinas - prof moro - ifscElementos de-maquinas - prof moro - ifsc
Elementos de-maquinas - prof moro - ifsc
 
Taller S10 Costos y Presupuestos - Parte 1
Taller S10 Costos y Presupuestos - Parte 1Taller S10 Costos y Presupuestos - Parte 1
Taller S10 Costos y Presupuestos - Parte 1
 
Elaboracion de presupuesto en s10 03
Elaboracion de presupuesto en s10 03Elaboracion de presupuesto en s10 03
Elaboracion de presupuesto en s10 03
 
Elementos de máquinas sarkis melconian
Elementos de máquinas   sarkis melconianElementos de máquinas   sarkis melconian
Elementos de máquinas sarkis melconian
 
Apostila elementos de_maquinas
Apostila elementos de_maquinasApostila elementos de_maquinas
Apostila elementos de_maquinas
 
Elementos de maquinas apostila Senai
Elementos de maquinas apostila SenaiElementos de maquinas apostila Senai
Elementos de maquinas apostila Senai
 
Elaboracion de presupuesto en s10 01
Elaboracion de presupuesto en s10 01Elaboracion de presupuesto en s10 01
Elaboracion de presupuesto en s10 01
 
Formulario hidrostatica
Formulario hidrostaticaFormulario hidrostatica
Formulario hidrostatica
 
Formulas de Física
Formulas de Física Formulas de Física
Formulas de Física
 
Ejercicios de presión
Ejercicios de presiónEjercicios de presión
Ejercicios de presión
 
Formulas de física
Formulas de físicaFormulas de física
Formulas de física
 
Formulas2
Formulas2Formulas2
Formulas2
 
Formulario fisica
Formulario fisicaFormulario fisica
Formulario fisica
 
Exercicios resolvidos
Exercicios resolvidosExercicios resolvidos
Exercicios resolvidos
 
Ejercicios resueltos blog definitivo
Ejercicios  resueltos  blog definitivoEjercicios  resueltos  blog definitivo
Ejercicios resueltos blog definitivo
 
Problemas resueltos fluidos
Problemas resueltos fluidosProblemas resueltos fluidos
Problemas resueltos fluidos
 
Ejercicios resueltos dinamica de fluidos
Ejercicios resueltos dinamica de fluidosEjercicios resueltos dinamica de fluidos
Ejercicios resueltos dinamica de fluidos
 

Semelhante a Lista de exercicios elementos de máquinas

Lista de exercicios elementos de máquinas
Lista de exercicios elementos de máquinasLista de exercicios elementos de máquinas
Lista de exercicios elementos de máquinashuanderlei
 
Analise ergonomica-soldador
Analise ergonomica-soldadorAnalise ergonomica-soldador
Analise ergonomica-soldadorCosmo Palasio
 
Curso de Introdução a Pesquisa Operacional - Matusalém Vieira Martins.pdf
Curso de Introdução a Pesquisa Operacional - Matusalém Vieira Martins.pdfCurso de Introdução a Pesquisa Operacional - Matusalém Vieira Martins.pdf
Curso de Introdução a Pesquisa Operacional - Matusalém Vieira Martins.pdfd7503d
 
Apostila de higiene e segurança do trabalho
Apostila de higiene e segurança do trabalhoApostila de higiene e segurança do trabalho
Apostila de higiene e segurança do trabalhoSergio Silva
 
39560783 apostila-seguranca-do-trabalho-i
39560783 apostila-seguranca-do-trabalho-i39560783 apostila-seguranca-do-trabalho-i
39560783 apostila-seguranca-do-trabalho-iPriscila Alves
 
Trabalho sobre rebites
Trabalho sobre rebitesTrabalho sobre rebites
Trabalho sobre rebiteshuanderlei
 
Fundamentos de Topografia
Fundamentos de TopografiaFundamentos de Topografia
Fundamentos de TopografiaWerika Goo
 
Tese final Rui Duarte.pdf
Tese final Rui Duarte.pdfTese final Rui Duarte.pdf
Tese final Rui Duarte.pdfsusana350279
 
Agregados 2012 2 janaína
Agregados 2012 2 janaínaAgregados 2012 2 janaína
Agregados 2012 2 janaínaRafael L. Fraga
 
Sgp tutorial tablet e web
Sgp tutorial tablet e webSgp tutorial tablet e web
Sgp tutorial tablet e webMixLeonel
 
Apostila hidráulica- v2-atualizada_
Apostila hidráulica- v2-atualizada_Apostila hidráulica- v2-atualizada_
Apostila hidráulica- v2-atualizada_Jardel Paixao
 

Semelhante a Lista de exercicios elementos de máquinas (20)

Lista de exercicios elementos de máquinas
Lista de exercicios elementos de máquinasLista de exercicios elementos de máquinas
Lista de exercicios elementos de máquinas
 
Trabalho sobre rebites
Trabalho sobre rebitesTrabalho sobre rebites
Trabalho sobre rebites
 
Analise ergonomica-soldador
Analise ergonomica-soldadorAnalise ergonomica-soldador
Analise ergonomica-soldador
 
Boletim 2º quad. 2011
Boletim 2º quad. 2011Boletim 2º quad. 2011
Boletim 2º quad. 2011
 
Curso de Introdução a Pesquisa Operacional - Matusalém Vieira Martins.pdf
Curso de Introdução a Pesquisa Operacional - Matusalém Vieira Martins.pdfCurso de Introdução a Pesquisa Operacional - Matusalém Vieira Martins.pdf
Curso de Introdução a Pesquisa Operacional - Matusalém Vieira Martins.pdf
 
Sht2009
Sht2009Sht2009
Sht2009
 
Apostila de higiene e segurança do trabalho
Apostila de higiene e segurança do trabalhoApostila de higiene e segurança do trabalho
Apostila de higiene e segurança do trabalho
 
Manual projeto (3)
Manual projeto (3)Manual projeto (3)
Manual projeto (3)
 
PÓS-QUALIFICAÇÃO - NERES TIDD
PÓS-QUALIFICAÇÃO - NERES TIDDPÓS-QUALIFICAÇÃO - NERES TIDD
PÓS-QUALIFICAÇÃO - NERES TIDD
 
39560783 apostila-seguranca-do-trabalho-i
39560783 apostila-seguranca-do-trabalho-i39560783 apostila-seguranca-do-trabalho-i
39560783 apostila-seguranca-do-trabalho-i
 
Trabalho sobre rebites
Trabalho sobre rebitesTrabalho sobre rebites
Trabalho sobre rebites
 
Fundamentos de Topografia
Fundamentos de TopografiaFundamentos de Topografia
Fundamentos de Topografia
 
Tese final Rui Duarte.pdf
Tese final Rui Duarte.pdfTese final Rui Duarte.pdf
Tese final Rui Duarte.pdf
 
Apostila fengseg
Apostila fengsegApostila fengseg
Apostila fengseg
 
Apostila fengseg
Apostila fengsegApostila fengseg
Apostila fengseg
 
Agregados 2012 2 janaína
Agregados 2012 2 janaínaAgregados 2012 2 janaína
Agregados 2012 2 janaína
 
Manual de tcc
Manual de tccManual de tcc
Manual de tcc
 
Tqs 04-conceitos e modelos
Tqs 04-conceitos e modelosTqs 04-conceitos e modelos
Tqs 04-conceitos e modelos
 
Sgp tutorial tablet e web
Sgp tutorial tablet e webSgp tutorial tablet e web
Sgp tutorial tablet e web
 
Apostila hidráulica- v2-atualizada_
Apostila hidráulica- v2-atualizada_Apostila hidráulica- v2-atualizada_
Apostila hidráulica- v2-atualizada_
 

Lista de exercicios elementos de máquinas

  • 1. UNIVERSIDADE TUIUTI DO PARANÁ ERIC LUIZ CAETANO FELIPE TOLEDO DE ALMEIDA HELENA LORUSSO JÚLIO CÉSAR DROSZCZAK LISTA DE EXERCÍCIOS – ELEMENTOS DE MÁQUINAS CURITIBA 2015
  • 2. ERIC LUIZ CAETANO FELIPE TOLEDO DE ALMEIDA HELENA LORUSSO JÚLIO CÉSAR DROSZCZAK LISTA DE EXERCÍCIOS – ELEMENTOS DE MÁQUINAS Trabalho apresentado ao Curso de Engenharia Mecânica, da Universidade Tuiuti do Paraná, como requisito avaliativo do 1º bimestre da disciplina de Elementos de Maquinas I. Professor: Paulo Lagos CURITIBA 2015
  • 3. SUMÁRIO MOVIMENTO CIRCULAR..............................................................................8 EXERCÍCIO 01 –.........................................................................................8 EXERCÍCIO 1.1 (Eric) –..............................................................................9 EXERCÍCIO 1.2 (Felipe) –........................................................................10 EXERCÍCIO 1.3 (Helena) - .......................................................................11 EXERCÍCIO 1.4 (Júlio) –..........................................................................11 EXERCÍCIO 02 –.......................................................................................12 EXERCÍCIO 2.1 (Eric) –............................................................................13 EXERCÍCIO 2.2 (Felipe) –........................................................................14 EXERCÍCIO 2.3 (Helena) - .......................................................................14 EXERCÍCIO 2.4 (Júlio) –..........................................................................15 EXERCÍCIO 03 –.......................................................................................16 EXERCÍCIO 3.1 (Eric) –............................................................................17 EXERCÍCIO 3.2 (Felipe) –........................................................................17 EXERCÍCIO 3.3 (Helena) - .......................................................................17 EXERCÍCIO 3.4 (Júlio) –..........................................................................18 RELAÇÃO DE TRANSMISSÃO...................................................................19 EXECÍCIO 04 – .........................................................................................19 EXERCÍCIO 4.1 (Eric) –............................................................................21 EXERCÍCIO 4.2 (Felipe) –........................................................................22 EXERCÍCIO 4.3 (Helena) - .......................................................................24 EXERCÍCIO 4.4 (Júlio) –..........................................................................26 EXERCÍCIO 05 –.......................................................................................27 EXERCICIO 5.1 (Eric) –............................................................................30 EXERCICIO 5.2 (Felipe) –........................................................................32 EXERCÍCIO 5.3 (Helena) - .......................................................................34 EXERCÍCIO 5.4 (Júlio) –..........................................................................36 TORÇÃO SIMPLES .....................................................................................39 EXERCÍCIO 06 –.......................................................................................39 EXERCÍCIO 6.1 (Eric) –............................................................................39 EXERCICIO 6.2 (Felipe) –........................................................................40 EXERCÍCIO 6.3 (Helena) - .......................................................................40 EXERCÍCIO 6.4 (Júlio) –..........................................................................40 EXERCÍCIO 07 –.......................................................................................41 EXERCÍCIO 7.1 (Eric) –............................................................................41
  • 4. EXERCICIO 7.2 (Felipe) –........................................................................42 EXERCÍCIO 7.3 (Helena) - .......................................................................42 EXERCÍCIO 7.4 (Júlio) –..........................................................................43 TORQUE NAS TRANSMISSÕES ................................................................43 EXERCÍCIO 08 –.......................................................................................43 EXERCÍCIO 8.1 (Eric) –............................................................................44 EXERCICIO 8.2 (Felipe) –........................................................................45 EXERCÍCIO 8.3 (Helena) - .......................................................................46 EXERCÍCIO 8.4 (Júlio) –..........................................................................46 POTÊNCIA ...................................................................................................47 EXERCÍCIO 09 –.......................................................................................47 EXERCÍCIO 9.1 (Eric) –............................................................................48 EXERCICIO 9.2 (Felipe) –........................................................................49 EXERCÍCIO 9.3 (Helena) - .......................................................................49 EXERCÍCIO 9.4 (Júlio) –..........................................................................50 EXERCÍCIO 10 –.......................................................................................51 EXERCÍCIO 10.1 (Eric) –..........................................................................51 EXERCICIO 10.2 (Felipe) –......................................................................52 EXERCÍCIO 10.3 (Helena) - .....................................................................53 EXERCÍCIO 10.4 (Júlio) –........................................................................53 EXERCÍCIO 11 –.......................................................................................54 EXERCÍCIO 11.1 (Eric) –..........................................................................54 EXERCICIO 11.2 (Felipe) –......................................................................55 EXERCÍCIO 11.3 (Helena) - .....................................................................55 EXERCÍCIO 11.4 (Júlio) –........................................................................56 EXERCÍCIO 12 –.......................................................................................56 EXERCÍCIO 12.1 (Eric) –..........................................................................57 EXERCICIO 12.2 (Felipe) –......................................................................57 EXERCÍCIO 12.3 (Helena) - .....................................................................58 EXERCÍCIO 12.4 (Júlio) –........................................................................58 EXERCÍCIO 13 –.......................................................................................59 EXERCÍCIO 13.1 (Eric) –..........................................................................61 EXERCICIO 13.2 (Felipe) –......................................................................63 EXERCÍCIO 13.3 (Helena) - .....................................................................65 EXERCÍCIO 13.4 (Júlio) –........................................................................67 EXERCÍCIO 14 –.......................................................................................69 EXERCÍCIO 14.1 (Eric) –..........................................................................72
  • 5. EXERCICIO 14.2 (Felipe) –......................................................................75 EXERCÍCIO 14.3 (Helena) - .....................................................................78 EXERCÍCIO 14.4 (Júlio) –........................................................................81 RELAÇÃO DE TRANSMISSÃO POR ENGRENAGENS .............................84 EXERCÍCIO 15 –.......................................................................................84 EXERCÍCIO 15.1 (Eric) –..........................................................................87 EXERCICIO 15.2 (Felipe) -.......................................................................89 EXERCÍCIO 15.3 (Helena) –.....................................................................91 EXERCÍCIO 15.4 (Júlio) –........................................................................93 REFERÊNCIAS ............................................................................................97
  • 6. ÍNDICE DE FIGURAS FIGURA 1 .......................................................................................................8 FIGURA 2 .......................................................................................................9 FIGURA 3 .....................................................................................................10 FIGURA 4 .....................................................................................................11 FIGURA 5 .....................................................................................................12 FIGURA 6 .....................................................................................................13 FIGURA 7 .....................................................................................................14 FIGURA 8 .....................................................................................................14 FIGURA 9 .....................................................................................................15 FIGURA 10 ...................................................................................................16 FIGURA 11 ...................................................................................................18 FIGURA 12 ...................................................................................................18 FIGURA 13 ...................................................................................................19 FIGURA 14 ...................................................................................................21 FIGURA 15 ...................................................................................................23 FIGURA 16 ...................................................................................................24 FIGURA 17 ...................................................................................................26 FIGURA 18 ...................................................................................................28 FIGURA 19 ...................................................................................................30 FIGURA 20 ...................................................................................................32 FIGURA 21 ...................................................................................................34 FIGURA 22 ...................................................................................................37 FIGURA 23 ...................................................................................................39 FIGURA 24 ...................................................................................................40 FIGURA 25 ...................................................................................................41 FIGURA 26 ...................................................................................................42 FIGURA 27 ...................................................................................................42 FIGURA 28 ...................................................................................................43 FIGURA 29 ...................................................................................................43 FIGURA 30 ...................................................................................................44 FIGURA 31 ...................................................................................................45 FIGURA 32 ...................................................................................................46 FIGURA 33 ...................................................................................................47 FIGURA 34 ...................................................................................................48 FIGURA 35 ...................................................................................................49 FIGURA 36 ...................................................................................................50 FIGURA 37 ...................................................................................................50 FIGURA 38 ...................................................................................................51 FIGURA 39 ...................................................................................................52 FIGURA 40 ...................................................................................................52 FIGURA 41 ...................................................................................................53 FIGURA 42 ...................................................................................................56 FIGURA 43 ...................................................................................................57 FIGURA 44 ...................................................................................................58 FIGURA 45 ...................................................................................................59 FIGURA 46 ...................................................................................................61 FIGURA 47 ...................................................................................................63
  • 7. FIGURA 48 ...................................................................................................67 FIGURA 49 ...................................................................................................69 FIGURA 50 ...................................................................................................73 FIGURA 51 ...................................................................................................75 FIGURA 52 ...................................................................................................81 FIGURA 53 ...................................................................................................84 FIGURA 54 ...................................................................................................89 FIGURA 55 ...................................................................................................94
  • 8. MOVIMENTO CIRCULAR EXERCÍCIO 01 – A roda da figura possui d = 300 mm e gira com velocidade angular ω = 10π rad/s. FIGURA 1 FONTE: MELCONIAN. SARKIS: Elementos de Máquinas – 9 Ed. Revisada. Editora Érica. Ano 2009. Determine: a) Período (T) 𝑇 = 2𝜋 𝜔 𝑇 = 2𝜋 10𝜋 𝑇 = 1 5 𝑠 = 0,2 𝑠 b) Frequência (f) 𝑓 = 1 𝑇 𝑓 = 1 0,2 = 5 𝐻𝑧 c) Rotação (n) 𝑛 = 60𝑓 𝑛 = 60 . 5 𝑛 = 300 𝑟𝑝𝑚 d) Velocidade Periférica (𝒱p) 𝑟 = 𝑑 2 𝑟 = 0,3 2 𝑟 = 0,15 𝑚 𝒱𝑝 = 𝜔. 𝑟 𝒱𝑝 = 10𝜋 . 0,15
  • 9. 𝒱𝑝 = 1,5𝜋 𝑚/𝑠 = 4,71 𝑚/ EXERCÍCIO 1.1 (Eric) – Uma partícula se movendo em MCU (Movimento Circular Uniforme) completa uma volta a cada 10 segundos em uma circunferência de diâmetro d= 10 cm. Determine: FIGURA 2 FONTE: Autor Júlio César Droszczak. a) Período (T) O enunciado nos diz que a partícula completa uma volta a cada 10 segundos, logo o período (T)= 10 s. b) Velocidade angular (𝜔) 𝑇 = 2𝜋 𝜔  10 = 2𝜋 𝜔  𝜔 = 2𝜋 10 = 0,2𝜋 rad/s  c) Frequência (f)  𝑓 = 1 𝑇 𝑓 = 1 10 = 0,1 𝐻𝑧  d) Rotação (n) 𝑛 = 60𝑓 𝑛 = 60 . 0,1 = 6 𝑟𝑝𝑚 e) Velocidade Periférica (Vp) 𝑟 = 𝑑 2
  • 10. 𝑟 = 10 2 = 5𝑐𝑚 = 0,05𝑚 𝑉𝑝 = 𝜔. 𝑟 𝑉𝑝 = 0,2𝜋 . 0,05 = 0,01𝜋 𝑚 𝑠⁄ 𝑜𝑢 0,031 𝑚/𝑠 EXERCÍCIO 1.2 (Felipe) – Uma polia que gira no sentido horário e cujo diâmetro é d = 500mm, sua rotação é de n = 600rpm. Determine: a) Frequência; b) Velocidade periférica; Velocidade angular e d) Período. FIGURA 3 FONTE: http://www.casadaspolias.com.br/produtos/especificacoes-tecnicas.html a) Frequência (𝑓) 𝑛 = 60. 𝑓 600 = 60. 𝑓 𝑓 = 600 60 = 10 𝐻𝑍 b) Velocidade periférica (𝑉𝑝) 𝑟 = 𝑑 2 = 500 2 = 250 𝑚𝑚 𝑟 = 0,25 𝑚 ( 𝑉𝑝) = 20 . 0,25 = 5𝜋 𝑚 𝑠 𝑜𝑢 15,7 𝑚 𝑠 c) Período (𝑇) 𝑇 = 1 𝑓 = 1 10 = 0,1𝑠 d) Velocidade angular (ω) 𝜔 = 2𝜋 𝑇 = 2𝜋 0,1 = 62,83 𝑟𝑎𝑑 𝑠
  • 11. EXERCÍCIO 1.3 (Helena) - A roda trabalha numa rotação n=1710rpm. Determine: a) Velocidade Angular (𝜔) 𝜔 = 1710𝜋 30 𝜔 = 1710𝜋 30 𝜔 = 57𝜋 𝑟𝑎𝑑/𝑠 b) Período (T) 𝑇 = 2𝜋 𝜔 𝑇 = 2𝜋 57𝜋 𝑇 = 0.035𝑠 c) Frequência (𝑓) 𝑓 = 1 𝑇 𝑓 = 1 0,035 𝑓 = 28,5 𝐻𝑧 EXERCÍCIO 1.4 (Júlio) – Uma pedra de esmeril de d = 120 mm e acionada por um motor de rotação n = 1200 rpm. Determine: FIGURA 4 FONTE: http://www.vegamaquinas.com.br/retifica-para-torno-pr-1313-229827.htm a) Velocidade angular (ω) 𝜔 = 𝜋𝑛 30 𝜔 = 𝜋1200 30
  • 12. 𝜔 = 40𝜋 𝑟𝑎𝑑/𝑠 b) Velocidade Periférica (𝒱p) 𝒱𝑝 = 𝜔. 𝑟 𝒱𝑝 = 40𝜋. 0.06 𝒱𝑝 = 2.4𝜋 𝑚/𝑠 c) Período (T) 𝑇 = 2𝜋 𝜔 𝑇 = 2𝜋 40𝜋 𝑇 = 1 20 = 0,05 𝑠 d) Frequência (f) 𝑓 = 1 𝑇 𝑓 = 1 0,05 𝑓 = 20 𝐻𝑧 EXERCÍCIO 02 – O motor elétrico possui como característica de desempenho a rotação n = 1740rpm. Determine as seguintes características de desempenho do motor: FIGURA 5 FONTE: MELCONIAN. SARKIS: Elementos de Máquinas – 9 Ed. Revisada. Editora Érica. Ano 2009. a) Velocidade Angular (ω) 𝜔 = 𝜋𝑛 30 𝜔 = 1470𝜋 30 𝜔 = 58𝜋 𝑟𝑎𝑑/𝑠 b) Período (T)
  • 13. 𝑇 = 2𝜋 𝜔 𝑇 = 2𝜋 58𝜋 𝑇 = 1 29 = 0,0345 𝑠 c) Frequência (f) 𝑓 = 1 𝑇 𝑓 = 1 0,0345 𝑓 = 29 𝐻𝑧 EXERCÍCIO 2.1 (Eric) – (Cefet-SP) Um motor executa 600 rpm. Determine a frequência e o período no SI. Determine também a velocidade angular. FIGURA 6 FONTE: MELCONIAN. SARKIS: Elementos de Máquinas – 9 Ed. Revisada. Editora Érica. Ano 2009. a) Frequência (f) 𝑛 = 60𝑓 600 = 60𝑓 𝑓 = 600 60 = 10 𝐻𝑧 b) Período (T) 𝑓 = 1 𝑇 10 = 1 𝑇 𝑇 = 1 10 = 0,1 𝑠 c) Velocidade Angular (ω)  𝑇 = 2𝜋 𝜔
  • 14. 0,1 = 2𝜋 𝜔  = 2𝜋 0,1 = 20𝜋 𝑟𝑎𝑑/𝑠 EXERCÍCIO 2.2 (Felipe) – Um motor elétrico tem como característica um período de T= 0,029s. Determine as seguintes características de desempenho deste motor: a) Frequência (𝑓); b) Rotação (𝑛) e c) Velocidade angular (). FIGURA 7 FONTE: MELCONIAN. SARKIS: Elementos de Máquinas – 9 Ed. Revisada. Editora Érica. Ano 2009. a) Frequência (𝑓) 𝑓 = 1 𝑇 = 0,029 = 34,48 𝐻𝑧 b) Rotação (𝑛) 𝑛 = 60. 𝑓 𝑛 = 60.34,48 = 2069 𝑟𝑝𝑚 c) Velocidade angular (𝑤) 𝜔 = 2𝜋 𝑇 = 2𝜋 0,029 = 216,7 𝑟𝑎𝑑 𝑠 𝑜𝑢 68,96 𝑟𝑎𝑑/𝑠 EXERCÍCIO 2.3 (Helena) - O motor elétrico possui como característica de desempenho a rotação n = 2730rpm. Determine as seguintes características de desempenho do motor: FIGURA 8 FONTE: MELCONIAN. SARKIS: Elementos de Máquinas – 9 Ed. Revisada. Editora Érica. Ano 2009. d) Velocidade Angular (ω)
  • 15. 𝜔 = 𝜋𝑛 30 𝜔 = 2730𝜋 30 𝜔 = 91𝜋 𝑟𝑎𝑑/𝑠 e) Período (T) 𝑇 = 2𝜋 𝜔 𝑇 = 2𝜋 91𝜋 𝑇 = 1 45.5 = 0,022 𝑠 f) Frequência (f) 𝑓 = 1 𝑇 𝑓 = 1 0,022 𝑓 = 45.45 𝐻𝑧 EXERCÍCIO 2.4 (Júlio) – O pneu de um carro gira a uma rotação por minuto de n = 793 rpm . Determine as seguintes características de desempenho do carro: FIGURA 9 FONTE: http://pt.clipartlogo.com/premium/detail/car-or-truck-tire-line-art_109442720.html a) Velocidade Angular (ω) 𝜔 = 𝜋𝑛 30 𝜔 = 𝜋793 30 𝜔 = 793𝜋 30 𝑟𝑎𝑑/𝑠
  • 16. b) Período (T) 𝑇 = 2𝜋 𝜔 𝑇 = 2𝜋 793𝜋 30 𝑇 = (2𝜋)( 30 793𝜋 ) 𝑇 = 60 793 = 0,0756 𝑠 c) Frequência (f) 𝑓 = 1 𝑇 𝑓 = 1 0,0756 𝑓 = 13,23 𝐻𝑧 EXERCÍCIO 03 – O ciclista monta uma bicicleta aro 26 (d = 660 mm), viajando com um movimento que faz com que as rodas girem n = 240 rpm. Qual a velocidade do ciclista? FIGURA 10 FONTE: MELCONIAN. SARKIS: Elementos de Máquinas – 9 Ed. Revisada. Editora Érica. Ano 2009. Velocidade Periférica (𝒱p) 𝒱p = 𝜋. 𝑛. 𝑟 30 𝒱p = 𝜋. 240.0,33 30 𝒱p = 8,29𝑚/𝑠 Transformando para km/h: 𝒱p = 8,29 𝑥 3,6 𝒱p = 30𝑘𝑚/ℎ
  • 17. EXERCÍCIO 3.1 (Eric) – Um carrinho de rolimã foi construído com rodas de raio = 15 cm. Em uma descida, as rodas do carrinho atingiram 600 rpm. Determine a velocidade que o carrinho atingiu à essa rotação. 𝑉 = 𝜋. 𝑛. 𝑟 30 𝑉 = 𝜋 . 600 . 0,15 30 𝑉 = 9,42 𝑚/𝑠 Ou 𝑉 = 33,91 𝑘𝑚/ℎ EXERCÍCIO 3.2 (Felipe) – Um motoqueiro passeia em sua moto, sendo que o diâmetro de suas rodas é de (d= 720 mm), neste trajeto o movimento que faz com que as rodas girem a uma frequência de 6,83 HZ. Qual é a velocidade do motoqueiro? E qual é a sua rotação? 𝑛 = 60. 𝑓 𝑛 = 60 . 6,83 = 410 𝑟𝑝𝑚 𝑇 = 1 𝑓 = 1 6,83 = 0,146𝑠 𝜔 = 2𝜋 𝑇 = 2𝜋 0,146 = 43,03 𝑟𝑎𝑑/𝑠 𝑉𝑝 = 𝑤. 𝑟 𝑟 = 720 2 = 360 1000 = 0,36 𝑚 𝑉𝑝 = 43,03 .0,36 = 15,49 𝑚 𝑠 𝑜𝑢 55,76 𝑘𝑚/ℎ EXERCÍCIO 3.3 (Helena) - Um ventilador de D = 522mm, trabalhando com um movimento circular que faz com que as pás girem a n = 28500 rpm. Qual a velocidade periférica do ventilador?
  • 18. FIGURA 11 FONTE: http://portuguese.alibaba.com/product-gs/hot-new-products-for-2015-kitchen-outer-rotor- 450mm-ac-axial-fan-60210360658.html Velocidade Periférica (𝒱p) 𝒱p = 𝜋. 𝑛. 𝑟 30 𝒱p = 𝜋. 2850.0,261 30 𝒱p = 77,9𝑚/𝑠 Transformando para km/h: 𝒱p = 77,9 𝑥 3,6 𝒱p = 280,42𝑘𝑚/ℎ EXERCÍCIO 3.4 (Júlio) – Uma Roda D’água de diâmetro D = 236,22in, gira com uma rotação de n = 27 rpm. Qual a velocidade da Roda D’água? (1in = 25,4mm). FIGURA 12 FONTE: http://www.cepa.if.usp.br/energia/energia1999/Grupo2B/Hidraulica/roda.htm. Transformando Polegadas em metros: 1in = 25,4mm 236,22 x 25,4 = 5999,999mm ≅ 6m
  • 19. Velocidade Periférica (𝒱p) 𝒱 = 𝜋. 𝑛. 𝑟 30 𝒱 = 𝜋. 27.3 30 𝒱 = 81𝜋 30 𝒱 = 8,48 𝑚/𝑠 𝒱 = 30,54 𝑘𝑚/ℎ RELAÇÃO DE TRANSMISSÃO EXECÍCIO 04 – A transmissão por correias é composta por duas polias com os seguintes diâmetros, respectivamente: Polia 1 (motora) – d1=100mm Polia 2 (movida) – d2=180mm FIGURA 13 FONTE: MELCONIAN. SARKIS: Elementos de Máquinas – 9 Ed. Revisada. Editora Érica. Ano 2009. A polia 1 atua com velocidade angular ω1 = 39π rad/s. Determinar: a) Período da polia 1 (T1) 𝑇1 = 2𝜋 𝜔 𝑇1 = 2𝜋 39𝜋 𝑇1 = 0,051𝑠 b) Frequência da polia 1 (f1) 𝑓1 = 1 𝑇1 𝑓1 = 1 0,051 𝑓1 = 19,5 𝐻𝑧
  • 20. c) Rotação da polia 1 (n1) 𝑛1 = 60. 𝑓1 𝑛1 = 60 . 19,5 𝑛1 = 1170 𝑟𝑝𝑚 d) Velocidade Angular da polia 2 (ω2) 𝜔2 = 𝜔1 𝑑1 𝑑2 𝜔2 = 39𝜋 . 100 180 𝜔2 = 21,67𝜋 𝑟𝑎𝑑/𝑠 e) Frequência da polia 2 (f2) 𝑓2 = 1 𝑇2 𝑓2 = 1 0,092 𝑓2 ≅ 10,835 𝐻𝑧 f) Período da polia 2 (T2) 𝑇2 = 2𝜋 𝜔2 𝑇2 = 2𝜋 21,67𝜋 𝑇2 ≅ 0,092 𝑠 g) Rotação da polia 2 (n2) 𝑛2 = 60. 𝑓2 𝑛2 = 60 . 10,835 𝑛2 = 650 𝑟𝑝𝑚 h) Velocidade periférica da transmissão (𝒱p) 𝒱𝑝 = 𝜔. 𝑟 𝒱𝑝 ≅ 21,67𝜋 . 0,09 𝒱𝑝 ≅ 6,127 𝑚/𝑠 i) Relação de transmissão (i)
  • 21. 𝐼 = 𝑑2 𝑑1 𝐼 = 180 100 𝐼 = 1,8 EXERCÍCIO 4.1 (Eric) – Uma transmissão por correias ampliadora de velocidade possui as seguintes características: Polia 1 motora 𝑑1 = 160 𝑚𝑚 Polia 2 movida 𝑑2 = 140 𝑚𝑚 A polia 1 atua com velocidade angular 𝜔1 = 50𝜋 𝑟𝑎𝑑/𝑠. FIGURA 14 FONTE: Autor Eric Luiz Caetano. Determine: a) Período da polia 1 (𝑇1) 𝑇1 = 2𝜋 𝜔1 𝑇1 = 2𝜋 50𝜋 = 2 50 𝑠 𝑜𝑢 0,04 𝑠 b) Frequência da polia 1 (𝑓1) 𝑓1 = 1 𝑇1 𝑓1 = 1 0,04 = 25 𝐻𝑧 c) Rotação da polia 1 (𝑛1) 𝑛1 = 60𝑓1 𝑛1 = 60 . 25 = 1500 𝑟𝑝𝑚 d) Velocidade angular da polia 2 (𝜔2)
  • 22. 𝜔2 = 𝜔1 𝑑1 𝑑2 𝜔2 = 50𝜋 . 160 140 = 57,14𝜋 𝑟𝑎𝑑/𝑠 e) Frequência da polia 2 (𝑓2) 𝑓2 = 𝜔2 2𝜋 𝑓2 = 57,14𝜋 2𝜋 = 28,57 𝐻𝑧 f) Período da polia 2 (𝑇2) 𝑇2 = 2𝜋 𝜔2 𝑇2 = 2𝜋 57,14𝜋 = 0,035 𝑠 g) Rotação da polia 2 (𝑛2) 𝑛2 = 𝑛1 𝑑1 𝑑2 𝑛2 = 1500 . 160 140 = 1714,28 𝑟𝑝𝑚 h) Velocidade periférica da transmissão (Vp) 𝑉𝑝 = 𝜔1 𝑟1 𝑉𝑝 = 50𝜋 . 0,08 = 4𝜋 𝑚 𝑠⁄ 𝑜𝑢 12,56 𝑚/𝑠 i) Relação de transmissão (i) 𝐼 = 𝑑2 𝑑1 𝐼 = 140 160 = 0,875 EXERCÍCIO 4.2 (Felipe) – A transmissão por correias é composta por duas polias, cujo seus diâmetros são: Polia 1 Motora d1 = 50 mm Polia 2 Movida d2 = 80 mm
  • 23. FIGURA 15 FONTE: MELCONIAN. SARKIS: Elementos de Máquinas – 9 Ed. Revisada. Editora Érica. Ano 2009. a) Período da polia (𝑇1) 𝑇1 = 2𝜋 𝜔1 = 2𝜋 28𝜋 = 0,0714 𝑠 b) Frequência da polia ( 𝑓1): 𝑓1 = 1 𝑇1 = 28 2 = 14 𝐻𝑧 c) Rotação da polia (𝑁1): 𝑁1 = 60. 𝑓1 𝑁1 = 60 . 14 = 840 𝑟𝑝𝑚 d) Velocidade angular da polia 2 (𝜔2): 𝜔2 = 𝜔1. 𝑑1 𝑑2 = 28𝜋. 50 80 = 17,5 𝜋 𝑟𝑎𝑑 𝑠 e) Frequência da polia 2 (𝑓2): 𝑓2 = 𝜔2 2𝜋 = 17,5𝜋 2𝜋 = 8,75 𝐻𝑧 f) Período da polia 2 (𝑇2): 𝑇2 = 2𝜋 17,5𝜋 = 0,114 𝑠 g) Rotação da polia 2 (𝑛2): 𝑛2 = 𝑛1. 𝑑1 𝑑2 = 840.50 80 = 525 𝑟𝑝𝑚 h) Velocidade periférica (𝑉𝑝):
  • 24. 𝑉𝑝 = 𝜔1. 𝑑1 2 = 28.0,05 2 = 0,7 i) Relação de transmissão (𝑖) 𝑖 = 𝑑2 𝑑1 = 80 50 = 1,6 EXERCÍCIO 4.3 (Helena) - Uma transmissão por correias composta por duas polias: Polia 1 – d1=120mm Polia 2 – d2=220mm A polia 1 atua com rotação n=1140rpm. Determine: FIGURA 16 FONTE: Autor Júlio César Droszczak a) Velocidade Angular da polia 1 (𝜔1) 𝜔1 = 𝜋𝑛 30 𝜔1 = 1140𝜋 30 𝜔1 = 38𝜋 𝑟𝑎𝑑/𝑠 b) Frequência da polia 1 (𝑓1) 𝑓1 = 1 𝑇1 𝑇1 = 2𝜋 𝜔1 𝑓1 = 𝜔1 2𝜋 𝑓1 = 38𝜋 2𝜋 𝑓1 = 19 𝐻𝑧 c) Período da polia 1 (𝑇1)
  • 25. 𝑇1 = 2𝜋 𝜔1 𝑇1 = 2𝜋 38𝜋 𝑇1 = 0,0526𝑠 d) Velocidade Angular da polia 2 (𝜔2) 𝜔2 = 𝜔1 𝑑1 𝑑2 𝜔2 = 38𝜋 . 120 220 𝜔2 = 38𝜋 . 120 220 𝜔2 = 20,727𝜋 𝑟𝑎𝑑/𝑠 e) Período da polia 2 (𝑇2) 𝑇2 = 2𝜋 𝜔2 𝑇2 = 2𝜋 20,727𝜋 𝑇1 = 0,096𝑠 f) Frequência da polia 2 (𝑓2) 𝑓2 = 1 𝑇2 𝑓2 = 1 0,096 𝑓2 = 10,36 𝐻𝑧 g) Rotação da polia 2 (𝑛2) 𝑛2 = 60𝑓 𝑛2 = 60 . 10,36 𝑛2 = 621,6𝑟𝑝𝑚 h) Velocidade periférica da transmissão (Vp) 𝑉𝑝 = 𝜔1. 𝑟1 𝑉𝑝 = 38𝜋 . 0,06 𝑉𝑝 = 7,16 𝑚/𝑠 i) Relação de transmissão (I)
  • 26. 𝐼 = 𝑑2 𝑑1 𝐼 = 220 120 𝐼 = 1,83 EXERCÍCIO 4.4 (Júlio) – Um motor que esta chavetado a uma polia de d1=160mm de diâmetro, desenvolve n1=1200 rpm e move um eixo de transmissão cuja polia tem d2=300mm de diâmetro. Calcule: FIGURA 17 FONTE: MELCONIAN. SARKIS: Elementos de Máquinas – 9 Ed. Revisada. Editora Érica. Ano 2009. a) Frequência da polia 1 (f1) 𝑛1 = 60. 𝑓1 1200 = 60 . 𝑓1 𝑓1 = 1200 60 𝑓1 = 20 𝐻𝑧 b) Período da polia 1 (T1) 𝑓1 = 1 𝑇1 20 = 1 𝑇1 𝑇1 = 1 20 𝑇1 = 0.05 𝑠 c) Velocidade Angular da polia 1 (ω 1) 𝜔1 = 𝜋. 1200 30 𝜔1 = 𝜋. 1200 30 𝜔1 = 40𝜋 𝑟𝑎𝑑/𝑠 d) Velocidade Angular da polia 2 (ω2)
  • 27. 𝜔2 = 𝜔1 𝑑1 𝑑2 𝜔2 = 40𝜋 . 160 300 𝜔2 = 21.33𝜋 𝑟𝑎𝑑/𝑠 e) Período da polia 2 (T2) 𝑇2 = 2𝜋 𝜔2 𝑇2 = 2𝜋 21,33𝜋 𝑇2 ≅ 0,094 𝑠 f) Frequência da polia 2 (f2) 𝑓2 = 1 𝑇2 𝑓2 = 1 0,094 𝑓2 ≅ 10,638 𝐻𝑧 g) Rotação da polia 2 (n2) 𝑛2 = 60. 𝑓2 𝑛2 = 60 . 10,638 𝑛2 = 638.28 𝑟𝑝𝑚 h) Velocidade periférica da transmissão (𝒱p) 𝒱𝑝 = 𝜔. 𝑟 𝒱𝑝 ≅ 21,33𝜋 . 0,15 𝒱𝑝 ≅ 10,05 𝑚/𝑠 i) Relação de transmissão (i) 𝐼 = 𝑑2 𝑑1 𝐼 = 300 160 𝐼 = 1,875 EXERCÍCIO 05 – A transmissão por correias de um motor a combustão para automóvel, que aciona simultaneamente as polias da bomba de água e do alternador: d1 = 120 mm [Motor]; d2 = 90 mm [Bomba D`água]; d3 = 80 mm [Alternador]. A velocidade econômica do motor ocorre a rotação de n1 = 2800 rpm.
  • 28. FIGURA 18 FONTE: MELCONIAN. SARKIS: Elementos de Máquinas – 9 Ed. Revisada. Editora Érica. Ano 2009. Nessa condição pode-se determinar: Polia 1 (Motor). a) Velocidade angular (ω1) 𝜔1 = 𝜋𝑛1 30 𝜔1 = 𝜋2800 30 𝜔1 = 93,33𝜋 𝑟𝑎𝑑/𝑠 𝑜𝑢 293,2 𝑟𝑎𝑑/𝑠 b) Frequência (f1) 𝑓1 = 𝜔1 2𝜋 𝑓1 = 93,33𝜋 2𝜋 𝑓1 = 46,665 𝐻𝑧 Polia 2 (Bomba D`água). c) Velocidade angular (ω2) 𝜔2 = 𝑑1 𝜔1 𝑑2 𝜔2 = 120𝑥93,33𝜋 90 𝜔2 = 124,44𝜋 𝑟𝑎𝑑/𝑠 d) Frequência (f2)
  • 29. 𝑓2 = 𝜔2 2𝜋 𝑓2 = 124,44𝜋 2𝜋 𝑓2 = 62,22 𝐻𝑧 e) Rotação (n2) 𝑛2 = 60𝑓2 𝑛2 = 60𝑥62,22 𝑛2 = 3733,2 𝑟𝑝𝑚 Polia 3 (Alternador). f) Velocidade angular (ω3) 𝜔2 = 𝑑1 𝜔1 𝑑3 𝜔2 = 120𝑥93,33𝜋 80 𝜔2 = 140𝜋 𝑟𝑎𝑑/𝑠 g) Frequência (f3) 𝑓2 = 𝜔3 2𝜋 𝑓2 = 140𝜋 2𝜋 𝑓2 = 70 𝐻𝑧 h) Rotação (n3) 𝑛2 = 60𝑓3 𝑛2 = 60𝑥70 𝑛2 = 4200 𝑟𝑝𝑚 Transmissão i) Velocidade periférica (𝒱p) 𝒱p = 𝜔1. 𝑟1 𝒱p = 93,33𝜋. 0,06 𝒱p ≅ 17,59 𝑚/𝑠 j) Relação de transmissão (Motor/Polia 1) (i1) 𝑖1 = 𝑑1 𝑑2
  • 30. 𝑖1 = 120 90 𝑖1 = 1,33 k) Relação de transmissão (Polia 2/Polia 3) (i3) 𝑖3 = 𝑑2 𝑑3 𝑖3 = 120 80 𝑖3 = 1,5 EXERCICIO 5.1 (Eric) – Uma transmissão por correias de um automóvel possui as seguintes características: Polia 1 motor 𝑑1 = 160 𝑚𝑚 Polia 2 bomba d’ água 𝑑2 = 120 𝑚𝑚 Polia 3 alternador 𝑑3 = 110 𝑚𝑚 FIGURA 19 FONTE: Autor Eric Luiz Caetano. Para a rotação constante de 3000 rpm do motor, determine: a) Velocidade angular da polia 1 (𝜔1) 𝜔1 = 𝜋𝑛1 30 𝜔1 = 𝜋 . 3000 30 = 100𝜋 𝑟𝑎𝑑 𝑠⁄ 𝑜𝑢 314,15 𝑟𝑎𝑑/𝑠 b) Frequência da polia 1 (𝑓1) 𝑓1 = 𝜔1 2𝜋 𝑓1 = 100𝜋 2𝜋 = 50 𝐻𝑧
  • 31. c) Velocidade angular da polia 2 (𝜔2) 𝜔2 = 𝜔1 𝑑1 𝑑2 𝜔2 = 100𝜋 . 160 120 = 133,3𝜋 𝑟𝑎𝑑 𝑠⁄ 𝑜𝑢 418,8 𝑟𝑎𝑑/𝑠 d) Frequência da polia 2 (𝑓2) 𝑓2 = 𝜔2 2𝜋 𝑓2 = 133,3𝜋 2𝜋 = 66,5 𝐻𝑧 e) Rotação da polia 2 (𝑛2) 𝑛2 = 60𝑓2 𝑛2 = 60 . 66,5 = 3990 𝑟𝑝𝑚 f) Velocidade angular da polia 3 (𝜔3) 𝜔3 = 𝜔1 𝑑1 𝑑3 𝜔3 = 100𝜋 . 160 110 = 145,4𝜋 𝑟𝑎𝑑 𝑠⁄ 𝑜𝑢 456,9 𝑟𝑎𝑑/𝑠 g) Frequência da polia 3 (𝑓3) 𝑓3 = 𝜔3 2𝜋 𝑓3 = 145,4𝜋 2𝜋 = 72,7 𝐻𝑧 h) Rotação da polia 3 (𝑛3) 𝑛3 = 60𝑓3 𝑛3 = 60 . 72,7 = 4362 𝑟𝑝𝑚 i) Velocidade Periférica (Vp) 𝑉𝑝 = 𝜔1. 𝑟1 𝑉𝑝 = 100𝜋 . 0,08 = 8𝜋 𝑚 𝑠⁄ 𝑜𝑢 25,13 𝑚/𝑠 j) Relação de Transmissão (𝑖1) 𝑖1 = 𝑑1 𝑑2
  • 32. 𝑖1 = 160 120 = 1,33 𝑜𝑢 33% k) Relação de Transmissão (𝑖2) 𝑖2 = 𝑑1 𝑑3 𝑖2 = 160 110 = 1,45 𝑜𝑢 45% l) Relação de Transmissão (𝑖3) 𝑖3 = 𝑑2 𝑑3 𝑖3 = 120 110 = 1,09 𝑜𝑢 9% EXERCICIO 5.2 (Felipe) – Uma transmissão por correias de um motor. FIGURA 20 FONTE: MELCONIAN. SARKIS: Elementos de Máquinas – 9 Ed. Revisada. Editora Érica. Ano 2009. d1: 150 mm (motor) d2: 100 mm (Bomba d`água) d3: 90 mm (alternador) Sabe-se que a velocidade econômica do motor ocorre a uma rotação de n= 3000 rpm. Nessa condição podemos determinar: a) Velocidade angular da polia 1 (𝜔1): 𝜔1 = 𝜋. 𝑛1 30 𝜔1 = 𝜋 . 3000 30 = 100𝜋 𝑟𝑎𝑑 𝑠⁄
  • 33. b) Frequência da polia 1 (𝑓1): 𝑓1 = 𝜔1 2𝜋 𝑓1 = 100𝜋 2𝜋 = 50 𝐻𝑧 c) Velocidade angular da polia 2 (𝜔2): 𝜔2 = 𝜔1 𝑑1 𝑑2 𝜔2 = 150 . 100𝜋 100 𝜔2 = 150𝜋 𝑟𝑎𝑑/𝑠 d) Frequência da polia 2 (𝑓2): 𝑓2 = 𝜔2 2𝜋 𝑓2 = 150𝜋 2𝜋 = 75 𝐻𝑧 e) Rotação da polia 2 (𝑛2): 𝑛2 = 60𝑓2 𝑛2 = 60 .75 = 4500 𝑟𝑝𝑚 f) Velocidade angular da polia 3 (𝜔3): 𝜔3 = 𝜔1 𝑑1 𝑑3 𝜔3 = 100𝜋 . 150 90 = 166,66𝜋 𝑟𝑎𝑑 𝑠⁄ g) Frequência na polia 3 (𝑓3): 𝑓3 = 𝜔3 2𝜋 𝑓3 = 166,66𝜋 2𝜋 = 83,33 𝐻𝑧 h) Rotação da polia 3 (𝑛3): 𝑛3 = 60𝑓3 𝑛3 = 60 . 83,33 = 4999,8 𝑟𝑝𝑚
  • 34. i) Velocidade periférica (𝑉𝑝): 𝑉𝑝 = 100𝜋. 0,075 = 7,5𝜋 𝑚/𝑠 j) Relação de Transmissão (i1): 𝑖1 = 𝑑1 𝑑2 𝑖1 = 150 100 = 1,5 k) Relação de Transmissão (𝑖2) 𝑖2 = 𝑑1 𝑑3 𝑖2 = 150 90 = 1,667 l) Relação de Transmissão (𝑖3) 𝑖3 = 𝑑2 𝑑3 𝑖3 = 100 90 = 1,111 EXERCÍCIO 5.3 (Helena) - As polias de um motor à combustão são acionadas simultaneamente. Polia 1 (motor) – d1=100mm Polia 2 (bomba d’agua) – d2=80mm Polia 3 (alternador) – d3=60mm O motor trabalha numa rotação n=2000rpm. FIGURA 21
  • 35. FONTE: MELCONIAN. SARKIS: Elementos de Máquinas – 9 Ed. Revisada. Editora Érica. Ano 2009. Determine: a) Velocidade angular na polia 1 (𝜔1) 𝜔1 = 2000𝜋 30 𝜔1 = 66,67𝜋 𝑟𝑎𝑑/𝑠 b) Frequência na polia 1 (𝑓1) 𝑓1 = 1 𝑇 𝑓1 = 𝜔 2𝜋 𝑓1 = 66,67𝜋 2𝜋 𝑓1 = 33,3 𝐻𝑧 c) Velocidade angular na polia 2 (𝜔2) 𝜔2 = 𝜔1 𝑑1 𝑑2 𝜔2 = 66,67𝜋 . 100 80 𝜔2 = 83,34𝜋 𝑟𝑎𝑑/𝑠 d) Frequência na polia 2 (𝑓2) 𝑓2 = 1 𝑇 𝑓2 = 𝜔2 2𝜋 𝑓2 = 83,34𝜋 2𝜋 𝑓2 = 41,67 𝐻𝑧 e) Rotação na Polia 2 (𝑛2) 𝑛2 = 60𝑓2 𝑛2 = 60 . 41,47 𝑛2 = 2500,125 𝑟𝑝𝑚 f) Velocidade angular na polia 3 (𝜔3) 𝜔3 = 𝜔2 𝑑2 𝑑3 𝜔3 = 83,34𝜋 . 80 60
  • 36. 𝜔3 = 111,12𝜋 𝑟𝑎𝑑/𝑠 g) Frequência na polia 3 (𝑓3) 𝑓3 = 1 𝑇 𝑓3 = 𝜔3 2𝜋 𝑓3 = 111,12𝜋 2𝜋 𝑓3 = 55,56 𝐻𝑧 h) Rotação na Polia 3 (𝑛3) 𝑛3 = 60𝑓3 𝑛3 = 60 . 55,56 𝑛3 = 3333,6 𝑟𝑝𝑚 i) Velocidade Periférica (Vp) 𝑉𝑝 = 𝜔1. 𝑟1 𝑉𝑝 = 66,67𝜋. 0,5 𝑉𝑝 = 10,47 𝑚/𝑠 j) Relação de Transmissão (𝐼1) 𝐼1 = 𝑑1 𝑑2 𝐼1 = 100 80 𝐼1 = 1,25 k) Relação de Transmissão (𝐼2) 𝐼2 = 𝑑1 𝑑3 𝐼2 = 100 60 𝐼2 = 1,67 EXERCÍCIO 5.4 (Júlio) – Um sistema de transmissão por correias de uma determinada máquina movida por um motor elétrico chavetado a uma polia, move simultaneamente duas outras polias de diâmetros d3=80mm; d2=100mm, com rotações n1=2700 e n2=8100 rpm. Determine:
  • 37. FIGURA 22 FONTE: MELCONIAN. SARKIS: Elementos de Máquinas – 9 Ed. Revisada. Editora Érica. Ano 2009. Polia 1 (Motor): a) Diâmetro Polia 1 (Motor) [d1] 𝑑1 = 𝑑2. 𝑛2 𝑛1 𝑑1 = 100.8100 2700 𝑑1 = 300𝑚𝑚 b) Velocidade angular (ω1) 𝜔1 = 𝜋𝑛1 30 𝜔1 = 𝜋. 2700 30 𝜔1 = 90𝜋 𝑟𝑎𝑑/𝑠 c) Frequência (f1) 𝑓1 = 𝜔1 2𝜋 𝑓1 = 90𝜋 2𝜋 𝑓1 = 45 𝐻𝑧 Polia 2: d) Velocidade angular (ω2) 𝜔2 = 𝑑1 𝜔1 𝑑2
  • 38. 𝜔2 = 300.90𝜋 100 𝜔2 = 270𝜋 𝑟𝑎𝑑/𝑠 e) Frequência (f2) 𝑓2 = 𝜔2 2𝜋 𝑓2 = 270𝜋 2𝜋 𝑓2 = 135 𝐻𝑧 f) Rotação (n2) 𝑛2 = 60𝑓2 𝑛2 = 60.135 𝑛2 = 8100 𝑟𝑝𝑚 Polia 3: g) Velocidade angular (ω3) 𝜔3 = 𝑑1 𝜔1 𝑑3 𝜔3 = 300.90𝜋 80 𝜔3 = 337,5𝜋 𝑟𝑎𝑑/𝑠 h) Frequência (f3) 𝑓3 = 𝜔3 2𝜋 𝑓3 = 337,5𝜋 2𝜋 𝑓3 = 168,75 𝐻𝑧 i) Rotação (n3) 𝑛3 = 60𝑓3 𝑛3 = 60.168,75 𝑛3 = 10125 𝑟𝑝𝑚 Características de transmissão: j) Velocidade periférica (𝒱p) 𝒱p = 𝜔1. 𝑟1 𝒱p = 90𝜋. 0,15
  • 39. 𝒱p ≅ 42,41 𝑚/𝑠 k) Relação de transmissão (Motor/Polia 1) (i1) 𝑖1 = 𝑑1 𝑑2 𝑖1 = 300 100 𝑖1 = 3 l) Relação de transmissão (Polia 2/Polia 3) (i3) 𝑖3 = 𝑑2 𝑑3 𝑖3 = 100 80 𝑖3 = 1,25 TORÇÃO SIMPLES EXERCÍCIO 06 – Determinar torque de aperto na chave que movimenta as castanhas da placa do torno. A carga aplicada nas extremidades da haste é F = 80N. O comprimento da haste é L = 200mm. FIGURA 23 FONTE: MELCONIAN. SARKIS: Elementos de Máquinas – 9 Ed. Revisada. Editora Érica. Ano 2009. 𝑀 𝑇 = 2. 𝐹. 𝑆 𝑀 𝑇 = 2𝑥80𝑥100 𝑀 𝑇 = 16000𝑁. 𝑚𝑚 𝑀 𝑇 = 16𝑁. 𝑚 EXERCÍCIO 6.1 (Eric) – Para se ter um torque correto no parafuso do volante de um motor 1.8 AP é necessário aproximadamente 80 Nm. Com uma chave com hastes de 25 cm, determine a força necessária que se deve aplicar nas duas extremidades da chave. 𝑀𝑡 = 2 . 𝐹 . 𝑆 
  • 40. 80 = 2 . 𝐹 . 0,25 𝐹 = 80 0,5 = 160 𝑁 EXERCICIO 6.2 (Felipe) – Determine o torque de aperto na chave que realiza a abertura e fechamento do parafuso por onde é retirado o óleo de uma motocicleta de uso urbano comum. Sabendo que a carga aplicada nas extremidades da haste é 50 N, e o comprimento da haste é de L=280 mm. 𝑀 𝑇 = 2 . 𝐹𝑠 𝑀 𝑇 = 2 . 50 . 140 𝑀 𝑇 = 14000 𝑁𝑚 Ou 𝑀 𝑇 = 14 𝑁𝑚 EXERCÍCIO 6.3 (Helena) - Um mecânico precisa fazer a manutenção de um motor. Para abri-lo ele precisará aplicar uma força de 30N numa chave de 150mm de comprimento. Determine o torque aplicado pelo mecânico: 𝑀 𝑇 = 2 . 𝐹 . 𝑆  𝑀 𝑇 = 2 . 30 . 0,15 𝑀 𝑇 = 9 𝑁𝑚 EXERCÍCIO 6.4 (Júlio) – A figura representa a força aplicada na vertical, sobre uma chave de boca, por um motorista de caminhão tentando desatarraxar uma das porcas que fixa uma roda. FIGURA 24 FONTE: http://pt.slideshare.net/CentroApoio/exercequilibrio-corpo-rigido. O ponto de aplicação da força dista 150mm do centro da porca e o módulo da força máxima aplicada é F = 400N. Nesta situação, suponha que o motorista está próximo de conseguir desatarraxar a porca. Em seguida, o motorista acopla uma extensão à chave de boca, de forma que o novo ponto de aplicação da força 750mm do centro da porca. Calcule o novo valor do módulo da força F’, em Newtons, necessária para que o motorista novamente esteja próximo de desatarraxar a porca.
  • 41. 𝑀 𝑇1 = 2. 𝐹1. 𝑆1 𝑀 𝑇1 = 2𝑥400𝑥150 𝑀 𝑇1 120000𝑁𝑚𝑚 𝑀 𝑇2 = 2. 𝐹2. 𝑆2 𝑀 𝑇2 = 2𝑥𝐹2 𝑥750 𝑀 𝑇2 = 1500𝐹2 𝑀 𝑇2 = 𝑀 𝑇1 1500𝐹2 = 120000 𝐹2 = 120000 1500 𝐹2 = 80𝑁 EXERCÍCIO 07 – Determinar torque (MT) no parafuso da roda do automóvel. A carga aplicada pelo operador em cada braço da chave é F = 120N. O comprimento dos braços é L = 200 mm. FIGURA 25 FONTE: MELCONIAN. SARKIS: Elementos de Máquinas – 9 Ed. Revisada. Editora Érica. Ano 2009. 𝑀 𝑇 = 2. 𝐹. 𝑆 𝑀 𝑇 = 2𝑥120𝑥100 𝑀 𝑇 = 48000 𝑁. 𝑚𝑚 𝑀 𝑇 = 16 𝑁. 𝑚 EXERCÍCIO 7.1 (Eric) – O manual de um certo veículo determina que o torque ideal para os parafusos da roda é de 100 nm. A chave de roda que estava no conjunto do veículo tem braços de 30 cm. Determine a força necessária que deve-se aplicar nas duas extremidades da chave.
  • 42. FIGURA 26 FONTE: Autor Eric Luiz Caetano. 𝑀 𝑇 = 2 . 𝐹 . 𝑆 100 = 2 . 𝐹 . 0,3 𝐹 = 100 0,6 = 166,6 𝑁 EXERCICIO 7.2 (Felipe) – Dada à figura, determine o torque de aperto (𝑀 𝑇) no parafuso no trilho de um elevador. A carga aplicada pelo operador em cada braço da chave é de F= 180 N, e o comprimento dos braços da chave são de L= 210 mm. FIGURA 27 FONTE: Autor Felipe Toledo. 𝑀 𝑇 = 2 . F . L 𝑀 𝑇 = 2 . 180.210 𝑀 𝑇 = 75600 Nm ou 𝑀 𝑇 = 75,6 Nm EXERCÍCIO 7.3 (Helena) - Para trocar o pneu de um carro é necessário levantá-lo com um “macaco”. Dado o torque de 20Nm e o comprimento L= 200mm da manivela de acionamento do levantador, determine a força aplicada na operação:
  • 43. 𝑀 𝑇 = 2 . 𝐹 . 𝐿  20 = 2 . 𝐹 . 0,20 𝐹 = 20 0,4 = 50 𝑁 EXERCÍCIO 7.4 (Júlio) – Para a figura abaixo calcule o torque provocado pela manivela de comprimento L = 250mm, em relação ao centro do eixo e considerando a carga de acionamento igual a F = 600N. FIGURA 28 FONTE: http://formacaopiloto.blogspot.com.br/2014_06_01_archive.html. 𝑀 𝑇 = 2. 𝐹. 𝑆 𝑀 𝑇 = 2𝑥600𝑥250 𝑀 𝑇 = 300000 𝑁. 𝑚𝑚 𝑀 𝑇 = 300 𝑁. 𝑚 TORQUE NAS TRANSMISSÕES EXERCÍCIO 08 – A transmissão por correia é composta pela polia motora (1) que possui diâmetro d1 = 100mm e a polia movida (2) que possui diâmetro d2 = 240mm. A transmissão é acionada por uma força tangencial FT = 600N. Determinar: FIGURA 29 FONTE: MELCONIAN. SARKIS: Elementos de Máquinas – 9 Ed. Revisada. Editora Érica. Ano 2009. a) Torque na Polia (1) 𝑟1 = 𝑑1 2
  • 44. 𝑟1 = 100 2 𝑟1 = 50𝑚𝑚 𝑟1 = 0.05𝑚 𝑀 𝑇 = 𝐹 𝑇. 𝑟1 𝑀 𝑇 = 600𝑥0,05 𝑀 𝑇 = 30𝑁𝑚 b) Torque na Polia (2) 𝑟1 = 𝑑2 2 𝑟1 = 240 2 𝑟1 = 120𝑚𝑚 𝑟1 = 0.12𝑚 𝑀 𝑇 = 𝐹 𝑇. 𝑟1 𝑀 𝑇 = 600𝑥0,12 𝑀 𝑇 = 72𝑁𝑚 EXERCÍCIO 8.1 (Eric) – Uma transmissão por correia ampliadora de velocidade é movimentada por uma força inicial tangencial de FT = 500 N. A polia motora dessa transmissão possui um diâmetro de 150 mm e a polia movida possui um diâmetro de 100 mm. Determine o torque na polia motora e na polia movida. FIGURA 30 FONTE: Autor Júlio César Droszczak 𝑀𝑡1 = 𝐹𝑡. 𝑟1 𝑀𝑡1 = 500 . ( 0,15 2 ) 𝑀𝑡1 = 37,5 𝑁𝑚 𝑀𝑡2 = 𝐹𝑡. 𝑟2 𝑀𝑡2 = 500 . ( 0,1 2 )
  • 45. 𝑀𝑡2 = 25 𝑁𝑚 EXERCICIO 8.2 (Felipe) – A transmissão por correias, representada na figura, é composta pela polia motora 1 que possui diâmetro de d1= 230 mm e a polia movida 2 possui diâmetro d2= 500 mm. A transmissão será acionada por uma força tangencial 𝐹 𝑇=850 N. FIGURA 31 FONTE: MELCONIAN. SARKIS: Elementos de Máquinas – 9 Ed. Revisada. Editora Érica. Ano 2009. COM ALTERAÇÃO DO AUTOR. Determine o torque na polia 1: Determine o torque na polia 2: RESOLUÇÃO: a) Torque na polia 1: Raio da polia 1: 𝑟1 = 𝑑1 2 = 230 2 = 115 𝑚𝑚 𝑟1 = 115 𝑚𝑚 𝑜𝑢 𝑟1 = 0,115𝑚 Torque na polia: 𝑀 𝑇1 = 𝐹 𝑇. 𝑟1 𝑀 𝑇1 = 850𝑁 . 0,115𝑚 𝑀 𝑇1 = 97,75 𝑁𝑚 b) Torque na polia 2: Raio da polia 2: 𝑟2 = 𝑑2 2 = 500 2 = 250 𝑚𝑚 𝑟2 = 250𝑚𝑚 𝑜𝑢 𝑟2 = 0,25𝑚 Torque na polia
  • 46. 𝑀 𝑇2 = 𝐹 𝑇. 𝑟2 𝑀 𝑇2 = 850 .0,25 = 212,5 𝑁𝑚 EXERCÍCIO 8.3 (Helena) - Uma transmissão por correia é movimentada por uma força inicial tangencial de FT = 300N. A polia motora dessa transmissão possui um diâmetro de 250 mm e a polia movida trabalha com um torque de 75Nm. Determine: a) Torque na polia motora 𝑀 𝑇1 = 𝐹 𝑇. 𝑟1 𝑀 𝑇1 = 300 . ( 0,25 2 ) 𝑀 𝑇1 = 37,5 𝑁𝑚 b) Diâmetro da polia movida. 𝑀 𝑇1 = 𝐹 𝑇 . ( 𝐷2 2 ) 75 = 300 . ( 𝐷2 2 ) 𝐷2 = 0,50𝑚 = 500𝑚𝑚 EXERCÍCIO 8.4 (Júlio) – A transmissão por correia é composta pela polia motora (A) que possui diâmetro d1 = 80mm e a polia movida (B) que possui diâmetro d2 = 210mm. A transmissão é acionada por uma força tangencial FT = 730N. Determinar: FIGURA 32 FONTE: http://www.vdl.ufc.br/solar/aula_link/lfis/semestre01/Fisica_I/Aula_03/02.html a) Torque na Polia (A) 𝑟1 = 𝑑1 2 𝑟1 = 80 2 𝑟1 = 40𝑚𝑚
  • 47. 𝑟1 = 0.04𝑚 𝑀 𝑇 = 𝐹 𝑇. 𝑟1 𝑀 𝑇 = 730𝑥0,04 𝑀 𝑇 = 29,2𝑁𝑚 b) Torque na Polia (B) 𝑟1 = 𝑑2 2 𝑟1 = 210 2 𝑟1 = 105𝑚𝑚 𝑟1 = 0.105𝑚 𝑀 𝑇 = 𝐹 𝑇. 𝑟1 𝑀 𝑇 = 730𝑥0,105 𝑀 𝑇 = 76,65𝑁𝑚 POTÊNCIA EXERCÍCIO 09 – O elevador projetado para transportar carga máxima CMÁX. = 7000N (10 pessoas). O peso do elevador é PE = 1KN e o contrapeso possui a mesma carga CP = 1KN. Determine a potência do motor M para que o elevador se desloque com velocidade constante V = 1m/s. FIGURA 33 FONTE: MELCONIAN. SARKIS: Elementos de Máquinas – 9 Ed. Revisada. Editora Érica. Ano 2009. Resolução:
  • 48. O peso do elevador é compensado pelo contrapeso, eliminando o seu efeito. Portanto. Para dimensionar a potência do motor, a carga a ser utilizada é CMÁX. = 7000N Potência do motor (PMOTOR) 𝑃 𝑀𝑂𝑇𝑂𝑅 = 𝐹. 𝑉 𝑃 𝑀𝑂𝑇𝑂𝑅 = 7000𝑥1 𝑃 𝑀𝑂𝑇𝑂𝑅 = 7000𝑊 𝑃𝐶𝑉 = 𝑃 𝑊 735,5 𝑃𝐶𝑉 = 7000 735,5 𝑃𝐶𝑉 ≅ 9,5 𝐶𝑉 EXERCÍCIO 9.1 (Eric) – Um elevador de veículos tem a capacidade máxima de 4800 N (cerca de 6 pessoas com massas de 80 kg). O elevador tem sem peso irrelevante, pois há um contrapeso que possui o mesmo peso do elevador. O elevador possui uma velocidade constante de 5 m/s. Determine a potência do motor que movimenta este elevador. FIGURA 34 FONTE: http://www.gpmotorsbrasil.com.br/info1.html (Força de tração no cabo é igual à força peso da capacidade máxima: 4800N) 𝑃 𝑚𝑜𝑡𝑜𝑟 = 𝐹𝑐𝑎𝑏𝑜 . 𝑉 𝑃 𝑚𝑜𝑡𝑜𝑟 = 7000 .5 𝑃 𝑚𝑜𝑡𝑜𝑟 = 35000 𝑊 𝑜𝑢 35 𝐾𝑊 𝑃 𝑚𝑜𝑡𝑜𝑟 = 35000 735,5 = 47,28 𝐶𝑣
  • 49. EXERCICIO 9.2 (Felipe) – Em um elevador comum, cuja seu projeto tem como especificação de carga máxima 560 kg (70 kg/por pessoa). Sabendo que o contra peso e a cabina possuem a mesma carga de 1 kN. Determine a potência do motor M para que o elevador de desloque com velocidade constante de V= 2,5 m/s. Determine a potencia do motor. FIGURA 35 FONTE: http://seguranca-na-construcao.dashofer.pt/?s=modulos&v=capitulo&c=7655 𝐾𝑔 𝑝𝑎𝑟𝑎 𝑁 = 𝐾𝑔. 100 = 560. 10 = 5600 𝑁 𝑃 𝑚𝑜𝑡𝑜𝑟 = 𝐹𝑐𝑎𝑏𝑜. 𝑉 𝑃 𝑚𝑜𝑡𝑜𝑟 = 5600 .2,5 𝑃 𝑚𝑜𝑡𝑜𝑟 = 14000 w 𝑊 𝑝𝑎𝑟𝑎 𝐶𝑉 = 𝑃( 𝑊) 735,5 = 14000 735,5 = 19 𝐶𝑉 EXERCÍCIO 9.3 (Helena) - Um elevador de carga foi projetado para transportar carga máxima CMÁX. = 10kN. O peso do elevador é PE = 1,4KN e o contrapeso possui a mesma carga CP = 1,4KN. Determine a potência do motor M para que o elevador se desloque com velocidade constante V = 0,8m/s.
  • 50. FIGURA 36 FONTE: http://www.pauluzzi.com.br/alvenaria.php?PHPSESSID=ccd0dd0c90aa9901b2a2e49d3182897c 𝑃 𝑀𝑂𝑇𝑂𝑅 = 𝐹. 𝑉 𝑃 𝑀𝑂𝑇𝑂𝑅 = 10000𝑥0,8 𝑃 𝑀𝑂𝑇𝑂𝑅 = 8000𝑊 𝑃𝐶𝑉 = 𝑃 𝑊 735,5 𝑃𝐶𝑉 = 8000 735,5 𝑃𝐶𝑉 ≅ 10,88 𝐶𝑉 EXERCÍCIO 9.4 (Júlio) – Cada um dos dois motores a jato de um avião Boeing 767 desenvolve uma propulsão (força que acelera o avião) igual a F = 197000N. Quando o avião esta voando a V = 250m/s, qual a potência instantânea que cada motor desenvolve? Em W e CV. FIGURA 37 FONTE: http://pt.wikipedia.org/wiki/Turbina_aeron%C3%A1utica 𝑃 𝑀𝑂𝑇𝑂𝑅 = 𝐹. 𝑉 𝑃 𝑀𝑂𝑇𝑂𝑅 = 197000.250
  • 51. 𝑃 𝑀𝑂𝑇𝑂𝑅 = 49250000𝑊 𝑃𝐶𝑉 = 𝑃 𝑊 735,5 𝑃𝐶𝑉 = 49250000 735,5 𝑃𝐶𝑉 ≅ 66961,25 𝐶𝑉 EXERCÍCIO 10 – Um servente de pedreiro erguendo uma lata de concreto com peso PC = 200N. A corda e a polia são ideais. A altura da laje é h = 8m, o tempo de subida é t = 20s. Determine a potência útil do trabalho do operador. FIGURA 38 FONTE: MELCONIAN. SARKIS: Elementos de Máquinas – 9 Ed. Revisada. Editora Érica. Ano 2009. R = Como a carga esta sendo elevada com movimento uniforme, conclui-se que a aceleração do movimento é nula, portanto: 𝐹 𝑂𝑃𝐸𝑅𝐴𝐷𝑂𝑅. = 𝐹𝐶 = 200𝑁 𝑉𝑆𝑈𝐵𝐼𝐷𝐴 = ℎ 𝑡 𝑉𝑆𝑈𝐵𝐼𝐷𝐴 = 8 20 𝑉𝑆𝑈𝐵𝐼𝐷𝐴 = 0,04 𝑚/𝑠 Potência útil do operador. 𝑃 = 𝐹 𝑂𝑃.. 𝑉𝑆𝑈𝐵. 𝑃 = 200𝑥0,4 𝑃 = 80𝑊 EXERCÍCIO 10.1 (Eric) – Um motor de 5 KW está posicionado no quinto andar de um prédio e trabalha para puxar uma carga P = 750N através de uma corda e uma polia. Determine a velocidade de subida dessa carga.
  • 52. FIGURA 39 FONTE: Autor Eric Luiz Caetano. 𝑃 𝑚𝑜𝑡𝑜𝑟 = 𝐹𝑐𝑎𝑟𝑔𝑎 . 𝑉 5000 = 750 . 𝑉 𝑉 = 5000 750 = 6,66 𝑚/𝑠 EXERCICIO 10.2 (Felipe) – Em um canteiro de obra um servente de pedreiro ergue um balde de cimento com peso 𝑃𝑐 = 10 𝐾𝑔. Considerando que tanto a polia quanto a corda serão consideradas como ideais. A altura em que será levantado é de h= 5 metros, e sua velocidade de subida de 0,556 m/s. Determine qual será o tempo de subida, e qual será a potência útil do trabalho do operador. FIGURA 40 FONTE: MELCONIAN. SARKIS: Elementos de Máquinas – 9 Ed. Revisada. Editora Érica. Ano 2009.COM ALTERAÇÃO DO AUTOR. 𝐾𝑔 𝑝𝑎𝑟𝑎 𝑁 = 10 . 10 = 100𝑁 𝐹𝑜 𝑝 = 𝐹𝑜𝑟ç𝑎 𝑎𝑝𝑙𝑖𝑐𝑎𝑑𝑎 𝑝𝑒𝑙𝑜 𝑜𝑝𝑒𝑟𝑎𝑑𝑜𝑟 𝑃𝑐 = 𝑃𝑒𝑠𝑜 𝑑𝑎 𝑏𝑎𝑙𝑑𝑒 𝑑𝑒 𝑐𝑖𝑚𝑒𝑛𝑡𝑜 𝐹𝑜 𝑝 = 𝐹𝑐 = 100𝑁 Tempo total de subida: (𝑡 𝑆) 𝑡 𝑠 = ℎ 𝑉𝑠 = 5 𝑚 0,556 𝑚/𝑠 = 9 𝑠
  • 53. Potência útil do operador: 𝑃 = 𝐹𝑜 𝑝 . 𝑉𝑠 𝑃 = 100 𝑁 .0,556 𝑚 𝑠 𝑃 = 55,56 𝑊 EXERCÍCIO 10.3 (Helena) - Uma pessoa fazendo uma mudança para o segundo andar de um prédio cujas escadas são muito estreitas, precisa puxar o sofá de peso P = 5000N pela parte de fora do prédio. Dados altura até a janela h=12m e o tempo para colocar para dentro de 2min, determine a potência útil do trabalho aplicado pela pessoa. 𝑉𝑆 = ℎ 𝑡 𝑉𝑆 = 12 120 𝑉𝑆 = 0,1 𝑚/𝑠 Potencia útil 𝑃 = 𝐹𝑃𝐸𝑆... 𝑉𝑆𝑈𝐵. 𝑃 = 500𝑥0,1 𝑃 = 50𝑊 EXERCÍCIO 10.4 (Júlio) – Uma pessoa erguendo um balde de água de um andar a outro com um peso de P = 300N. A altura do andar é de h = 6m, o tempo necessário para erguer o balde desde o andar de baixo até o andar de cima é de t = 27s. Determine a potência útil de trabalho da pessoa. Considerando a corda e a polia como sendo ideais. FIGURA 41 FONTE:http://crv.educacao.mg.gov.br/sistema_crv/index.aspx?ID_OBJETO=58360&tipo=ob &cp=780031&cb=&n1=&n2=M%EF%BF%BDdulos%20Did%EF%BF%BDticos&n3=Ensino%20M%EF %BF%BDdio&n4=F%EF%BF%BDsica&b=s. 𝐹𝑃𝐸𝑆𝑆𝑂𝐴. = 𝐹𝐶 = 300𝑁
  • 54. 𝑉𝑆𝑈𝐵𝐼𝐷𝐴 = ℎ 𝑡 𝑉𝑆𝑈𝐵𝐼𝐷𝐴 = 8 20 𝑉𝑆𝑈𝐵𝐼𝐷𝐴 = 0,22 𝑚/𝑠 Potência útil do operador. 𝑃 = 𝐹𝑃𝐸𝑆... 𝑉𝑆𝑈𝐵. 𝑃 = 300𝑥0,22 𝑃 = 66,660𝑊 EXERCÍCIO 11 – Um motor elétrico com potência P = 0,25W esta erguendo uma lata de concreto com peso PC = 200N. A corda e a polia são ideais. A altura da laje é h = 8m. Determine: a) Velocidade de subida da lata de concreto (VSUB.). 𝐹 𝑀𝑂𝑇𝑂𝑅. = 𝐹𝐶 = 200𝑁 𝑃 = 250𝑊 𝑃 = 𝐹 𝑀𝑂𝑇.. 𝑉𝑆𝑈𝐵. 𝑉𝑆𝑈𝐵. = 𝑃 𝐹 𝑀𝑂𝑇. 𝑉𝑆𝑈𝐵. = 250 200 𝑉𝑆𝑈𝐵. = 1,25𝑚/𝑠 b) Tempo de subida da lata (tSUB.). 𝑉𝑆𝑈𝐵. = ℎ 𝑡 𝑆𝑈𝐵. 𝑡 𝑆𝑈𝐵. = ℎ 𝑉𝑆𝑈𝐵. 𝑡 𝑆𝑈𝐵. = 8 1,25 𝑡 𝑆𝑈𝐵. = 6,4𝑠 EXERCÍCIO 11.1 (Eric) – Para empurrar uma caixa, duas pessoas aplicam uma força de 800N de modo que a caixa se move com velocidade constante em uma distância de 10 metros no tempo de 5 segundos. Desconsiderando o atrito, determinar a velocidade em que a caixa se move e a potência útil das duas pessoas. 𝑣𝑚 = ∆𝑆 ∆𝑡
  • 55. 𝑣𝑚 = 10 5 = 2 𝑚/𝑠 𝑃ú𝑡𝑖𝑙 = 𝐹 . 𝑣𝑚 𝑃ú𝑡𝑖𝑙 = 800 .2 𝑃ú𝑡𝑖𝑙 = 1600 𝑊 EXERCICIO 11.2 (Felipe) – Seguindo a mesmo raciocínio do exercício anterior, substituiremos o servente de pedreiro por um motor elétrico com potência de 0,43 KW. Determine: a) Velocidade de subida do balde de concreto: ( 𝑉𝑠) 𝐾𝑊 𝑝𝑎𝑟𝑎 𝑊 = 𝐾𝑊 . 1000 = 0,43 . 1000 = 430 𝑊 𝐾𝑔 𝑝𝑎𝑟𝑎 𝑁 = 10 . 10 = 100𝑁 𝑉𝑠 = 𝑃 𝑚𝑜𝑡𝑜𝑟 𝐹𝑠𝑢𝑏𝑖𝑑𝑎 = 430𝑊 100𝑁 = 4,5 𝑚/𝑠 b) Tempo de subida do balde: ( 𝑡 𝑠) 𝑡 𝑠 = ℎ 𝑉𝑠 = 5 4,5 = 1,11 𝑠 EXERCÍCIO 11.3 (Helena) - Um andaime elétrico é acionado através de um motor de potencia P. Considerando a altura h = 50m, velocidade v = 0,8m/s e o peso P = 200N, determine: a) Tempo de subida do andaime 𝑉𝑆𝑈𝐵. = ℎ 𝑡 𝑆𝑈𝐵. 𝑡 𝑆𝑈𝐵. = ℎ 𝑉𝑆𝑈𝐵. 𝑡 𝑆𝑈𝐵. = 50 0,8 𝑡 𝑆𝑈𝐵. = 62,5𝑠 b) Potencia do motor 𝑃 = 𝐹 𝑀𝑂𝑇.. 𝑉𝑆𝑈𝐵. 𝐹 𝑚𝑜𝑡. = 𝑃 𝑉𝑠𝑢𝑏. 𝐹 𝑚𝑜𝑡. = 200 0,8
  • 56. 𝐹 𝑚𝑜𝑡. = 250𝑊 EXERCÍCIO 11.4 (Júlio) – (UNESP-SP) Um motor de potência útil P = 125W, funcionando como elevador, eleva a altura h = 10m, com velocidade constante, um corpo de peso igual a 50N. FIGURA 42 FONTE: http://fisicaevestibular.com.br/exe_din_15.htm. a) Velocidade de subida da lata de concreto (VSUB.). 𝐹 𝑀𝑂𝑇𝑂𝑅. = 𝐹𝐶 = 50𝑁 𝑃 = 125𝑊 𝑃 = 𝐹 𝑀𝑂𝑇.. 𝑉𝑆𝑈𝐵. 𝑉𝑆𝑈𝐵. = 𝑃 𝐹 𝑀𝑂𝑇. 𝑉𝑆𝑈𝐵. = 125 50 𝑉𝑆𝑈𝐵. = 2,5 𝑚/𝑠 b) Tempo de subida da lata (tSUB.). 𝑉𝑆𝑈𝐵. = ℎ 𝑡 𝑆𝑈𝐵. 𝑡 𝑆𝑈𝐵. = ℎ 𝑉𝑆𝑈𝐵. 𝑡 𝑆𝑈𝐵. = 10 2,5 𝑡 𝑆𝑈𝐵. = 4𝑠 EXERCÍCIO 12 – Uma pessoa empurra o carrinho de supermercado, aplicando uma carga de F = 150N, deslocando-se em um percurso de 42m no tempo de 1min. Determine a potência que movimenta o veículo.
  • 57. FIGURA 43 FONTE: MELCONIAN. SARKIS: Elementos de Máquinas – 9 Ed. Revisada. Editora Érica. Ano 2009. 𝑉𝐶𝐴𝑅. = 𝑆 𝑡 𝑉𝐶𝐴𝑅. = 42 60 𝑉𝐶𝐴𝑅. = 0,7𝑚/𝑠 𝑃 = 𝐹. 𝑉𝐶𝐴𝑅. 𝑃 = 150𝑥0,7 𝑃 = 105𝑊 EXERCÍCIO 12.1 (Eric) – Dois trabalhadores estão puxando do terceiro andar de um prédio uma caixa com várias latas de concreto. O total em peso das latas e da caixa é de 800 N. Supondo que não há perdas de potência pela corda e que a caixa sobe a uma velocidade constante de 1,2 m/s, determine a potência útil de cada trabalhador, supondo que ambos trabalham com a mesma intensidade. 𝑃 = 𝐹 2 . 𝑉𝑠 𝑃 = 800 2 . 1,2 𝑃 = 480 𝑊 EXERCICIO 12.2 (Felipe) – Um cilindro hidráulico aplica a força de F= 320 N sobre um carro de ferramentas em uma determinada fábrica, seu deslocamento durante um percurso de 70 metros no tempo de 54 segundos. Determine a velocidade deste carrinho, e qual será a potência que movimentará este carro de ferramentas. 1) Velocidade do carro de ferramentas: (𝑉𝑐) 𝑉𝑐 = 𝑠 𝑡 = 70𝑚 54𝑠 = 1,296 𝑚 𝑠 2) Potência do carro: ( 𝑃) 𝑃 = 𝐹 . 𝑉𝑐
  • 58. 𝑃 = 320 𝑁 . 1,296 𝑚 𝑠 𝑃 = 414,72 𝑊 EXERCÍCIO 12.3 (Helena) - Uma diarista precisa movimentar um móvel 2,5 metros para conseguir fazer a limpeza. Levando em consideração que ela fez isso em 15s e aplicou uma força de 100N, determine: a) Velocidade deslocamento: (𝑉𝑑) 𝑉𝑑 = 𝑠 𝑡 = 2,5 15 = 0,167𝑚/𝑠 b) Potência do movimento: ( 𝑃) 𝑃 = 𝐹 . 𝑉𝑑 𝑃 = 100 . 0,167 𝑃 = 16,7 𝑊 EXERCÍCIO 12.4 (Júlio) – Um trabalhador precisa movimentar um carrinho de mão cheio de pedras por 100m para descarregar, para isso ele aplica uma carga P = 350N, a potência que movimenta o veículo é de 270W. Quanto tempo ele levará para chegar ao local especificado para descarga? FIGURA 44 FONTE:http://crv.educacao.mg.gov.br/sistema_crv/index.aspx?ID_OBJETO=58360&tipo=ob&cp=780 031&cb=&n1=&n2=M%EF%BF%BDdulos%20Did%EF%BF%BDticos&n3=Ensino%20M%EF%BF%B Ddio&n4=F%EF%BF%BDsica&b=s 𝑃 = 𝐹. 𝑉𝐶𝐴𝑅. 𝑉𝐶𝐴𝑅. = 𝑃 𝐹 𝑉𝐶𝐴𝑅. = 270 350 𝑉𝐶𝐴𝑅. = 0,771 𝑚/𝑠 𝑉𝐶𝐴𝑅. = 𝑆 𝑡
  • 59. 𝑡 = 𝑆 𝑉𝐶𝐴𝑅. 𝑡 = 100 0,771 𝑡 = 129,7𝑠 EXERCÍCIO 13 – A transmissão por Corrêa é acionada por um motor elétrico com potência de P = 5,5KW, com rotação de n = 1720 rpm, chavetado a polia (1) do sistema. As polias possuem respectivamente os seguintes diâmetros: d1 = 120mm [Polia (1) Motora]; d2 = 300mm [Polia (2) Movida]. Desprezar as perdas. Determinar para a transmissão: FIGURA 45 FONTE: MELCONIAN. SARKIS: Elementos de Máquinas – 9 Ed. Revisada. Editora Érica. Ano 2009. a) Velocidade angular da polia (1) [ω1] 𝜔1 = 𝑛𝜋 30 𝜔1 = 1720𝜋 30 𝜔1 = 57,33𝜋 𝑟𝑎𝑑/𝑠 b) Frequência da polia (1) [f1] 𝑓1 = 𝑛1 60 𝑓1 = 1720 60 𝑓1 = 28,66 𝐻𝑧 c) Torque da polia (1) [MT1] A rotação da polia (1) é a mesma rotação do motor, pois a polia esta chavetada no eixo-árvore do motor. 𝑀 𝑇1 = 𝑃 𝜔1 𝑀 𝑇1 = 5500 57,33𝜋
  • 60. 𝑀 𝑇1 = 30,5𝑁𝑚 d) Velocidade angular da polia (2) [ω2] 𝜔2 = 𝑑1. 𝜔1 𝑑2 𝜔2 = 120𝑥53,33𝜋 300 𝜔2 = 22,93𝜋 𝑟𝑎𝑑/𝑠 e) Frequência da polia (2) [f2] 𝑓2 = 𝜔2 2𝜋 𝑓2 = 22,93𝜋 2𝜋 𝑓2 = 11,465 𝐻𝑧 f) Torque da polia (2) [MT2] 𝑀 𝑇1 = 𝑃 𝜔2 𝑀 𝑇1 = 5500 22,93𝜋 𝑀 𝑇1 = 76,3𝑁𝑚 g) Rotação da polia (2) [n2] 𝑛2 = 60𝑓2 𝑛2 = 60𝑥11,465 𝑛2 = 688 𝑟𝑝𝑚 h) Relação de transmissão [i] 𝑖 = 𝑑2 𝑑1 𝑖 = 300 120 𝑖 = 2,5 i) Velocidade periférica da transmissão [VP] A velocidade periférica da transmissão é a mesma da polia (1) com a da polia (2), portanto podemos utilizar: 𝑉𝑃 = 𝜔1. 𝑟1 = 𝜔2. 𝑟2 𝑉𝑃 = 𝜔1. 𝑟1
  • 61. 𝑉𝑃 = 57,33𝜋𝑥0,06 𝑉𝑃 = 3,44𝜋 𝑚/𝑠 𝑉𝑃 = 10,8 𝑚/𝑠 j) Força tangencial [FT] 𝐹 𝑇 = 𝑀 𝑇1 𝑟1 = 𝑀 𝑇2 𝑟2 𝐹 𝑇 = 𝑀 𝑇1 𝑟1 𝐹 𝑇 = 30,5 0,06 𝐹 𝑇 = 508,3𝑁 EXERCÍCIO 13.1 (Eric) – O pinhão de uma transmissão de uma moto é movimentado por um motor de 25 cv de potência. O pinhão possui um diâmetro de 100 mm e a coroa possui um diâmetro de 200 mm. Para uma rotação de 2400 rpm, determine: FIGURA 46 FONTE: Autor Eric Luiz Caetano. a) Velocidade angular do pinhão 𝜔1 = 𝑛 . 𝜋 30 𝜔1 = 2400 . 𝜋 30 𝜔1 = 80𝜋 𝑟𝑎𝑑 𝑠⁄ b) Frequência do pinhão 𝑓1 = 𝑛 60 𝑓1 = 2400 60 𝑓1 = 40 𝐻𝑧
  • 62. c) Torque do pinhão 𝑀𝑡 = 𝑃 𝜔1 𝑀𝑡 = 𝑃 80𝜋 𝑃 = 𝑃𝐶𝑣 .735,5 𝑃 = 25 .735,5 𝑃 = 18387,5 𝑊 𝑀𝑡 = 18387,5 80𝜋 𝑀𝑡 = 73,16 𝑁𝑚 d) Velocidade angular da coroa 𝜔2 = 𝜔1 𝑑1 𝑑2 𝜔2 = 80𝜋 . 100 200 𝜔2 = 40𝜋 e) Frequência da coroa 𝑓2 = 𝜔2 2𝜋 𝑓2 = 40𝜋 2𝜋 𝑓2 = 20 𝐻𝑧 f) Torque da coroa 𝑀𝑡 = 𝑃 𝜔2 𝑀𝑡 = 18387,5 40𝜋 𝑀𝑡 = 146,32 𝑁𝑚 g) Rotação da coroa 𝑛2 = 60 . 𝑓2 𝑛2 = 60 . 20 𝑛2 = 1200 𝑟𝑝𝑚 h) Relação de transmissão 𝑖 = 𝑑2 𝑑1
  • 63. 𝑖 = 200 100 𝑖 = 2 i) Velocidade periférica 𝑉𝑝 = 𝜔1 . 𝑟1 𝑉𝑝 = 80𝜋 . 0,05 𝑉𝑝 = 12,57 𝑚/𝑠 j) Força tangencial da transmissão 𝐹𝑡 = 𝑀𝑡1 𝑟1 𝐹𝑡 = 73,16 0,05 𝐹𝑡 = 1463,2 𝑁 EXERCICIO 13.2 (Felipe) – É dada uma transmissão por correias, representada na figura, é acionada por um motor elétrico com potência P= 4 kW com rotação n=1400 rpm, tendo em vista que a rotação 𝑛1 será a mesma do motor pois a polia encontra-se chavetada ao eixo arvore do motor. Como representada na figura a seguir: FIGURA 47 FONTE: MELCONIAN. SARKIS: Elementos de Máquinas – 9 Ed. Revisada. Editora Érica. Ano 2009. COM ALTERAÇÃO DO AUTOR. Diâmetros das polias: 𝑑1 = 100 𝑚𝑚( 𝑑𝑖â𝑚𝑒𝑡𝑟𝑜 𝑑𝑎 𝑝𝑜𝑙𝑖𝑎 1) 𝑑2 = 280 𝑚𝑚 (𝑑𝑖â𝑚𝑒𝑡𝑟𝑜 𝑑𝑎 𝑝𝑜𝑙𝑖𝑎 2) Determinar para transmissão: a) Velocidade angular da polia 1 ( 𝜔1) b) Frequência da polia 1 ( 𝑓1) c) Torque da polia 1(𝑀 𝑇1 ) d) Velocidade angular da polia 2 ( 𝜔2) e) Frequência da polia 2 ( 𝑓2)
  • 64. f) Rotação da polia 2 ( 𝑛2) g) Torque da polia 2 (𝑀 𝑇2 ) h) Relação de transmissão (i) i) Velocidade periférica da transmissão (𝑉𝑝) j) Força tangencial da transmissão( 𝐹 𝑇) Resolução: a) Velocidade angular da polia 1 ( 𝜔1) 𝜔1 = 𝑛𝜋 30 = 1400𝜋 30 = 146.6 𝑟𝑎𝑑 𝑠 𝑜𝑢 46,6𝜋 𝑟𝑎𝑑/𝑠 b) Frequência da polia 1 ( 𝑓1) 𝑓1 = 𝑛1 60 = 1400 60 = 23,33 𝐻𝑧 c) Torque da polia 1(𝑀 𝑇1 ) 𝑀 𝑇1 = 𝑃 𝑤1 = 4000 146,6 = 27,28 𝑁𝑚 d) Velocidade angular da polia 2 ( 𝜔2) 𝜔2 = 𝑑1 𝑑2 . 𝑤1 = 100.146,6 280 = 52,35 𝑟𝑎𝑑 𝑠 e) Frequência da polia 2 ( 𝑓2) 𝑓2 = 𝑤2 2𝜋 = 52,35 6,283 = 8,33 𝐻𝑧 f) Rotação da polia 2 ( 𝑛2) 𝑛2 = 60. 𝑓2 = 60 . 8,33 = 499,8 𝑟𝑝𝑚 g) Torque da polia 2 (𝑀 𝑇2 ) 𝑀 𝑇2 = 𝑃 𝑤2 = 4000𝑊 52,35𝑟𝑎𝑑 𝑠 = 76,4 𝑁𝑚 h) Relação de transmissão (i) 𝑖 = 𝑑2 𝑑1 = 280 100 = 2,8
  • 65. i) Velocidade periférica da transmissão (𝑉𝑝) 𝑉𝑝 = 𝑤1. 𝑟1 𝑜𝑢 𝑤2. 𝑟2 𝑟1 = 100 2 = 50 = 50 1000 = 0,05𝑚 𝑉𝑝 = 146,6. 0,05 = 7,33 𝑚 𝑠 j) Força tangencial da transmissão( 𝐹 𝑇) 𝐹 𝑇 = 𝑀 𝑇1 𝑟1 = 27,28 0,05 = 545,6 𝑁 EXERCÍCIO 13.3 (Helena) - Uma transmissão é acionada por um motor elétrico com potência de P = 8KW, com rotação de n = 1720 rpm, chavetado a polia motora (1) do sistema. Dados os diâmetros: d1 = 210mm; d2 = 450mm. Determine para a transmissão: a) Velocidade angular da polia (1) [ω1] 𝜔1 = 𝑛𝜋 30 𝜔1 = 1720𝜋 30 𝜔1 = 57,33𝜋 𝑟𝑎𝑑/𝑠 b) Frequência da polia (1) [f1] 𝑓1 = 𝑛1 60 𝑓1 = 1720 60 𝑓1 = 28,66 𝐻𝑧 c) Torque da polia (1) [MT1] 𝑀 𝑇1 = 𝑃 𝜔1 𝑀 𝑇1 = 8000 57,33𝜋 𝑀 𝑇1 = 44,42𝑁𝑚 d) Velocidade angular da polia (2) [ω2] 𝜔2 = 𝑑1. 𝜔1 𝑑2 𝜔2 = 210𝑥57,33𝜋 450
  • 66. 𝜔2 = 26,75𝜋 𝑟𝑎𝑑/𝑠 e) Frequência da polia (2) [f2] 𝑓2 = 𝜔2 2𝜋 𝑓2 = 26,75𝜋 2𝜋 𝑓2 = 13,377 𝐻𝑧 f) Torque da polia (2) [MT2] 𝑀 𝑇1 = 𝑃 𝜔2 𝑀 𝑇1 = 8000 26,75𝜋 𝑀 𝑇1 = 95,2𝑁𝑚 g) Rotação da polia (2) [n2] 𝑛2 = 60𝑓2 𝑛2 = 60𝑥13,377 𝑛2 = 802,62 𝑟𝑝𝑚 h) Relação de transmissão [i] 𝑖 = 𝑑2 𝑑1 𝑖 = 450 210 𝑖 = 2,14 i) Velocidade periférica da transmissão [VP] A velocidade periférica da transmissão é a mesma da polia (1) com a da polia (2), portanto podemos utilizar: 𝑉𝑃 = 𝜔1. 𝑟1 = 𝜔2. 𝑟2 𝑉𝑃 = 𝜔1. 𝑟1 𝑉𝑃 = 57,33𝜋𝑥0,105 𝑉𝑃 = 6,02𝜋 𝑚/𝑠 𝑉𝑃 = 18,9 𝑚/𝑠 j) Força tangencial [FT] 𝐹 𝑇 = 𝑀 𝑇1 𝑟1 = 𝑀 𝑇2 𝑟2
  • 67. 𝐹 𝑇 = 𝑀 𝑇1 𝑟1 𝐹 𝑇 = 95,2 0,105 𝐹 𝑇 = 906,67𝑁 EXERCÍCIO 13.4 (Júlio) – Um conjunto de transmissão por correias possui na polia movida um diâmetro d2 = 200mm, e sabendo que a polia motora de diâmetro d1 = 100mm está acoplada a um motor com potência de 1/2cv que gira a 1750 rpm. Como segue na imagem abaixo. FIGURA 48 FONTE: http://www.blogdaengenharia.com/wp-content/uploads/2013/05/PoliaseCorreias.pdf Determine: a) Velocidade angular da polia (1) [ω1] 𝜔1 = 𝑛𝜋 30 𝜔1 = 1750𝜋 30 𝜔1 = 58,33𝜋 𝑟𝑎𝑑/𝑠 b) Frequência da polia (1) [f1] 𝑓1 = 𝑛1 60 𝑓1 = 1750 60 𝑓1 = 29,166 𝐻𝑧 c) Torque da polia (1) [MT1] Antes devo transformar a pot6encia de CV para W.
  • 68. 𝑃𝐶𝑉 = 𝑃 𝑊 735,5 𝑃 𝑊 = 𝑃𝐶𝑉. 735,5 𝑃 𝑊 = 0.5𝑥735,5 𝑃 𝑊 = 367,75𝑊 𝑀 𝑇1 = 𝑃 𝜔1 𝑀 𝑇1 = 367,75 58,33𝜋 𝑀 𝑇1 = 2,01𝑁𝑚 d) Velocidade angular da polia (2) [ω2] 𝜔2 = 𝑑1. 𝜔1 𝑑2 𝜔2 = 100𝑥58,33𝜋 200 𝜔2 = 29,165𝜋 𝑟𝑎𝑑/𝑠 e) Frequência da polia (2) [f2] 𝑓2 = 𝜔2 2𝜋 𝑓2 = 29,165𝜋 2𝜋 𝑓2 = 14,58 𝐻𝑧 f) Torque da polia (2) [MT2] 𝑀 𝑇1 = 𝑃 𝜔2 𝑀 𝑇1 = 367,75 29,165𝜋 𝑀 𝑇1 = 4,01𝑁𝑚 g) Rotação da polia (2) [n2] 𝑛2 = 60𝑓2 𝑛2 = 60𝑥14,58 𝑛2 = 874,8 𝑟𝑝𝑚 h) Relação de transmissão [i] 𝑖 = 𝑑2 𝑑1
  • 69. 𝑖 = 200 100 𝑖 = 2 i) Velocidade periférica da transmissão [VP] A velocidade periférica da transmissão é a mesma da polia (1) com a da polia (2), e sempre utilizar o raio em metros, portanto podemos utilizar: 𝑉𝑃 = 𝜔1. 𝑟1 = 𝜔2. 𝑟2 𝑉𝑃 = 𝜔1. 𝑟1 𝑉𝑃 = 58,33𝜋𝑥0,1 𝑉𝑃 = 5,833𝜋 𝑚/𝑠 𝑉𝑃 = 18,33 𝑚/𝑠 j) Força tangencial [FT] 𝐹 𝑇 = 𝑀 𝑇1 𝑟1 = 𝑀 𝑇2 𝑟2 𝐹 𝑇 = 𝑀 𝑇1 𝑟1 𝐹 𝑇 = 2,01 0,1 𝐹 𝑇 = 20,1𝑁 EXERCÍCIO 14 – A esquematização da figura representa um motor a combustão para automóvel, que aciona simultaneamente as polias da bomba D`água e do alternador. As curvas de desempenho do motor apresentam para o torque máximo a potência P = 35,3 KW (P = 48cv), atuando com rotação n = 2000 rpm. Determine para a condição de torque máximo. FIGURA 49 FONTE: MELCONIAN. SARKIS: Elementos de Máquinas – 9 Ed. Revisada. Editora Érica. Ano 2009.
  • 70. Polia do motor (1) a) Velocidade angular da polia (1) [ω1] 𝜔1 = 𝑛𝜋 30 𝜔1 = 2000𝜋 30 𝜔1 = 66,66𝜋 𝑟𝑎𝑑/𝑠 b) Frequência da polia (1) [f1] 𝑓1 = 𝑛1 60 𝑓1 = 2000 60 𝑓1 = 33,33 𝐻𝑧 c) Torque da polia (1) [MT1] 𝑀 𝑇1 = 𝑃 𝜔1 𝑀 𝑇1 = 35300 66,66𝜋 𝑀 𝑇1 = 168,56𝑁𝑚 Polia bomba D`água (2) d) Velocidade angular da polia (2) [ω2] 𝜔2 = 𝑑1. 𝜔1 𝑑2 𝜔2 = 120𝑥66,66𝜋 90 𝜔2 = 88,88𝜋 𝑟𝑎𝑑/𝑠 e) Frequência da polia (2) [f2] 𝑓2 = 𝜔2 2𝜋 𝑓2 = 88,88𝜋 2𝜋 𝑓2 = 44,44 𝐻𝑧 f) Rotação da polia (2) [n2] 𝑛2 = 60𝑓2 𝑛2 = 60𝑥44,44
  • 71. 𝑛2 = 2666,4 𝑟𝑝𝑚 g) Torque da polia (2) [MT2] 𝑀 𝑇2 = 𝑃 𝜔2 𝑀 𝑇2 = 35300 88,88𝜋 𝑀 𝑇2 = 126,42𝑁𝑚 Polia Alternador (3) h) Velocidade angular da polia (3) [ω3] 𝜔3 = 𝑑1. 𝜔1 𝑑3 𝜔3 = 120𝑥66,66𝜋 80 𝜔3 = 99,99𝜋 𝑟𝑎𝑑/𝑠 i) Frequência da polia (3) [f3] 𝑓3 = 𝜔3 2𝜋 𝑓3 = 99,99𝜋 2𝜋 𝑓3 = 49.995 𝐻𝑧 j) Rotação da polia (3) [n3] 𝑛3 = 60𝑓3 𝑛3 = 60𝑥49,995 𝑛3 = 2999,7 𝑟𝑝𝑚 k) Torque da polia (3) [MT3] 𝑀 𝑇3 = 𝑃 𝜔3 𝑀 𝑇3 = 35300 99,99𝜋 𝑀 𝑇3 = 112,37𝑁𝑚 Características da transmissão. l) Relação de transmissão [i] (Motor/Bomba D`água)
  • 72. 𝑖 = 𝑑1 𝑑2 𝑖 = 120 90 𝑖 = 1,33 m) Relação de transmissão [i] (Motor/Alternador) 𝑖 = 𝑑1 𝑑2 𝑖 = 120 80 𝑖 = 1,5 n) Força tangencial [FT] 𝐹 𝑇 = 𝑀 𝑇1 𝑟1 = 𝑀 𝑇2 𝑟2 𝐹 𝑇 = 𝑀 𝑇1 𝑟1 𝐹 𝑇 = 168,56 0,06 𝐹 𝑇 = 2809,33𝑁 o) Velocidade periférica da transmissão [VP] 𝑉𝑃 = 𝜔1. 𝑟1 = 𝜔2. 𝑟2 𝑉𝑃 = 𝜔1. 𝑟1 𝑉𝑃 = 33,33𝜋𝑥0,12 𝑉𝑃 = 3,9996𝜋 𝑚/𝑠 𝑉𝑃 = 12,56 𝑚/𝑠 EXERCÍCIO 14.1 (Eric) – Conforme os dados do exercício 5.1 (polia motora = 160mm, polia bomba d’água = 120 mm, polia alternador = 110 mm), a uma rotação de 3000 rpm do motor e o motor com potência de 110 cv, determine: ‘
  • 73. FIGURA 50 FONTE: Autor Eric Luiz Caetano. a) Velocidade angular da polia motora. (𝜔1) 𝜔1 = 𝑛 . 𝜋 30 𝜔1 = 3000 . 𝜋 30 𝜔1 = 100𝜋 𝑟𝑎𝑑 𝑠⁄ b) Frequência da polia motora (𝑓1) 𝑓1 = 𝜔1 2𝜋 𝑓1 = 100𝜋 2𝜋 = 50 𝐻𝑧 c) Torque na polia motora (𝑀𝑡) 𝑀𝑡 = 𝑃 𝜔1 𝑀𝑡 = (110 . 735,5) 100𝜋 𝑀𝑡 = 257,5 𝑁𝑚 d) Velocidade angular da polia da bomba d’água (𝜔2) 𝜔2 = 𝜔1 𝑑1 𝑑2 𝜔2 = 100𝜋 . 160 120 𝜔2 = 133,3𝜋 𝑟𝑎𝑑 𝑠⁄ e) Frequência da polia da bomba d’água (𝑓2) 𝑓2 = 𝜔2 2𝜋
  • 74. 𝑓2 = 133,3𝜋 2𝜋 𝑓2 = 66,5 𝐻𝑧 f) Torque na polia da bomba d’água (𝑀𝑡) 𝑀𝑡 = 𝑃 𝜔2 𝑀𝑡 = (110 . 735,5) 133,3𝜋 𝑀𝑡 = 193,63 𝑁𝑚 g) Rotação da polia da bomba d’água (𝑛2) 𝑛2 = 60 . 𝑓2 𝑛2 = 60 . 66,5 𝑛2 = 3390 𝑟𝑝𝑚 h) Velocidade angular da polia do alternador (𝜔3) 𝜔3 = 𝜔1 𝑑1 𝑑3 𝜔3 = 100𝜋 . 160 110 𝜔3 = 145,45𝜋 𝑟𝑎𝑑/𝑠 i) Frequência da polia do alternador (𝑓3) 𝑓3 = 𝜔3 2𝜋 𝑓3 = 145,45𝜋 2𝜋 𝑓3 = 72,72 𝐻𝑧 j) Torque na polia do alternador (𝑀𝑡) 𝑀𝑡 = 𝑃 𝜔3 𝑀𝑡 = (110 . 735,5) 145,45𝜋 𝑀𝑡 = 177,06 𝑁𝑚 k) Rotação da polia do alternador () 𝑛3 = 60 . 𝑓3 𝑛3 = 60 . 72,72 𝑛3 = 4363,2 𝑟𝑝𝑚
  • 75. EXERCICIO 14.2 (Felipe) – A figura abaixo mostra a esquematização de um motor a Diesel de um automóvel de porte médio, que aciona simultaneamente as polias da bomba D`água e do alternador. As curvas de desempenho do motor apresentam para o torque máximo a potência P = 50 KW (P = 68cv), atuando com rotação n = 3000 rpm. Determine para a condição de torque máximo. FIGURA 51 FONTE: MELCONIAN. SARKIS: Elementos de Máquinas – 9 Ed. Revisada. Editora Érica. Ano 2009. COM ALTERAÇÃO DO AUTOR. Polia do motor (1) a) Velocidade angular da polia (1) (ω1) 𝜔1 = 𝑛𝜋 30 𝜔1 = 3000𝜋 30 𝜔1 = 100𝜋 𝑟𝑎𝑑/𝑠 b) Frequência da polia (1) (f1) 𝑓1 = 𝑛1 60 𝑓1 = 3000 60 𝑓1 = 50 𝐻𝑧 c) Torque da polia (1) (MT1) 𝑀 𝑇1 = 𝑃 𝜔1 𝑀 𝑇1 = 50000 100𝜋 𝑀 𝑇1 = 159,15𝑁𝑚
  • 76. Polia bomba D`água (2) d) Velocidade angular da polia (2) (ω2) 𝜔2 = 𝑑1. 𝜔1 𝑑2 𝜔2 = 140𝑥100𝜋 110 𝜔2 = 127,27𝜋 𝑟𝑎𝑑/𝑠 e) Frequência da polia (2) (f2) 𝑓2 = 𝜔2 2𝜋 𝑓2 = 127,27𝜋 2𝜋 𝑓2 = 63,63 𝐻𝑧 f) Rotação da polia (2) (n2) 𝑛2 = 60𝑓2 𝑛2 = 60𝑥63,63 𝑛2 = 3818𝑟𝑝𝑚 g) Torque da polia (2) (MT2) 𝑀 𝑇2 = 𝑃 𝜔2 𝑀 𝑇2 = 50000 127,27𝜋 𝑀 𝑇2 = 125,06𝑁𝑚 Polia Alternador (3) h) Velocidade angular da polia (3) (ω3) 𝜔3 = 𝑑1. 𝜔1 𝑑3 𝜔3 = 140𝑥100𝜋 100 𝜔3 = 140𝜋 𝑟𝑎𝑑/𝑠 i) Frequência da polia (3) (f3) 𝑓3 = 𝜔3 2𝜋 𝑓3 = 140𝜋 2𝜋
  • 77. 𝑓3 = 70 𝐻𝑧 j) Rotação da polia (3) (n3) 𝑛3 = 60𝑓3 𝑛3 = 60𝑥70 𝑛3 = 4200 𝑟𝑝𝑚 k) Torque da polia (3) (MT3) 𝑀 𝑇3 = 𝑃 𝜔3 𝑀 𝑇3 = 50000 140𝜋 𝑀 𝑇3 = 113,68𝑁𝑚 Características da transmissão. l) Relação de transmissão (i) (Motor/Bomba D`água) 𝑖 = 𝑑1 𝑑2 𝑖 = 140 110 𝑖 = 1,27 m) Relação de transmissão (i) (Motor/Alternador) 𝑖 = 𝑑1 𝑑3 𝑖 = 140 100 𝑖 = 1,4 n) Força tangencial (FT) 𝐹 𝑇 = 𝑀 𝑇1 𝑟1 𝑜𝑢 𝑀 𝑇2 𝑟2 𝐹 𝑇 = 𝑀 𝑇1 𝑟1 𝐹 𝑇 = 159,15 0,07 𝐹 𝑇 = 2273,6𝑁 o) Velocidade periférica da transmissão (VP)
  • 78. 𝑉𝑃 = 𝜔1. 𝑟1 𝑜𝑢 𝜔2. 𝑟2 𝑉𝑃 = 𝜔1. 𝑟1 𝑉𝑃 = 100𝜋𝑥0,07 𝑉𝑃 = 7𝜋 𝑚/𝑠 𝑉𝑃 = 21,99 𝑚/𝑠 EXERCÍCIO 14.3 (Helena) - Um motor à combustão aciona simultaneamente as polias da bomba D’água e do alternador. As curvas de desempenho do motor apresentam para o torque máximo a potência P = 25,5 KW, atuando com rotação n = 1720 rpm. Dados: d1 (motor) = 100mm d2 (bomba) = 80mm d3 (alternador) = 75mm Determine para a condição de torque máximo. Polia do motor (1) a) Velocidade angular da polia (1) [ω1] 𝜔1 = 𝑛𝜋 30 𝜔1 = 1720𝜋 30 𝜔1 = 57,33𝜋 𝑟𝑎𝑑/𝑠 b) Frequência da polia (1) [f1] 𝑓1 = 𝑛1 60 𝑓1 = 1720 60 𝑓1 = 28,66 𝐻𝑧 c) Torque da polia (1) [MT1] 𝑀 𝑇1 = 𝑃 𝜔1 𝑀 𝑇1 = 25500 57,33𝜋 𝑀 𝑇1 = 141,58𝑁𝑚 Polia bomba D`água (2) d) Velocidade angular da polia (2) [ω2] 𝜔2 = 𝑑1. 𝜔1 𝑑2
  • 79. 𝜔2 = 100𝑥57,33𝜋 80 𝜔2 = 71,66𝜋 𝑟𝑎𝑑/𝑠 e) Frequência da polia (2) [f2] 𝑓2 = 𝜔2 2𝜋 𝑓2 = 71,66𝜋 2𝜋 𝑓2 = 35,83 𝐻𝑧 f) Rotação da polia (2) [n2] 𝑛2 = 60𝑓2 𝑛2 = 60𝑥35,83 𝑛2 = 2149,875 𝑟𝑝𝑚 g) Torque da polia (2) [MT2] 𝑀 𝑇2 = 𝑃 𝜔2 𝑀 𝑇2 = 25500 71,66𝜋 𝑀 𝑇2 = 113,27𝑁𝑚 Polia Alternador (3) h) Velocidade angular da polia (3) [ω3] 𝜔3 = 𝑑1. 𝜔1 𝑑3 𝜔3 = 100𝑥57,33𝜋 75 𝜔3 = 76,44𝜋 𝑟𝑎𝑑/𝑠 i) Frequência da polia (3) [f3] 𝑓3 = 𝜔3 2𝜋 𝑓3 = 76,44𝜋 2𝜋 𝑓3 = 38,22 𝐻𝑧 j) Rotação da polia (3) [n3] 𝑛3 = 60𝑓3 𝑛3 = 60𝑥38,22
  • 80. 𝑛3 = 2293,2 𝑟𝑝𝑚 k) Torque da polia (3) [MT3] 𝑀 𝑇3 = 𝑃 𝜔3 𝑀 𝑇3 = 25500 76,44𝜋 𝑀 𝑇3 = 106,19𝑁𝑚 Características da transmissão. l) Relação de transmissão [i] (Motor/Bomba D`água) 𝑖 = 𝑑1 𝑑2 𝑖 = 100 80 𝑖 = 1,25 m) Relação de transmissão [i] (Motor/Alternador) 𝑖 = 𝑑1 𝑑3 𝑖 = 100 75 𝑖 = 1,33 n) Força tangencial [FT] 𝐹 𝑇 = 𝑀 𝑇1 𝑟1 = 𝑀 𝑇2 𝑟2 𝐹 𝑇 = 𝑀 𝑇1 𝑟1 𝐹 𝑇 = 141,58 0,05 𝐹 𝑇 = 2831,6𝑁 o) Velocidade periférica da transmissão [VP] 𝑉𝑃 = 𝜔1. 𝑟1 = 𝜔2. 𝑟2 𝑉𝑃 = 𝜔1. 𝑟1 𝑉𝑃 = 57,33𝜋𝑥0,05 𝑉𝑃 = 2,8665𝜋 𝑚/𝑠 𝑉𝑃 = 9 𝑚/𝑠
  • 81. EXERCÍCIO 14.4 (Júlio) – A esquematização da figura abaixo representa um conjunto de engrenagens, acionadas por um motor que tem como suas curvas de desempenho máximo a potência de P = 47KW para um torque máximo e com uma rotação de 2830 rpm. Este motor esta acoplada a engrenagem (C) e aciona simultaneamente as engrenagens (B) e (A). As características da engrenagem são: Pinhão (C): ZC = 8 dentes, M = 2mm (Módulo). Engrenagem (B): ZB = 11 dentes, M = 2mm (Módulo). Engrenagem (A): ZA = 15 dentes, M = 2mm (Módulo). FIGURA 52 FONTE:http://2.bp.blogspot.com/_4zd06fOobnY/TUcQnhg0FfI/AAAAAAAAAmM/fQHW2wXYFD0/s16 00/denovo.png Determine para a condição de torque máximo. Pinhão motor (C) a) Velocidade angular do pinhão (C) [ωC] 𝜔 𝐶 = 𝑛𝜋 30 𝜔 𝐶 = 2830𝜋 30 𝜔 𝐶 = 94,33𝜋 𝑟𝑎𝑑/𝑠 b) Frequência do pinhão (C) [fC] 𝑓𝐶 = 𝑛 𝐶 60 𝑓𝐶 = 2830 60 𝑓𝐶 = 47,17 𝐻𝑧 c) Torque do pinhão (C) [MTC] 𝑀 𝑇 𝐶 = 𝑃 𝜔 𝐶
  • 82. 𝑀 𝑇 𝐶 = 47000 94,33𝜋 𝑀 𝑇 𝐶 = 158,6𝑁𝑚 Engrenagem (B) d) Velocidade angular da engrenagem (B) [ωB] Para encontrar velocidade angular, antes devo encontrar os diâmetros das engrenagens C e B. 𝑑 𝐶 = 𝑀. 𝑍 𝐶 𝑑 𝐶 = 2𝑥8 𝑑 𝐶 = 16𝑚𝑚 𝑑 𝐵 = 𝑀. 𝑍 𝐵 𝑑 𝐵 = 2𝑥11 𝑑 𝐵 = 22𝑚𝑚 𝜔 𝐵 = 𝑑 𝐶. 𝜔 𝐶 𝑑 𝐵 𝜔 𝐵 = 16𝑥94,33𝜋 22 𝜔 𝐵 = 68,6𝜋 𝑟𝑎𝑑/𝑠 e) Frequência da engrenagem (B) [fB] 𝑓𝐵 = 𝜔 𝐵 2𝜋 𝑓𝐵 = 68,6𝜋 2𝜋 𝑓𝐵 = 34,3 𝐻𝑧 f) Rotação da engrenagem (B) [nB] 𝑛 𝐵 = 60𝑓𝐵 𝑛 𝐵 = 60𝑥34,3 𝑛 𝐵 = 2058,1 𝑟𝑝𝑚 g) Torque da engrenagem (B) [MTB] 𝑀 𝑇 𝐵 = 𝑃 𝜔 𝐵 𝑀 𝑇 𝐵 = 47000 68,6𝜋 𝑀 𝑇 𝐵 = 218,08𝑁𝑚 Engrenagem (A)
  • 83. h) Velocidade angular da engrenagem (A) [ωA] 𝑑 𝐴 = 𝑀. 𝑍 𝐴 𝑑 𝐴 = 2𝑥15 𝑑 𝐴 = 30𝑚𝑚 𝜔 𝐴 = 𝑑 𝐶. 𝜔 𝐶 𝑑 𝐴 𝜔 𝐴 = 16𝑥94,33𝜋 30 𝜔 𝐴 = 50,31𝜋 𝑟𝑎𝑑/𝑠 i) Frequência da engrenagem (A) [fA] 𝑓𝐴 = 𝜔 𝐴 2𝜋 𝑓𝐴 = 50,31𝜋 2𝜋 𝑓𝐴 = 25,16 𝐻𝑧 j) Rotação da engrenagem (A) [nA] 𝑛 𝐴 = 60𝑓𝐴 𝑛 𝐴 = 60𝑥25,16 𝑛 𝐴 = 1509,3 𝑟𝑝𝑚 k) Torque da engrenagem (A) [MTA] 𝑀 𝑇 𝐴 = 𝑃 𝜔 𝐴 𝑀 𝑇 𝐴 = 47000 50,31𝜋 𝑀 𝑇 𝐴 = 297,37𝑁𝑚 Características da transmissão. l) Relação de transmissão [i] [Pinhão (C)/Engrenagem (B)] 𝑖 = 𝑍 𝐵 𝑍 𝐶 𝑖 = 11 8 𝑖 = 1,375 m) Relação de transmissão [i] [Pinhão (C)/Engrenagem (A)]
  • 84. 𝑖 = 𝑍 𝐴 𝑍 𝐶 𝑖 = 15 8 𝑖 = 1,875 n) Força tangencial [FT] 𝐹 𝑇 = 𝑀 𝑇1 𝑟1 = 𝑀 𝑇2 𝑟2 𝐹 𝑇 = 𝑀 𝑇 𝐶 𝑟𝐶 𝐹 𝑇 = 158,6 0,016 𝐹 𝑇 = 9912,5𝑁 o) Velocidade periférica da transmissão [VP] 𝑉𝑃 = 𝜔 𝐶. 𝑟𝐶 = 𝜔 𝐵. 𝑟𝐵 𝑉𝑃 = 𝜔 𝐶. 𝑟𝐶 𝑉𝑃 = 94,33𝜋𝑥0,016 𝑉𝑃 = 1,5093𝜋 𝑚/𝑠 𝑉𝑃 = 4,74 𝑚/𝑠 RELAÇÃO DE TRANSMISSÃO POR ENGRENAGENS EXERCÍCIO 15 – A transmissão por engrenagem é acionada por meio do pinhão (1) acoplado a um motor elétrico de IV polos com potência P = 15KW, (P = 20cv) e rotação n 1720 rpm. As características da engrenagem são: Pinhão (1): Z1 = 24 dentes, M = 4mm (Módulo). Coroa (2): Z2 = 73 dentes, M = 4mm (Módulo) FIGURA 53 FONTE: MELCONIAN. SARKIS: Elementos de Máquinas – 9 Ed. Revisada. Editora Érica. Ano 2009. Determinar para a transmissão:
  • 85. Pinhão (1). a) Velocidade angular do pinhão (1) [ω1] 𝜔1 = 𝑛𝜋 30 𝜔1 = 1720𝜋 30 𝜔1 = 57,33𝜋 𝑟𝑎𝑑/𝑠 b) Frequência da polia (1) [f1] 𝑓1 = 𝑛1 60 𝑓1 = 1720 60 𝑓1 = 28,67 𝐻𝑧 c) Torque da polia (1) [MT1] 𝑀 𝑇1 = 𝑃 𝜔1 𝑀 𝑇1 = 15000 57,33𝜋 𝑀 𝑇1 = 83,28𝑁𝑚 Coroa (2). d) Velocidade angular da polia (2) [ω2] 𝜔2 = 𝑍1. 𝜔1 𝑍2 𝜔2 = 24𝑥57,33𝜋 73 𝜔2 = 18,85𝜋 𝑟𝑎𝑑/𝑠 e) Frequência da polia (2) [f2] 𝑓2 = 𝜔2 2𝜋 𝑓2 = 18,85𝜋 2𝜋 𝑓2 = 9,42 𝐻𝑧 f) Rotação da polia (2) [n2] 𝑛2 = 60𝑓2
  • 86. 𝑛2 = 60𝑥9,42 𝑛2 = 565,44 𝑟𝑝𝑚 g) Torque da polia (2) [MT2] 𝑀 𝑇2 = 𝑃 𝜔2 𝑀 𝑇2 = 15000 18,85𝜋 𝑀 𝑇2 = 253,3𝑁𝑚 Características da transmissão. h) Velocidade periférica da transmissão [VP] 𝑉𝑃 = 𝜔1. 𝑟1 Ou 𝑉𝑃 = 𝜔2. 𝑟2 𝑟1 = 𝑑1 2 𝑑1 = 𝑀. 𝑍1 𝑑1 = 𝑀. 𝑍1 𝑑1 = 4𝑥24 𝑑1 = 96𝑚𝑚 𝑟1 = 𝑑1 2 𝑟1 = 96 2 𝑟1 = 48𝑚𝑚 𝑟1 = 0.048𝑚 𝑉𝑃 = 𝜔1. 𝑟1 𝑉𝑃 = 57,33𝜋𝑥0,048 𝑉𝑃 = 2,75𝜋 𝑚/𝑠 𝑉𝑃 = 8,645 𝑚/𝑠 i) Força tangencial [FT] 𝐹 𝑇 = 𝑀 𝑇1 𝑟1 𝐹 𝑇 = 83,28 0,048 𝐹 𝑇 = 1735𝑁 j) Relação transmissão [i]
  • 87. 𝑖 = 𝑍2 𝑍1 𝑖 = 73 24 𝑖 = 3,04 EXERCÍCIO 15.1 (Eric) – Uma transmissão por engrenagens ampliadora de velocidade é composta por um motor elétrico com potência de 12 cv (8826 W), que gira uma coroa com 52 dentes com uma rotação de 1800 rpm. Essa, por sua vez, movimenta um pinhão com 27 dentes. Ambos os módulos das engrenagens são de 3 mm. Para essa transmissão, determine: a) Velocidade angular da coroa (1) 1 = 𝑛 . 𝜋 30 1 = 1800 . 𝜋 30 1 = 60𝜋 𝑟𝑎𝑑 𝑠⁄ b) Frequência da coroa (𝑓1) 𝑓1 = 𝜔1 2𝜋 𝑓1 = 60𝜋 2𝜋 𝑓1 = 30 𝐻𝑧 c) Torque na coroa (𝑀𝑡) 𝑀𝑡 = 𝑃 1 𝑀𝑡 = 8826 60𝜋 𝑀𝑡 = 46,82 𝑁𝑚 d) Velocidade angular do pinhão (2) (Z= número de dentes de cada engrenagem) 2 = 1 . 𝑍1 𝑍2 2 = 60𝜋 . 52 27 2 = 115,5 𝜋 𝑟𝑎𝑑 𝑠⁄ e) Frequência do pinhão (𝑓2)
  • 88. 𝑓2 = 𝜔2 2𝜋 𝑓2 = 115,5 𝜋 2𝜋 𝑓2 = 57,75 𝐻𝑧 f) Torque no pinhão (𝑀𝑡2) 𝑀𝑡2 = 𝑀𝑡1 . 𝑍2 𝑍1 𝑀𝑡2 = 46,82 . 27 52 𝑀𝑡2 = 24,31 𝑁𝑚 g) Rotação do pinhão (𝑛2) 𝑛2 = 60 . 𝑓2 𝑛2 = 60 . 57,75 𝑛2 = 3465 𝑟𝑝𝑚 h) Relação de transmissão (𝑖) 𝑖 = 𝑍2 𝑍1 𝑖 = 27 52 𝑖 = 0,51 i) Força tangencial (𝑑01) 𝑑01 = 𝑚 . 𝑍1 𝑑01 = 3 . 52 𝑑01 = 156 𝑚𝑚 𝑑01 = 0,156 𝑚 𝐹 𝑇 = 2 . 𝑀 𝑇1 𝑑01 𝐹 𝑇 = 2 . 46,82 0,156 𝐹 𝑇 = 600,25 𝑁 j) Velocidade periférica (𝑟01) 𝑟01 = 𝑑01 2 𝑟01 = 0,156 2
  • 89. 𝑟01 = 0,078 𝑚 𝑉𝑝 = 𝜔1. 𝑟01 𝑉𝑝 = 60𝜋 . 0,078 𝑉𝑝 = 14,70 𝑚/𝑠 EXERCICIO 15.2 (Felipe) - Em uma determinada transmissão por engrenagem, é feito o acionamento por meio do pinhão (1) acoplado a um motor com potência P = 18KW, (P = 25cv) e rotação n 2100 rpm. As características da engrenagem são: Pinhão (1): Z1 = 28 dentes, M = 6mm (Módulo). Coroa (2): Z2 = 79 dentes, M = 6mm (Módulo) FIGURA 54 FONTE: http://www.portalcad.com/blog/index.php/10-dicas-e-truques. Determinar para a transmissão: Pinhão (1). a) Velocidade angular do pinhão (1) (ω1) 𝜔1 = 𝑛𝜋 30 𝜔1 = 2100𝜋 30 𝜔1 = 70𝜋 𝑟𝑎𝑑/𝑠 b) Frequência da polia (1) (𝑓1) 𝑓1 = 𝑛1 60 𝑓1 = 2100 60 𝑓1 = 35 𝐻𝑧 c) Torque da polia (1) (MT1) 𝑀 𝑇1 = 𝑃 𝜔1
  • 90. 𝑀 𝑇1 = 18000 57,33𝜋 𝑀 𝑇1 = 81,85𝑁𝑚 Coroa (2). d) Velocidade angular da polia (2) (ω2) 𝜔2 = 𝑍1. 𝜔1 𝑍2 𝜔2 = 28𝑥70𝜋 79 𝜔2 = 24,81𝜋 𝑟𝑎𝑑/𝑠 e) Frequência da polia (2) (𝑓2) 𝑓2 = 𝜔2 2𝜋 𝑓2 = 24,81𝜋 2𝜋 𝑓2 = 12,4 𝐻𝑧 f) Rotação da polia (2) (n2) 𝑛2 = 60𝑓2 𝑛2 = 60𝑥12,4 𝑛2 = 744,3 𝑟𝑝𝑚 g) Torque da polia (2) (MT2) 𝑀 𝑇2 = 𝑃 𝜔2 𝑀 𝑇2 = 18000 24,81𝜋 𝑀 𝑇2 = 230,9𝑁𝑚 Características da transmissão. h) Velocidade periférica da transmissão (VP) 𝑉𝑃 = 𝜔1. 𝑟1 Ou 𝑉𝑃 = 𝜔2. 𝑟2 𝑟1 = 𝑑1 2 𝑑1 = 𝑀. 𝑍1 𝑑1 = 𝑀. 𝑍1
  • 91. 𝑑1 = 6𝑥28 𝑑1 = 168𝑚𝑚 𝑟1 = 𝑑1 2 𝑟1 = 168 2 𝑟1 = 84𝑚𝑚 𝑟1 = 0.084𝑚 𝑉𝑃 = 𝜔1. 𝑟1 𝑉𝑃 = 70𝜋𝑥0,084𝑚 𝑉𝑃 = 5,88𝜋 𝑚/𝑠 𝑉𝑃 = 18,47 𝑚/𝑠 i) Força tangencial (FT) 𝐹 𝑇 = 𝑀 𝑇1 𝑟1 𝐹 𝑇 = 81,85 0,084 𝐹 𝑇 = 974,4𝑁 j) Relação transmissão (i) 𝑖 = 𝑍2 𝑍1 𝑖 = 79 28 𝑖 = 2,82 EXERCÍCIO 15.3 (Helena) – A transmissão por engrenagem é acionada por meio do pinhão (1) acoplado a um motor elétrico de IV polos com potência P = 20KW e rotação n=2000 rpm. As características da engrenagem são: Pinhão (1): Z1 = 28 dentes, M = 6mm (Módulo). Coroa (2): Z2 = 79 dentes, M = 6mm (Módulo) Determinar para a transmissão: Pinhão (1). a) Velocidade angular do pinhão (1) [ω1] 𝜔1 = 𝑛𝜋 30 𝜔1 = 2000𝜋 30 𝜔1 = 66,67𝜋 𝑟𝑎𝑑/𝑠
  • 92. b) Frequência da polia (1) [f1] 𝑓1 = 𝑛1 60 𝑓1 = 2000 60 𝑓1 = 33,33 𝐻𝑧 c) Torque da polia (1) [MT1] 𝑀 𝑇1 = 𝑃 𝜔1 𝑀 𝑇1 = 20000 66,67𝜋 𝑀 𝑇1 = 95,49𝑁𝑚 Coroa (2). d) Velocidade angular da polia (2) [ω2] 𝜔2 = 𝑍1. 𝜔1 𝑍2 𝜔2 = 28𝑥66,67𝜋 79 𝜔2 = 23,63𝜋 𝑟𝑎𝑑/𝑠 e) Frequência da polia (2) [f2] 𝑓2 = 𝜔2 2𝜋 𝑓2 = 23,63𝜋 2𝜋 𝑓2 = 11,81𝐻𝑧 f) Rotação da polia (2) [n2] 𝑛2 = 60𝑓2 𝑛2 = 60𝑥11,81 𝑛2 = 708,9 𝑟𝑝𝑚 g) Torque da polia (2) [MT2] 𝑀 𝑇2 = 𝑃 𝜔2 𝑀 𝑇2 = 20000 23,63𝜋 𝑀 𝑇2 = 269,4𝑁𝑚
  • 93. Características da transmissão. h) Velocidade periférica da transmissão [VP] 𝑉𝑃 = 𝜔1. 𝑟1 Ou 𝑉𝑃 = 𝜔2. 𝑟2 𝑟1 = 𝑑1 2 𝑑1 = 𝑀. 𝑍1 𝑑1 = 𝑀. 𝑍1 𝑑1 = 6𝑥28 𝑑1 = 168𝑚𝑚 𝑟1 = 𝑑1 2 𝑟1 = 168 2 𝑟1 = 84𝑚𝑚 𝑟1 = 0.084𝑚 𝑉𝑃 = 𝜔1. 𝑟1 𝑉𝑃 = 66,67𝜋𝑥0,084 𝑉𝑃 = 5,60𝜋 𝑚/𝑠 𝑉𝑃 = 17,59 𝑚/𝑠 i) Força tangencial [FT] 𝐹 𝑇 = 𝑀 𝑇1 𝑟1 𝐹 𝑇 = 95,49 0,084 𝐹 𝑇 = 1136,785𝑁 j) Relação transmissão [i] 𝑖 = 𝑍2 𝑍1 𝑖 = 79 28 𝑖 = 2,82 EXERCÍCIO 15.4 (Júlio) – Na imagem abaixo é representado um sistema de engrenagens onde a engrenagem (1) pinhão é acionada por um motor de P = 2cv, por meio de um acoplamento e a rotação é de n = 1000 rpm. As características das engrenagens
  • 94. são: Pinhão (1): Z1 = 36 dentes, M = 2mm (Módulo). Coroa (2): Z2 = 91 dentes, M = 2mm (Módulo). FIGURA 55 FONTE: http://www.etepiracicaba.org.br/cursos/apostilas/mecanica/1_ciclo/tec_projeto1.pdf Determinar para a transmissão: Pinhão (1). a) Velocidade angular do pinhão (1) [ω1] 𝜔1 = 𝑛𝜋 30 𝜔1 = 1000𝜋 30 𝜔1 = 33,33𝜋 𝑟𝑎𝑑/𝑠 b) Frequência da polia (1) [f1] 𝑓1 = 𝑛1 60 𝑓1 = 1000 60 𝑓1 = 16,67 𝐻𝑧 c) Torque da polia (1) [MT1] Antes devo transformar a pot6encia de CV para W. 𝑃𝐶𝑉 = 𝑃 𝑊 735,5 𝑃 𝑊 = 𝑃𝐶𝑉. 735,5 𝑃 𝑊 = 2𝑥735,5 𝑃 𝑊 = 1471𝑊 𝑀 𝑇1 = 𝑃 𝜔1 𝑀 𝑇1 = 1471 33,33𝜋
  • 95. 𝑀 𝑇1 = 14,05𝑁𝑚 Coroa (2). d) Velocidade angular da polia (2) [ω2] 𝜔2 = 𝑍1. 𝜔1 𝑍2 𝜔2 = 36𝑥33,33𝜋 91 𝜔2 = 13,18𝜋 𝑟𝑎𝑑/𝑠 e) Frequência da polia (2) [f2] 𝑓2 = 𝜔2 2𝜋 𝑓2 = 13,18𝜋 2𝜋 𝑓2 = 6,59 𝐻𝑧 f) Rotação da polia (2) [n2] 𝑛2 = 60𝑓2 𝑛2 = 60𝑥6,59 𝑛2 = 395,4 𝑟𝑝𝑚 g) Torque da polia (2) [MT2] 𝑀 𝑇2 = 𝑃 𝜔2 𝑀 𝑇2 = 1471 13,18𝜋 𝑀 𝑇2 = 35,53𝑁𝑚 Características da transmissão. h) Velocidade periférica da transmissão [VP] 𝑉𝑃 = 𝜔1. 𝑟1 = 𝜔2. 𝑟2 𝑟1 = 𝑑1 2 𝑑1 = 𝑀. 𝑍1 𝑑1 = 𝑀. 𝑍1 𝑑1 = 2𝑥36 𝑑1 = 72𝑚𝑚
  • 96. 𝑟1 = 𝑑1 2 𝑟1 = 72 2 𝑟1 = 36𝑚𝑚 𝑟1 = 0.036𝑚 𝑉𝑃 = 𝜔1. 𝑟1 𝑉𝑃 = 33,33𝜋𝑥0,036 𝑉𝑃 = 1,19988𝜋 𝑚/𝑠 𝑉𝑃 = 3,77 𝑚/𝑠 i) Força tangencial [FT] 𝐹 𝑇 = 𝑀 𝑇1 𝑟1 𝐹 𝑇 = 14,05 0,036 𝐹 𝑇 = 390,3𝑁 j) Rolação transmissão [i] 𝑖 = 𝑍2 𝑍1 𝑖 = 91 36 𝑖 = 2,5
  • 97. REFERÊNCIAS ALIBABA. Products. Disponível em: <http://portuguese.alibaba.com/product-gs/hot- new-products-for-2015-kitchen-outer-rotor-450mm-ac-axial-fan-60210360658.html>. Acesso em: 03 de abr. 2015. BLOG DA ENGENHARIA. Polias e correias. Disponível em: <http://www.blogdaengenharia.com/wp- content/uploads/2013/05/PoliaseCorreias.pdf>. Acesso em: 03 de abr. 2015. BLOGSPOT. Disponível em: <http://2.bp.blogspot.com/_4zd06fOobnY/TUcQnhg0FfI/AAAAAAAAAmM/fQHW2wX YFD0/s1600/denovo.png>. Acesso em: 03 de abr. 2015. BLOGSPOT. Formação piloto. Disponível em: <http://formacaopiloto.blogspot.com.br/2014_06_01_archive.html>. Acesso em: 03 de abr. 2015. CASA DAS POLIAS. Disponível em: <http://www.casadaspolias.com.br/produtos/especificacoes-tecnicas.html>. Acesso em: 03 de abr. 2015. CEPA.IF.USP. Energia. Hidráulica. Disponível em: <http://www.cepa.if.usp.br/energia/energia1999/Grupo2B/Hidraulica/roda.htm>. Acesso em: 03 de abr. 2015. CLIP ART LOGO. Premiun. Disponível em: <http://pt.clipartlogo.com/premium/detail/car-or-truck-tire-line-art_109442720.html> Acesso em: 03 de abr. 2015. EDUCAÇÃO MG GOV. Sistema CRV. Disponível em: <http://crv.educacao.mg.gov.br/sistema_crv/index.aspx?ID_OBJETO=58360&tipo=o b&cp=780031&cb=&n1=&n2=M%EF%BF%BDdulos%20Did%EF%BF%BDticos&n3= Ensino%20M%EF%BF%BDdio&n4=F%EF%BF%BDsica&b=s>. Acesso em: 03 de abr. 2015. ETEPIRACICABA ORG. Cursos. Apostila mecânica. Disponível em: <http://www.etepiracicaba.org.br/cursos/apostilas/mecanica/1_ciclo/tec_projeto1.pdf >. Acesso em: 03 de abr. 2015. FISICA VESTIBULAR. Disponível em: <http://fisicaevestibular.com.br/exe_din_15.htm>. Acesso em: 03 de abr. 2015. GP MOTORS BRASIL. Disponível em: <http://www.gpmotorsbrasil.com.br/info1.html>. Acesso em: 03 de abr. 2015.
  • 98. PAULUZZI. Alvenaria. Disponível em: <http://www.pauluzzi.com.br/alvenaria.php?PHPSESSID=ccd0dd0c90aa9901b2a2e4 9d3182897c>. Acesso em: 03 de abr. 2015. PORTAL CAD. Blog. 10 truques e dicas. Disponível em: <http://www.portalcad.com/blog/index.php/10-dicas-e-truques>. Acesso em: 03 de abr. MELCONIAN. SARKIS: Elementos de Máquinas – 9 Ed. Revisada. Editora Érica. Ano 2009. SEGURANCA NA CONSTRUCAO. Disponível em: <http://seguranca-na- construcao.dashofer.pt/?s=modulos&v=capitulo&c=7655>. Acesso em: 03 de abr. 2015. SLIDESHARE. Centro Apoio. Exercício equilíbrio corpo rígido. Disponível em: <http://pt.slideshare.net/CentroApoio/exercequilibrio-corpo-rigido>. Acesso em: 03 de abr. 2015. VDL UFC. Solar. Aula. Disponível em: <http://www.vdl.ufc.br/solar/aula_link/lfis/semestre01/Fisica_I/Aula_03/02.html>. Acesso em: 03 de abr. 2015. VEGA MAQUINAS. Retifica para torno. Disponível em: <http://www.vegamaquinas.com.br/retifica-para-torno-pr-1313-229827.htm> Acesso em: 03 de abr. 2015. WIKIPEDIA. Turbina aeronave. Disponível em: <http://pt.wikipedia.org/wiki/Turbina_aeron%C3%A1utica>. Acesso em: 03 de abr. 2015.